You are on page 1of 174
BO GIAO DUC VA DAO TAO TRUONG DAI HOC KHOA HOC TU NHIEN, DHQGHN * Nguyén Van Mau, Bui Cong Huan, Dang Hung Thang, Tran Nam Ding, Dang Huy Ruan MOT SO CHUYEN DE TOAN HOC CHON LOC BOI DUONG HOC SINH GIOI HA NOI - 2004 MUC LUC Loi ndéi dau vd eeeer—rs—“—irssr—sS 0 Mot sodacitnume co ban cuahamso | 7, 3 Bat phuwongstrinh) am cO,ban, mp 5d do5 0056 15 Phuong trinh ham lién quan dén tam gidc ................00.. 27 Bat phuong trinh ham lién quan dén tam gidc ................ 35 : Bat phuong trinh ham trong tam gidc d6i véi bién p,R,r ....... 41 Dé6ng dir va phuong trinh déng du ................0000. eee 50 Phucne tinh Pe eeeeer—r—“Srrrs =< errr 52 } Lién phan s6Aa ting dung ......... 6.202. e eee eee eee eee 89 Mot s6 phuong trinh Diophant phi tuyén ...................- 105 ; Phuong trinh Diophant ................ 0.200 e eee eee eee 122 Phuong phap gidi bai todn chia hét................... 0.0000 140 Loi néi dau Chuong trinh dio tao va béi hoc sinh nang khiéu todn bac phé thong hien sang nam thé 39. Gan véi vic déi méi phuong phdp day va hoc chuong trint nam nay, nam 2004, chiing ta dang tich cue chudn bi cho viee t6 chtte Ky thi « ‘Todn quéc té nim 2007 tai Viet Nam, ky niém 40 ndm Tap chi Toan hoc va‘ 30 nim cdc doi tuyén nude ta tham du céc ky thi Olympic Todn quéc té. C6 thé ndi, giao duc mii nhon phé thong da thu duge nhiing thanh tuu rye Nha nuéc ddu tu cé higu qua, x hoi thim nhan va ban b& quéc té kham ph doi tuyén Toan quéc gia tham du cc ky thi Olympic Toan quéc té c6 bé di tich mang tinh én dinh va c6 tinh ké thit. Dac biét, nm nay, Doi tuyén To gia tham dy thi Olympic Todn quéc té da dat dug thanh tich ruc rd: 4 huy vang va 2 huy chuong bac, dig thit 4 thé gidi. Nhiéu nam céc doi tuyén Te gia tham dy céc ky thi Olympic Ton quéc té giit vimg duge vi tri tir thir 4 dé (Top Ten) trén téng s6 gn 100 doi tuyén quéc gia tham gia. Tir nhiéu nam nay, Céc He va cdc Throng THPT Chuyén thudng sit dung sc cde séch gido khoa dai tri két hop véi sich gido khoa cho Hé THPT Chuyi Hoe sinh céc lép nang khiéu da tiép thu tt cdc kign thtte co ban theo thdi luc hanh do Bo GD va DT ban hanh. Hien nay, chuong trinh cai céch giéo duc dang buéc vao giai doan hoin ¢ SGK méi. Thdi uong kign thc cing nhy trat ny kién thie co ban c6 nhimg | dang ké. Cac kién thite nay dang dugc can nhac dé né vin nam trong khuén k hanh cia cdc kign thtte nang cao d6i véi cdc 6p chuyén todn. Vi lé dé, vie ti viet cic SGK cho cdc 6p nang khiéu ton chua thé tién hanh c4p bach trong t ngan, doi hdi cé sur suy ngdm va xem xét to’n dién cia cdc chuyén gia gidc cdc cO gido, thdy gido dang truc tiép gidng day c4c [dp chuyen. Duge sy cho phép cia BO GD va DT, Trung Dai Hoc Khoa Hoc Tu DHQGHN phdi hop cing véi céc chuyén gia, cdc nha khoa hoc, cdc cO gi: giéo thuge DHSPHN, DHQG TpHCM, DH Vinh, Vien Toan Hoc, Hoi Toan Noi, NXBGD, Tap Chi Todn Hoc va Tui Tré, cic Trudng THPT Chuyen, Cac va DT... t6 chic Chuong trinh béi duéng nghiép vu sau dai hoc vé cdc chuyas dutng hoe sinh gidi toan. Noi dung chinh cia chuyén dé gém hai phan: Phuong trinh, bat phuong tr trong hinh hoc va Mot s6 van dé chon loc cia sé hoc. Dé d4p img cho nhu cdu béi dudng gido vién va béi dudng hoc sinh gidi t0i in cuén Ky yéu nay nham cung cap mot s6 kién thite co ban va img dung chuyen dé Toan Phé Thong. Day cing {4 chuyén dé va bai gidng mA cdc téc gid da gidng day cho b va sinh vien cdc doi tuyén thi olympic todn quéc gia va qudc té va 1a chuyér duéng nghiep vu sau dai hoc cho céc gido vin day cdc 6p chuyén toan. Ching toi cing xin chan thinh cdm on céc ban doc cho nhimg ¥ kién d dé cudn sich ngay cang hodn chinh. T/M Ban Té Chitc GS TSKH Nguyén Van Mau MOT SO DAC TRUNG CO BAN CUA HAM SO 5 Nguyén Van Mau L1. Déc trung ham cia m6t 86 ham sé so cép 1.2. Ham s6 chuyén déi phép tinh s6 hoc vé dai sé 1.1. Dac trung him cia mot so him sé so cép ‘Trong phan nay ta néu nhiing dic tnmg ham cia mot sé ham s6 so cdp, 1 gap trong chuong trinh phé thong. Nho céc dic tumg ham nay ma ta c6 thé di dip s6 cia céc phuong trinh ham tuong tg cling nhu cé thé dé xudt dang | tuong tu img vi cdc dic rung ham do. Cc ham s6 duge xét trong phén nay thoa man diéu kién lien tuc trén toar xc dinh cia ham s6, Néu ham s6 thod man cdc dic trmg ham da cho ma khé tinh lién tuc hodc duge xéc dinh trén cdc tap réi rac thi nghiém cia phuong trin c6 thé 4 mot biéu thie hoan toan khac. 1. Ham bac nhat: f(x) = ax +b (v6i a,b £0). Dac trmg ham: EZh = f+ f) v6i moi 2,y ER. v . Ham tuyéa tinh: f(c) = az (voi a # 0). Dac trmg ham: f(a+y) = f(z) + f(y) voi moi c,yeR. w . Ham ma: f(z) =a? (v6ia>0; #1). Dac trung ham: Se+y) =S@)-f(y) voi moi zy ER. 4. Ham logarit: f( Dac trumg ham: = loga|z| (v6ia>0; #1). fay) = f(z) + f(y) voi moi z,y€ R*. 5. Ham sin: f(x) = sina. Dac trumg ham: f (3x) =3f(z) —4 P(x) véi moi x ER. a 13. . Ham cos : f(z) = cosa Dac tung ham: f(x) = 2f%(x) 1 voi moi z ER. hodc f(a+y)+ f(e-y) =2f(x)f(y) voi moi z,y ER. . Ham tang: f(z) = tge. Dac tung ham: fl@+y)= frie v6i moi 2,yERc+y 4 Cee dF eZ. . Ham cotang: f(x) = cotg x. Dac tung ham: = FLY) =1 vg noi . S(et+y)= Fal+ fq) iminveRetudknk eZ. . Ham lug thia: f(x) = 2% (v6ia ER; 2 eR). Dac trung ham: L(zy) = f(@)F(y) voi moi zy ERT. ). Ham luong gide nguge: f(«) = arcsin 2. Dac wumg ham: f(x) + f(y) = flav -y t+ yV1—2?) voi moi x.y € [-1}). . Ham huong gide nguge: f(z) = arccos:r. Dic trumg ham: f(a) + f(y) = f(zy- VQ = 2?) —y?) v6i moi x,y € [-151) Ham luong gide nguoe: f(x) = arctge. Dac trung ham: r fle) +s) =f (=) vOi moi ,y ER: ay £1. Ham long gide nguve: f(x) = arccotge. Dac trung ham: | (22) vi moi ay ERs ety ¢0 xot+y . Ham sin hyperbolic: f(e) = ze —e-*) = she. Dac trmg ham: f(8x) = 3f(x) +.4f°(@) véi moi Ham cos hyperbolic: f(a) = Dac trung ham: Het D+ Sey) = 2) fly) vi moi 2.y eR. 5 . ete . Him tang hyperbolic: f(x) = Sy = the. Dac trung ham: oe 1+ f(2)fQ) f(et+y)= voi moi z.y < R. f(z) + f(y) a "= cothe = —. eneF The . Ham cotangenchyperbolic: f(x) = Dac trung ham: 1+ F(z) fu) fE+D = Fe TQ) voi moi z,y € R. ‘Tuong ty, ta cling cé cde dic trmg ham ciia cdc him s6 sau day. . f(z) = tger, thi f(r+y) = A(x) + fu) 1 ~ f(z)fy)’ F(z) Fly) = 1 f(z) + f(y)’ f(z) + fv) f(x) = cotgex, thi f(e+y)= f(a) =cthes, th f(xt+y) = fay’ =o om = £@FO)_ ECE) 2 thi f(@+y) = F@ +i’ 1 ; fe) + Fw) - 2f(2) Fy) $@)= Toga’ Me +n) = 1-2f(@)F@) 1 . a f@)+f@)-1 f= Tyga MIC+Y= TF a@) were F(2) +f) + 2F@) Fy) ee a ele) - tm f(e+y) = fee er He) = FRE, a fe $y) = LE iene, . fe) = an thi fle ty) = Le)* Mo) — fecha ant) ms. a ay i S@ety= dBA) f(z) F(y)cha 1— f(x)E(y) . f(x) = ae, thi af (x? $y) = f(x) fly), eta\t or aty . He) = (8), wT Hoe, e-2, 34. whi f(e+y)— f(a—y) =4VF(2)Iy), 35. thi f(e+y)+ fe—y) = 2Af(z) + Fv), 36. f(v)=ca+a, thi f(et+y)t f(e@-y) =2f(z), 37. f(z) =cx, thi f(z+y)-f(e—y) = 24). 1.2. Ham sé chuyén déi phép tinh sé hoc va dai sé Trong muc nay, ta khdo s4t mot sO tinh chat co ban cia mot sO dang ham s6 thong qua cdc hé thttc him don gidn. Ta cling khdo s4t mot s6 dang ham bio toan va chuyén d6i cde tinh chat co ban cba phép tinh dai s6 nhu giao hodn, phan b6 va két hop. Bai todn 1. Xéc dinh cdc ham s6 f(x) xdc dinh va lién tc wén R thod man diéu kién f(a+y) = f(z) + fly) + f(a) fy), Va,yeR. (1) Gidi. Dat f(z) = g(x) — 1, ta thu duge g(x + y) —1 = g{z)—1+9(y) —1+ lo(z) — ly) - 1], Ve.yeR hay oz +) =9(z)9(y), Ye,y ER. (2 Do f(z) lién tuc trén R nén g(x) cing 1a ham lién tuc trén R. Suy ra (2) c6 nghié g(x) = e%,a € R va (1) c6 nghiem f(2)=e* -1,0eER. Bai toan 2. Cho ham s6 F(u,v) (u,v € R). Gia sit phuong trinh ham: f@et+y) = FIf(@), fy), VayeR (3 c6 nghiém f(x) xdc dinh va lién tuc tren R. Chimg minh rang F(u,v) 1a ham dé xing (F(u, v) = F(v,u) va 06 tinh két hop F(F(u.v),w] = Flu, F(v,w)], Yuvw € Sf. (4 Gidi. han xét ring tinh d6i ximg cia F(u,v) duge suy truc tip tir (3). Mat khéc, the (3), ta c6 S@ty+z)=fl@ty) ta =F{FIS@) SWI} YeayzeR (F va Satyt2= [e+ (y +2) = flyt2) +2] = FFP), f()] S@) = F{f(«), Fif(y). fl}. Yay. eR Tir (5) va (6) suy ra (4): F[F(u,v),w] = Flu, F(v,w)], Yu,v,w € Sf. Bai toan 3. Gia sir phuong tinh him: f(e+y) = FIF(@), fy], YeyeR voi ham sO F(u,v) (u,v € R) [A mot da thic (khéc hang), 6 nghiém f(z) xdc va lien tue (khéc hang) trén R. Chimg minh ring F(u, v) c6 dang F(u,v) = auv + but bv +e. Gidi. Gia sit F(u,v) la da thite bac m theo u va bac n theo v. Khi d6, do F(u,u xting nén m =n. Theo (4) thi F(F(u,v), w] = Flu, F(v,w)}, Yu,e.w e Sf nen vé trai 1A mot da thie bac n theo w con vé phai IA da thiic bac n? theo w. S$ rn? =nhay n=1. Vay F(u,v) c6 dang F(u,v) = auv + bu + byu +e. Do F(u,v) la da thite déi ximg nén by = by va F(u,v) = auv + but bv te. Nhan xét ring, véi F(u,v) = auv + bu + bo +c va F(u,v) thoa man diéu (4) thi Vay voi a # 0 thi eb a ac=P—bec= #0. Bay gid, ta chuyén sang xét cdc dang dac biét cla (7). Bai toan 4. Cho da thie F(u,v) = bu + bute, 640. Xéc dinh cdc ham 6, xc dinh va lien tue én R thod man diéu kien S(e+y) = FIf@) SM), Ye.yeR tite 1a f(aty) = of(x) +of(y) +e, Ye, yeER. Gidi. Nhan xét rang, néu b # 1 thi ti (9) véi y = 0, ta c6 ngay f(r) = const. Khi 1 _ b= 2 va c = 0 thi moi ham hang déu thod man (8). Khi b = 2 va c ¥ 0 thi (9) vo nghiém. C4c trudng hop khac (6 # 1,0 # 5 thi nghiem cia (9) 18 f(a) = Xé trudng hop b = 1. Khi dé (9) c6 dang ¢ 1-20 S(z+y)=fe)+fy)te veyeR va phuong trinh ham nay c6 nghiém f(z) = ax —c. ed Bai ton 5. Cho da thie F(u,v) = auv + bu + bv + ham s6 f(z) x4c dinh va lién uc en R thod man diéu kien , #0. Xée dinh cdc f(et+y)=Fif(z), fy), YeyeR 2 Sle +u) =af(a)ftu) +b4(e) +040) + va,yeR. (10) a Gidi. Nhan xét ring, néu dat hi i@)= thi thi ti (10) ta nan duge A(z +y) =h(x)h(y), Ve,y ER va phuong trinh ham nay c6 nghi¢m h(x) = e%*. Suy ra nghiém cita (10) c6 dang ee, I@)=——. Bai todn 6, Gid sit f(x) la nghiém cia phuong tinh ham: Slax + by +0) = Af(2) + Bfly) +C(@bAB #0), ¥2,yER (11) Ching minh ring ham s6 g(x) = f(z) — (0) thoa man phwong trinh Cauchy g(r+y) =9(z)+9(y), Vz,yER. Gidi. Lin bun dat 2 = 2, y = 2S, ne - vao (11), ta thu duge cdc ding thie So fut) = As(2) + Bs (- fe) = 4s(®) +f (-F) +0, b fo) = Af + BS (ZS) +0, £(0) = AF(O) + BF( )+e. e ~ 8) Suy ra f(u+v) = flu) + f(v) — F(0). Tir day suy ra digu phai chtmg minh. Bai toan 7. Gia sir him s6 f(z) lien tye tren R 1a nghiém cia pbuong trinh 1 ) = Af(z) + Bfly) + C (abAB #0), Va,yER. Chiing minh ring khi dé A =a, B=. Gidi. ‘That vay, nghi¢m cla flav + by +6 g(z+y) = g(x) +9(y), Vey eR trong lép cde ham Lien tuc 1A ham tuyén tinh g(x) = ax. Do vay, nghiem f( dang f(z) = a +9. Thé vao (11), ta thu duge A= a, B=b va ac—~C=(a+b-1)3. Bai toan 8. Giai va bién luan phyong trinh ham sau trong lép cdc ham sé f(x tue trén R: faz + by +c) = Af(x) + Bf(y) +C(abAB £0), Va,yER. Gidi. ‘Theo Bai todn 7, thi digu kién cn dé phuong trinh him (11) ¢6 nghiém | a b=B. Gia sit diéu kign nay duge thod man. Theo (12), ta chia cdc trudng hop riéi khao sat. Xét cde trudng hop sau: Truéng hop b+a=1, c=0. Khi dé, (11) c6 dang f(ax + (1~a)y) =af(z) + (1-4) f(y) (abAB #0), Ya,yeR. Ta thu duge (13) thude Iép ham chuyén tiép céc dai lung trung binh cong. V (13) 6 nghim f(x) = 02 +8, 0,8€R. Truéng hop b+a=1, c#0. Khi dé, (11) c6 dang f(ax + (1—a)y +c) =af(x) +(1—2)f(y) + C (adAB #0), Ve,y ER. Dat f(e) = oe + A(z). Ta thu duoc (13) du6i dang A(ax + (1 ~a)y +c) =ah(z)+(1—-a)h(y), Ve,y eR. 10 Dé kiém tra, phuong tinh (14) chi c6 nghiém hang tuy ¥ (xem (12) va vi vay, (13) c6 nghiém f(x) = oe +6,8ER. Truéng hop b+a #1. Theo Bai todn 6 thi nghiém cba (13) c6 dang f(x) = ar+8. Theo (12) thi ac—C = (a+b—1)8. Vay néu cho a € R gid wi wy ¥ thi 6 = = F a Cha y Néu khong ddi héi nghiém cia (11) JA ham sé lién tuc trén R thi céc ding thite a= A, b= B va (12) c6 thé khong thod man. Tuy nhién, ta vin ¢6 céc tinh chat dai sO sau day. Bai toan 9. Gia six phuong trinh ham flax +y) = Af(z) + f(y) (aA #0), V2,yeR (a5) c6 nghiém khéc hing. Ching minh ring néu @ (hoc A) 1a s6 dai s6 véi da thic 161 tiéu P,(t)(tuong tmg P4(z)) thi A (tuong tng a) 1a s6 dai sO va Pa(t) = Pa(t). (16) Gidi. Ta thay f(0) = Onen f(az) minh af(x) va bang quy nap toan hoc, dé dang ching f(akz) = A* f(x), KEN. (17) Gia si = Pi(t) =" + Sorit!, 10,..- tn €@ i=0 Khi d6, theo (17) thi Vi f(x) khée hang nén ao ang Sri, =0 (a8) 0 va vi vay A Ia 86 dai 86, Suy ra P,(2) 18 wée cha Pa(é) va do Pa(t) 1a da thi 16i tiéu nen c6 (16). Neue lai, néu A 12 s6 dai sO thod man (18) thi thyc hin quy trinh nguge lai, ta thu duge (19) va tir d6 suy ra (16). Bai toan 10, Gia sir phuong trinh ham flax +y) = Af(z) + f(y) (aA 40,4 €Q), Ya,yeR c6 nghiém khdc hang. Chtmg minh ring khi d6 a = A. Giai. ‘That vay, theo Bai ton 9 thi P,(t) la da thie bac nhat va vi vay P(t) cting thite bac nhat (véi hé sé bac cao nhat déu bang 1) nén a = A. Bai toan 11. Giai phuong trinh ham sau trong Idp cdc ham s6 f(z) lien tue f(e+y) =a f(z) f(y) (a> 0), Ya,yeR. Gidi. Dé thay f(1) > 0. N&u f(1) = 0 thi tie (21) ta 06 ngay f(e! (1) > 0. Bing quy nap, dé ding kiém ching hé thac = 0. Xét truon f(z)", Yn eN*. Vay voi x = 1 thi J(n) =a" F"(f()", Vn e Nt. _ om Voie = ™, ta thu duge n fn) = BRE [5(@)]", vmnen. va Suy ra Do f(1) > 0 nan c6 thé vit 1 c= 5 + log, F()- Tur (21) suy ra f(z) e+e Ve Qt, Do f(z) lién tue nén (16) thod man véi moi c € R*. Voi « < 0, ta dat —2 = do f(0) = 1 nén ti gid thiét (21) ta nhan duge (x)al*/2)-, hay f(x) =a ere Ye eR. Nhén xét. Bang cach dat J (2) = a*/9(z) ta dua (15) vé dang quen biéi g(z+y) =9(2)o(y), Va eR. Bai toan 12. Xéc dinh cdc ham sé f xdc dinh va lién tuc tren R tho min diéu kien Paty) + fz) = fla) + f(yrz). Vey. 2 ER. (1) Gidi. Dat f(0) = a thi véi z = 0 trong (1) ta thu duge f(@w@+y)+a= f(z) +fly), VayeR. (2) Dat f(x) = g(x) + a. Ti (2) ta nhan duge oz +y) = 9(2) + oy), Ve,yeR. (3) Phuong trinh (3) 6 nghiém g(x) = az,a eR. Suy ra phuong tinh (1) cé nghiém f(z) =ar+8, a,8ER Thi lai, ta thay ham f(e) = aa + 8 thod man diéu kien bai ra. Bai toan 13. Xéc dinh cdc ham s6 f x4c dinh va lién tuc trén R thoa man diéu kign F(e+y)F(2) = F@)IL(y) + f(2)], ¥a.¥,2z ER. (4) Gidi Thay y = z = 0 trong (4), ta thu duge f(0) f(x) = 0. Vay f(0) =0. Vai 2 =0 thi Se +) FO) = f@)IY) +f). Ya.y eR hay S(2)f(y) =0, Va,yeR. Suy ra f(a) = 0. Bai tap Bai 1. X4c dinh ham s6 f(x) xéc dinh va lien tuc trong mién x4c dinh va thod man diéu kién: Bai 2. Xac dinh him s6 f(x) xéc dinh va lien tue trong mién xéc dinh va thod man diéu kien: Bai 3. X4c dinh him s6 f(r) xdc dinh va lin tuc trong mién x4c dinh va th diéu kien: Bai 4. Xée dink him s6 f(x) xe dinh va lien tuc trong mign x4e dinh va th digu kien: f(z) + FM) 1—2f| Bai 5. Xéc dinh ham s6 f(z) xdc dinh va lién tuc trong mién xéc dinh va th diéu kien: f(c+n= Bai 6. Xéc dinh ham s6 f(x) x4c dinh va lién tuc trong mién xéc dinh va th diéu kien: Bai 7. Xéc dinh ham s6 f(x) xc dinh va lin tuc trong mién xe dinh va th diéu kign: _f@ f(z +y) Bai 8. Xc dinh him s6 f(x) xc dinh va lién tuc trong mién xée dinh va the diéu kien: oe) 2f(2) f(y) cosa t) f(y) Bai 9. Xc dinh ham s6 f(z) xéc dinh va lien tuc trong mién xéc dinh va thi diéu kien: S(ety)= pet y= LoL =28e sn Bai 10. X4c dinh him sO f() x4e dinh va lien tue trong mign xdc dinh va thi digu kien: (x) + f@) + 2f(2)fy)cha 1~ f(x ty) Bai 11. Xéc dinh ham sO f(x) x4e dinh va lien tyc trong mién xde dink va thi digu kien: af (Vx? + y2) = f(z) f(y) Bai 12. Xéc dinh ham s6 f(x) xdc dinh va lién tue trong mign xée dinh va th digu kien: f(ety)= z+ $( 5H) = 1010. 1+ é Bai 13. Xac dinh ham s6 f(x) xdc dinh va lién tuc trong mién xdc dinh va the diéu kien: fla ty + ey) = Se)F0)- Bai 14. Xac dinh ham s6 (a:) xdc dink va li¢n tuc trong mién x4c dinh va thod man diéu kien. FH") =J@)+ Sly) ay 26 Bai 15. Xa diéu kien: dinh ham s6 f(x) xdc dinb va lign tuc trong mign xc dinh va thoa man t (va? +y+1) = f(x) + S(y)- Bai 16. Xéc dinh ham s6 fia diév kien: xde dinh va lién tuc trong mién x4c dinh va thoa man {(@+y) — f(e—y) = 4VT@)L)- Bai 17. Xéc dinh ham s6 f(«) x4c dinh va lién tuc trong mién xéc dinh va thod man digu kien: f(et+y) + fey) = Af(a) + fy))- Bai 18. Xéc dinh ham sO f(e) xéc dinh va lién tuc trong mién xc dinh va thod min digu kien: faty)+f@—y) = 2s Bai 19. X4c dinh ham s6 f (2) x4c dinh va lien tuc trong mién x4c dinh va thod man diéu kién: f(e@+y) - f(@~y) = 2F(y) BAT PHUONG TRINH HAM CO BAN Nguyén Van Mau 2.1 Bat phuong trinh ham véi cdp bién tu do 2.2 Biéu dién mét s6 dang ham s& 23 .3 Biéu dién cac da thite duong trén mét tap 2.1 Bat phuong trinh ham véi cap bién tu do Bai toan 1. X4c dinh cdc ham s6 f(x) thod man déng thdi cée digu @ f(x) 20, Ye ER, (i) f(e@+y) > f(@) + fy), YayeR. Bai giai. Thay c= 0 vio digu kien dau bai, ta thu duge FO)20 7 { 70) > 240) MY LO=0 Vay nén $(0) = fle +(-2)) > fle) + f(-2) 20. Suy ra f(x) =0. Thit lai, ta thay ham s6 f(x) = 0 thod min diéu kien bai ra. Bai toan 2. Cho truéc ham s6 A(z) = az, a € R. Xéc dinh cdc ham s6 f( man déng thoi cdc diéu kién sau: @) f(z) pax, Ve eR, Gi) f@+y) > fe) + fy), YayeR. Bai gidi. Dé y ring A(x + y) = h(x) + A(y). Dat f(c) = A(z) + 9(x). KI thu duoc cdc digu kien (i) g(x) > 0, Va ER, (i) g(v@+y) 2 9(z) + 9(y), Ya, y eR. Lip lai cach gidi Bai todn 1, Thay « = 0 vio digu kién dau bai, ta thu duc 90) 20 |. 7 { mo co g(0) = 0. 16, Vay nén 9(0) = g(x + (-2)) > g(x) + (2) > 0. Diéu nay kéo theo g(a) = 0 hay f(x) = ax. Thir lai, ta thay ham s6 f(x) = aa thoa man diéu kién bai ra. Bai toan 3. Cho s6 duong a. Xéc dinh cdc ham s6 f(z) thod min déng thoi céc digu kien sau: @ f(z) >a", VeeR, Gi) f(e@t+y) > S(@) sy), VayeR. Bai giai. Dé y rang f(x) > 0 véi moi x € R. Vay ta co thé logarit hod hai vé céc bat dang thitc ca diéu kién da cho. @ Inf(e) > (ina)e, V2 ER, Gi) Inf(@+y) > Inf(x) Inf(y), Ve,yeR. Dat In f(r) = y(a), ta thu duge (i) v(x) > (Ina), Vr ER, (i) ole +y) 2 o(z)ey), Vey ER. Ta nhan duge Bai todn 2. Bang cach dat p(x) = g(x) + ( Ina)z, ta thu duge cdc diéu kien (i) (2) 20, Ve ER, Gi) oe +y) 2 (a) + oly), Ve.yeR va g(x) = 0 hay y(x) = (ina)x. Suy ra f(x) = a® Thit lai, ta thay ham s6 f(a) = a® thod man diéu kién bai ra. Bai toan 4. Xéc dinh cac ham s6 f(x) thod man cdc diéu kign sau: fe) > 60), (24%) eLOAIO veyer re) Bai giai. Dat (0) =a va f(z) - a= g(a). Khi d6 (1) c6 dang g(x) 2 0, o( $4) z selt ay) ve,yeR (2) véi 9(0) = 0. Thay y = 0 vao (2), ta thu duge 9(5) > hay g(x) > 29(5): vr ER, (3) Ti (2) va @), ta suy ra o( =") >9(5) +9(¥), va,yeR 9(0) =0, g(x) 20, g(t +4) > ole) +9(y). Va,yeR. (4) Tip theo, ta nhan duge Bai todn 1. Suy ra g(x) = 0 va f(x) = const. Thir lai, ta thay ham s6 f(x) = c thod man diéu kien bai ra. 2.2. Biéu dién ham sé Trong mue nay, ta mo td mot sO cong thie biéu dién ham co ban. Cac bi nay thutmg gin vdi cdc muc dich mo 14 céc dic trmg ham, cdc tinh chat ¢ thie duéi dang tudng minh va don gidn hon. Day [A nhing hé thie rét quan tr¢ quan dén nhimg rang bude dang bat ding thitc cho tude. Trong mue tigp the xét riéng cho tru’ng hop bigu dién da théc duong tren mot tap. Bai toan 1. Xc dinh cic ham sO f(t) thda man diéu kien: f(z) = max{22y — fy)}. Ya eR. Giai. Trude het, ti (1) ta suy ra f(a) 2 2ay — fly), Vay eR. Thay c = y = ¢ vao (2), ta thu dugc bat ding thuc f(t) >a, WeeR. Ta c6 2ay — fly) < 2xy -y' MA max{2? — ver Suy ra f(x) < 2?. Ket hop véi (3), ta duge f(z’ : ‘Thit lai, ta thay ham s6 f(«) = x? thod man digu kién bai ra. Bai toan 2, Xéc dinh céc ham s6 f(t) théa man diéu kien: f(x) = max{2°y + ay? — fy}, Vee R*. yeRT Gidi. Tuong ty nhu Bai toan 1, ta suy ra f(a) 2 2?y+2y? - fly), Vey eR*. Thay x = y =¢ vao (4), ta thu duge bat ding thie f(t) 223, Vee Rt. 18 Tir (4) va (5), suy ra wy tay? — f(y) aytay?—y Ve,yeRt thi ti diéu kien f(e) = max{a?y + ay? — fy}, Vo e Rt yeR* ta c6 ngay ham c4n tim f(z) c6 dang f(x) = 2°. Bai toan 3. Chimg minh ring néu: +) Hoac f/(2) > 0 va h(x) > 0 voi moi x ENC Dy +) Hoac f’(x) > 0 va h(x) > 0 véi moi rE QC Dy thi trong Q ta c6: F(g(x)) + g(x)-h(z) > f(0) & g(x) 20. (a) Gidi. ‘Sir dung dinh ly Lagrange, ta c6 S(9(2)) + a(a).h(a) > £(0) = f(9(x)) — £0) + g(a).h(z) 2 0 & [f'(c) +Ale)lg(z) >0 (c nam gitta 0 va g(x) & g(x) >0 (do [f'(c) + A(x)] > 0) (pcm). Bai toan 4. Gidi bét phuong trinh: 3-44 (2? — 43°? > 1. m 6. f(x) = 3". Ham s6 f(2) xdc dinh, lien tue, kha vi v6i moi € R va f(x) = 3*.In3. Ta 06 (3) f(a? - 4) — f(0) + (w? -4)3°-? > f(a? — 4 — 0] + (2? — 4)3*-? > (c nam gitta 0 va «? — 4, theo dinh [2? — 4].[3°.n3 + 37-7] > 0 2-420 (do 3°.ln3+3°-2>0, 2eR) gg-2V2%<-a. (do f(0) =) co ly Lagrange) Vay bat phuong trinh 6 nghiém re-2vacz2 Bai toan 5. Cho cdc sé duong M, a. Tim man diéu kien ham s6 f(x); g(x) :R + If(y) ~ F(a) — 9(2)(e— y)| < Mle —yP**, Vary eR. Gidi. Gia sir tn tai cde ham sO f(x), g(x) thod man yeu cau bai ra. Déi chd 2; Q), ta duge If(z) — fy) — 9@)(y —2)| < MJy-2/?** (13.1) voi moi 2;y ER m Cong timg vé véi vé cita (3), ta thu duge (az) — 9(v))(e — y)| < 2M ly — 2)?" Suy ra <2M\2—yl*, Ya;yeR, oxy. a(x) - 9y) y | Trong (4) ta c6 dinh x, cho y + 2 ta duge g(x) =0, « ER nen g(x) =c=const , Vee R. Mat khdc, thay g(zr) = c vao (1) va [am tuong ny nh tren, ta cing nhan duge £0) =f) _ 1 @-y 4| < 2M 2 — yl", Vasy eR, cA y. CO dinh x vA cho y + « ta duge fi(z)=c, ER nen f(x) =ca+d, ER (d= const ). Thi lai thay hai ham sO f(x) = cr +d; gf ham s6 can tim [i c thod min yéu cdu bai ra. \ f(z) =cx +d; g(x) =c voi od lA cdc hing so ER. Bai toan 6. Ching minh ring = max(c2). Gidi. Dé dang kiém tra cong thie —|c| > az < |x|, Va € [-1,]]. Tir day ta c6 ngay dpem. 2.3 Biéu dién cc da thie duong trén mét tap Trong phan nay 1a xét mot sé biéu dién cita da thife dong wen mot tap dudi dang tdng, hiéu, tich.... cla cdc da thtte cé dang dac biét cho truéc. Bai toan 1. Cho da thie P(z) € R[z] va P(z) > 0 véi moi x € R. Ching minh ring da thie P(2) cé thé biéu dién duoc duéi dang ) = [A@)? + [B@)?, trong 46 A(x). B(e) cling 1a céc da thite. P(x Giai. Do P(x) > 0 véi moi x € R nén da thite P(x) 06 bac bing 2n va cé thé phan tich duge dudi dang tich cia cde nhan tir bac hai khong 4m, nghia 1a P(2) = []l(qjz+2,) +97), pal wong 46 aj,2;,y; €R, j= 1,2,...,n. Tur hang ding thitc (0? + a2) (3 + 93) = (Puyo + arn)? + (Pg — Poa)’, ta c6 két luan: Tich cha hai biéu thie dang [u(x)]? + [v(x)]* cing 1a mot biéu thitc c6 dang dé. Sau hitu han bude thuc hién quy trinh dé ta thu duge biéu thttc dang P(a) = [A(e)? + [B(a)). Bai toan 2. Cho f(c) = az? +br-+c € Ris] théa man diéu kien f(x) > 0 v6i moi x > 0. Chimg minh ring t6n tai da thite P(z) sao cho da thite Q(x) = f(x)P(x) 6 tat cdc hé s6 déu khong 4m. Giai. Do f(x) > 0 v6i moi z > 0 nena > 0 vac=f(0) 20. Néu b > 0 thi ta chon P(x) = 1 va ta nhan duoc ngay diéu phii chime minh. Néu b 0. Tatim P(e) dudi dang P(x) = (2+1)" voin >2. Ta c6 P(x) =(x+1)" = ote ct ka nén S()P(2) = (ax? + br + 0)(a +)" = a = ar"? + (b+ na)a™ +.-.4 Sack? 4.6081 + chat k=0 tet (b+ ne)e +e. Ta chon n sao cho b+na>0 q@ btne>0 (2) ack? 4508-1 4.cCK 30 Vk 22 (3) Nhan thay ngay ring voi n > max{—b/a, —b/c} thi cac digu kien (1) va (: thod min (do a > 0). ‘Ta bign déi vé trai cha (3): h(k) = (a —b + c)k? — [a ~ (n +.2)b + (2n + 3)cf& + (n+ 1)(n +2) > Dob<0,a>0,c>0nena-b+e>0, é (3) ding véi moi k ta chon n sao cho biét thite ciia tam thtic bac h khong duong (A, < 0). Bigu thie cha A, ciing 1a mot tam thiic bac hai theo : s6 cha n? la (b — 2c)? — 4c(a —b +c) = b? —dac < 0 (do f(x) > 0 Ve >0;>0,a>0. Do vay, ta cé lim Ay = —co. we Do dé voi n dit ton thi Ay, <0. Tir dé suy ra t6n tai n théa man déng thoi (1), (2) va (3). Bai toan 3. Cho da thtic G(z) = ana" + anyz"™1 +--+ +ayx +49 (n > 3) thoa man diéu kién g(x) > 0 véi moi x > 0. Chtmg minh ring khi dé t6n tai dé da thite Q(x) dang Q(x) = g(x)(x + 1)* c6 cdc he sé déu khong am. Giai. Ta xét hai trudng hop deg g(x) = 2m va deg g(x) = 2m +1 voi me! Khi n = 2m thi ta c6 thé phan tich o(2) = [](ana” + dex + cx), ei trong dé axa? + byt +c >0 Va > 0. ‘Theo Bai toan 2 v6i mdi da thtte a,27 + b,x +c, déu tén tai sé ty nhien m, ‘ da thie Qk(x) = (ag + byw + cx)(@ + 1) 6 c&c hé sé déu khong 4m. Tu d6 tex a(e)(o+ 1)" = gaylo-+ mht Tose) = k=l k=1 2 1a mot da thite cing e6 cdc he s6 déu khong am. Khi deg g(«) = 2m-+1 thi g(z) c6 it nhat mot nghi¢m khong duong I —a (a > 0). Tac g(x) = (x+a)h(z) véi h(w) > 0 véi moi x > 0 va deg A(x) = 2m. Do deg h(x) = 2mm nen theo trudng hop 1 t6n tai s6 nguyén duong s sao cho hfr)(2+ 1)* c6 cdc hé s6 déu khong 4m va vi vay da thite g(x)(a + 1)* cling c6 cdc hé sé déu Khong 4m. Bai ton 4. Héi c6 16n tai hay khong t6n tai cdc da thitc P(x), Q(x), (2) v6i cdc hé sO nguyen duong va thod man he thite 2or il T(2) = (? -32+3)P(@), Pla) = (5 - = +75) Ql). Giai. Viét lai cdc ding thic cia dé bai duéi dang 60T (2) = 60(x? ~ 32 +3)P(2) = (32? — 4v + 5)Q(2). (4) Céc da thitc (2? — 3x +3) va (32? — 4z +5) vO nghiém va nguyén t6 cing nha. Vi vay tir (4) suy ra t6n tai cde da thie P(2), Q(z), T(x) thod man diéu kign dé bai khi va chi khi t6n tai da thie S(x) sao cho cfc da thite (30? — de +5)S(2), 60(a? — 32 + 3)S(z) (32? — 4a + 5)(a? — 32 + 3)S(z) déu 1a nhing da thitc vi hé sO nguyén duong. Theo két qua cia Bai to4n 3 thi t6n tai sO nguyen duong ky di 1én sao cho (30? — 4a +5)(e + 1)" la mot da thtic c6 cdc hé s6 nguyén khong 4m va tir d6 dé dang suy ra ring khéng c6 hé s6 nao cia da thitc bang 0 va t6n tai sé nguyen duong ke dit ldn dé (2? — 3x + 3)(2 +1)" Ia da thitce c6 céc hé s6 déu nguyén duong. Tir 46 suy ra (30? — 42 + 5)(a? — 3a + 3)(a +1) 4%? cing [A mot da thie véi hé sé nguyen duong. Nhu vay cau tri di cha bai todn 1A khing dinh. Ching han ta c6 thé chon P(x) = (82? — 4x + 5)(e + 1)8 +, Q(x) = 60(2? — 3x + 3)(a + 1)8 +, ) = (30? — de + 5)(2? — 8a + 8)(a + 1) +H, (Bang c4ch thir truc tip c6 thé thiy ring ky > 3, kg > 15). Bai toan 5. Cling minh ring néu da thie P(x) > 0 voi moi > 0 thi 161 da thie A(r), B(x), C(x), D(w) a8 P(e) bigu dién duge duéi dang P(x) = [A(c)? + [B(a)]? + x{[C(x)]? + [D(a)}7}, Giai. 1. Truong hop deg P(x) = 2m. Néu m = 0 thi ta dé dang viét duoc biéu dign (5). Voi m > 1, ta ¢6 thé phan tich da thite P(x) dudi dang P(z) = [tz + bee + cx), A=) trong dé a, > 0, a,x? + bya +c, > 0 Va > 0. Nhan xét ring voi méi da thitc apa? + bea +ce 20 V2 30 ta déu 06 thé vist duge ax? + br +c = (aga? + By)? + a(op + 62), m Aol ~ Ih ona” + By) 2 + a(ok + 82)). Mat khac, ta ciing c6 tich cla hai da thttc dang (p? + q”) + 2(r? +s?) cing la thac c6 dang d6. That vay, ta c6 [@i + af) + 2(rf + sP)]l(e8 + 92) + 273 + 89)] = [(Pi-+af)(73 +98) +27 (r} + 53)(r3 + 83)] +el(pi +92) (r3 +53) + (r+ 57) 3 ‘Theo Bai todn 1 thi ta c6 bigu dién (PE + af) W3 + a8) + 27 (r3 + 82) 073 + 98) = AP + (BOP, (WE + a) (rd + 98) + (ri + 51) (08 + B) = [C@)P + [D@)P. Vay nén P(a) = (A(z)? + (BG)? + 2{C(@)? + [D(@)]?}.- Trréng hop deg P(x) =2m+1. Lap luan tuong ty nhu d6i v6i Bai todn 3 ta thu duge P(2) = S(t d)(aqu? + dex +e) kek (2)? + [B(a)P + 2{[B(@)?? + [C(@)P} = [B@)? + [C(@)P. 4 (Do c&c biéu thie trong ngoac nhon 1a khong 4m véi moi a nén 4p dung duoc két qua biéu dién cia Bai toén 1). Bai toan 6. Cho da thite f(x) € Riz] théa man diéu kien f(x) > 0 Wa € (-1:1). Ching minh rang da thie f(z) c6 thé bigu dién duoc duéi dang k L(x) = Saj(1 + 2) (4 — 2) j=0 v6i ay 20, aj, B EN. Giai. Gia sir deg f(x) = m. Dat lia t-1 st => rao. l= t+1 Do ¢ €(-1)1) nént > 0. Vay f(2) =1(4) > 0 Vé> 0. Do d6 da thie Q(t) v6i t eel am =e+y"s (55) 1a mot da thitc thda man diéu kien Q(t) > 0 Vt > 0. ‘Theo Bai ton 3 thi tén tai n € N sao cho 1 erat = w+ MS ) Wa mot da thitc c6 c&c hé s6 déu Khong am. Suy ra Lyymin (Eat * (+0) #53) =o, Far v6i by > 0, k 0. Tir dé ta 06 didu phai ching minh. Bai toan 7. Chwmg minh ring t6n tai da thie P(x) bac n va aban gid wi trong khoding (—1; 1) va a6 khong thé bidu din duoc duGi dang = Anal -2)*(1 +2)", wong d6 Ang 20, a+ 8<7n; o,f la céc sé nguyén khong 4m. Xét da thie P(x) = «? +¢, trong d6 = > 0. Gid sit c6 thé viet P(x) du PC YS Avale)G 2)" +2)%, at Bsd trong dé ¢ > 0, A(e) > 0, @ va @ chay tren tat cd cdc sO nguyen khong a+ < 2. Nhu vay, vi ¢ bat ky trong téng nay sé chita vita diing séu sé hang cAch thé x = 0 vao (6) ta nhan duge Ago(z) bi chan voi 0 atBen Bai tap Bai 1. Xéc dinh ham s6 f(z) déng bién trong (0, 277] va thod man digu kign |. f(z) < sine, Ve € (0,2 Bai 2. Xéc dinh ham s6 f(z) déng bién trong [0,27] va thod man digu kien f(z) < cosa, Yx € [0,27] Bai 3. Xéc dinh ham s6 f(x) déng bién trong [0, 4] va thoa man diéu kien f(z) <2 - 3x, Ya € (0/4). 26 Bai 4. X4c dinh ham s6 f(«) nghich bién trong [0,27] va thod man diéu ki¢n f(x) >sina, Ve € [0.27]. Bai 5. Xac dinh ham s6 f(z) nghich bin trong [0,2n] va thod man diéu ki¢n (a) > sina, Vz € [0,27]. Bai 6, Xéc dinh ham s6 f(z) nghich bién trong [0,4] va thoa man diéu kien f(z) > 423 — 32-1, Ver € [0,4]. PHUONG TRINH HAM LIEN QU. Nguyén Van Mau 3.1. Ham s6 chuyén déi cée tam giac 3.2. Phuong trinh ham lién quan dén tam gic 3.1. Ham sé chuyén déi cdc tam gide Ta nhac lai (khong chtmg minh) mot sé he thtte dic trmg cho tam gidc hoc sinh bac THPT déu quen biét. Day [2 nhiing he thitc don gidn mo ta su ry tw nhién cia céc yéu to canh va géc trong mot tam gidc. Bai toan I. Digu kien can va di dé 3 sO duong A, B,C 1A dd do cdc gée tam giic AABC I A+B+CeHrn, Bai toan II. Diéu kien cain va di dé 3 s6 duong a,b, khi gin véi cng m9 do lutmg lap thanh d0 dai cdc canh cla mot tam gic AABC la a+b>c bte>a et+ta>b N6i céch khéc, ta 06 thé phat bigu ngén gon nhu sau, Bai toan II’. Diéu kien cin va di dé 3 s6 duong a,b,c 1A do dai cdc canh c tam giéc AABC 1a |b-cel 0, f(b) >0. f(e)>0. Suy ra ata>0, b+a>0, c+a>0, VAABC hay a>-a, a>-b, a>-c, VAABC. Digu nay tuong duong vi @ > max{—a, —c}, VAABC haya 20. Neguoc lai, voi a > 0 thi f(a). f(b), f(c) 1 do dai c&c canh cla mot tam giée do a,b,c 1a do dai cdc canh cba mot tam giéc. Vay nén vi a > 0 thi ham s6 f(z) = z+a c6 tinh chat f(a), f(b). f(c) 1a dQ dai céc canh cia mot tam gide img v6i moi AABC. Bai toan 2. Xéc dinh a dé ham s6 f(x) = ax 06 tinh chat f(a), f(b), f(c) la do dai céc canh cha mot tam gide tng v6i moi AABC. Bai gidi DE f(a), f(b), f(c) 1a do dai cdc canh ca mot tam gidc, trudc hét phai cd f(a) >0, f(b) >0, f(c) > 0, VAABC. Suy ra aa>0, ab>0, ac>0, VAABC. (a) Tir (1) ta thu duge a > 0. That vay, néu a <0 thi f(a) <0. Vay véi a > 0 thi ham sO f(x) = aw c6 tinh chat f(a), f(b), f(c) Ia dd dai cde canh cla mot tam gide tng v6i moi AABC. Bai toan 3. Xéc dinh a, 8 dé ham s6 f(x) = ax +B c6 tinh chat f(a), f(b). f(c) la d6 dai cdc canh cia mot tam gidée tng voi moi AABC. Bai gidi DE f(a), f(b), f(c) 1 do dai cdc canh cha mot tam gidc, tude hét phai 6 f(a) >0, f(b) >0. f(c) > 0, VAABC. Suy ra aat+f>0, ab+8>0, ac+f>0, VAABC. (1) Tir (1) ta thu duoe a > 0. That vay, néu a < 0, 8 tuy ¥ cho trudc thi ta chon AABC 6 a dit 16n thi theo tinh chat cia nhi thie bie nhdt sé nhan duce aa + 4 <0. Tuong ty, cing ti (1) ta suy ra 8 > 0. That vay, néu f < 0, ta chon AABC 6 a di nhé thi theo tinh chat cia nhi thttc bac nhat sé nhan duge aa + 6 < 0. ‘Trudmg hop Kkhi déng thoi xdy ra a = 0,9 = 0 thi f(r) = 0 khong thoa man bai loan. Voia 20, B20 vda+f > 0 thi ta thay f(a), f(b), f(o) Ado dai ¢ cla mot tam giée do a,b,c [A do dai céc canh cia mot tam gide, Vay nen: Voi a > 0, 8 >0vVAa4+9 > 0 thi ham sO f(z) = ar +B cot F(a). f(b). f(c) 1 do dai cée canh cia mot tam gide img voi moi AABC. Bai toan 4. Xéc dinh a, @ dé ham s6 f(x) = aaaeal c6 tinh chat f(a), f 1h d9 dai cdc canh cla mot tam gic tng voi moi AABC. Bai gidi Khong mat tinh téng quat, ta lu6n ludn gia thiét a > b > c. i Nhan xét ring, phép nghich dio g(z) = = khong c6 tinh chat: g(a), 9(0) z dd dai cdc canh cita mot tam giée img voi moi AABC. That vay, xét tam 2,c=1 thi ta c6 DE f(a), f(b), F(c) 1A do dai cdc canh cha mot tam gidc, trude hét phai f(a) >0, f(b) >0, f(c) > 0, YAABC. Say ra aa+3>0, ab+8>0, ac+8>0, VAABC. Tir (1) ta thu duoc a > 0. That vay, néu a <0, 8 tuy ¥ cho truéc thi ta chon c6 a dit Idn thi theo tinh chat chia nhj thie bac nhat sé nhan duge aa + 8 < Tuong tu, cing tir (1) ta suy ra B > 0. That vay, néu 9 <0, ta chon AAi da nho thi theo tinh chat cia nhi thitc bac nhat sé nhan duge aa + 8 < 0. ‘Truémg hop khi déng thdi xdy ra 8 = 0 thi f(z) = 0 khong thod man (theo nhan xét & trén). V6i a = 0, 8 > 0, ta thu duge ham hing duong nen f(a) = f(b) = f(c J(a), f(b), f(c) 2 do dai cdc canh cia mot tam giéc déu. Xét trutmg hop a > 0, 3 > 0. Khi dé F(a) > Fb) 2 Fle). Vay ta cin xée dinh c&c s6 duong a, 9 sao cho luon c6 J(a) + f(b) > fle), VAABC, a>b>c hay stot aat+Z" abtB~ act+p’ Xét cfc tam giée ABC can déng dang véi tam gide canh 3,3, 1, tte a = c=d véi d> 0 tuy ¥. Khi dé, (1) 06 dang VAABC, azboc. | 7 1 soot Od Mate t 3dath? ara “>? hay 1 —— oe Mata” dara “> 30, hay 3 > 2da, Vd > 0. Digu nay khong xay ra khi d di ién Vay voi a = 0, 8 > 0 thi ham sO f(x) = s B06 tinh chat fla). (0). (6) + 1a do dai cdc canh cia mot tam gidc tng voi moi AABC. Bai toan 5. Xéc dinh cdc ham sO f(z) lien tue trong [0,7], f(0) = 0 va c6 dao ham trong (0, 7) sao cho f(A), {(B), {(C) tao thanh do do cdc géc cia mot tam gic ting v6i moi AABC cho trusc. Bai gidi Ta cin x4c dinh ham kha vi f(x) sao cho f(a) > 0, Ya € (0,7) FO)=0 S(A) + £(B) + f(C) = 7. Theo gid thiét thi f(0) = O nen f(n) = 7 va C= —(A+B). Suy ra f(A) + f(B) + f(7- A-B)=7, VA.B,A+B€ (0,7] hay S(x) + fy) + f(e—-2—-y) =n, Va,y,c+y€ (0,7). () Lay dao ham trong (0,7) theo bién «, ta thu duge f'(2) - fir —2-y) =0, Va,y,2+y € (0,7). (2) Tir (2) suy ra f"(z) 1a ham hang trong (0,7) va vi vay f(x) = px +q. Do f(0 nén g = 0 va vi vay f(x) = pe. Do f(m) = 7 nén p= 1 vata thu duge f(z) =. Vay ham s6 f(x) = « a lien tue trong [0, =]. f(0) = 0 va c6 dao ham trong (0,77) dé f(A), f(B), f(C) tao thanh do do cdc géc cia mot tam giéc img v6i moi AABC cho trudc, Bai toan 6. Xéc dinh céc ham s6 f(z) lien tue trong [0,7] va £(0) =0, f(z) >0 Va € (0,7) va f(A), f(B), F(C) tao thinh d6 do cdc géc cba mot tam gide mg voi moi AABC cho truéc. Bai gidi Ta phat biéu bai toan da cho duéi dang: Xie dinh ede ham sO f(z) lien tue trong [0,7] va £(0) =0, f(z) > 0, f(x)+f(y)+f(r-2-y) = 7, Va,y € (0,7), aty < Do f(0) = 0 nen voi y = 0, ta thu duge f(x) + £0) + f(w-2) =, Vr € [0,7]. Dat f(e) = + g(x) thi g(0) =0 va g(x) 1a ham lien tye trong [0, x]. Ta 6 Q) e2+9(z) + (1-2) +9(7 - 2 © g(x) +.9(7 ~ 2) =0, V2 € [0,7] hay g(t — 2) = —9(x), Ya € (0,7). ‘ThE f(x) = x + g(x) vao (1) va sit dung (2), ta thu duge r+g(z)t+ytgly)ta-(et+y)+o(r—-(ety)) =m, Vays [0,a) 2+ hay 9x) + 9(y) — g(a ty) =0, Va,y [0,7], 2+y <0 hay g(x+y) =g(x) +9(y), Ye,y € (0.a],c+y <7. Do f(z) lien tuc trong {0, 7] nén (3) [& phuong trinh ham Cauchy va g(2) = f(z) =(1+a)a. Dé f(x) > 0 voi moi z € (0,7), tacdn cd L+a>O0vadé L(A) + f(B) + £(C) = 7 tacin c6 1+a=1. Suyraa=0va f(2 Bai toan 7. Xc dinh cdc him s6 f(x) lien we trong (0, 7] sao cho f(A), f(B) tao thanh do do cdc géc cha mot tam gide img véi moi AABC cho truée. Bai gidi Ta thay 6 hai ham s6 hién nhién thod man bai todn, d6 1a f(x) = 2 va f(2° Ta phat biéu bai todn di cho dudi dang: Xéc dinh cdc ham s6 f(x) lign tuc trong [0,7] va f(z) >0, f(z) + fly) + f(r-2-y) =n, Va,y€ (0,7), aty<7 Cho y —+ 0, ta thu duge f(a) + f(0) + f(r-2) =x, Vx € (0,7) hay Fr 2) =~ f(0) ~ f(z), Ve € [0,7] ‘Thé vao (1), ta thu duge e+g(u) ty toy) +a -(e+y)t9(m-(ety)) = Vey [Oa], 2+ + f(y) +l - £0) - fe . Vary € [0,7], tyke hay f(a ty) + £0) = fe) +f), Yay e Or, ety cr Q Dat f(x) = £(0) + g(x). Khi d6 g(x) lién tue trong [0.7] va (2) e6 dang gla +y) = 9(2) +9(y), Ve.y € (0, (3) Do g(x) lign tuc trong [0,7] nén (3) 1a phuong trinh ham Cauchy va g(x) = az va f(z) = ax +, Ta cén xéc dinh a, 3 dé f(x) > 0 voi moi € (0, m) va dé fA)+ IB) + /C)=7 hay ar+p >0, Va € (0,7) o(A+B+C)4+3G=n. hay ar+9 >0, Vr € (0,7) on+38=n. hay He)=ar+ CSF 5 0, Var € (0,7). (4) Cho r+ 0 va a - 7, tir (4) ta thu duge a? <1 oo 7 V6i 5 f(m) =0. xét 1 thi f(z) = & hién nhien thod man diéu kign bai ra. Vay, cdc ham cén tim déu c6 dang thod man diéu kien bai ra. f(z) =ar+ Bai toan 8. Xéc dinh céc ham s6 f(x) lien tuc trong [0,1] sao cho f(a), f(b), f(c) tao thanh do do cdc canh cia mot tam gide nO tiép rong dutmg won dudng kinh J tng v6i moi AABC noi ep wong dudng tron dudng kinh 1 cho truéc. Bai gidi Ta c6 nhan xét sau: ‘Xét dudng tron O dudng kinh bing 1. Ky higu M(A) 2 tap hop ult cd gidc noi tigp trong dutmg tron O dé. Khi dé didéu kién cin va di dé ba s , 8,7 1 ba géc cha mot tam giée thude M(A) Ia sina, sin 8, siny tao thar cde canh cia mot tam gide thuge M(A). That vay, khi 2, 3.7 [a ba gée cia mot tam gide thi 2Rsine in 3, hay sina, sin Z, siny IA dd di cc canh tuong img cia tam gide noi tigp dun duing tron O dutng kinh bang 1. Ngute lai, khi sina, sin 8, sin 1a dO dai cde canh tuong tng cia tam ti€p dugc trong dutmg tron O dutng kinh bang 1 thi do céc géc a, 3,7 dur a, 8,77 ba géc cha mot tam gide. Vay, theo két qua bai todn 7 thi ce ham cdn tim ¢6 dang 3.2. Phuong trinh ham lién quan dén tam gidc ‘Trong muc nay, ta quan tam dén b cdc ham sO ( phyong trinh ham da é lap nén mot diy cdc tam gidc tmg v6i cdc gid wi tuong tmg cia d6i sé. ‘Trude hét, ta nhan xét ring nghiém cia phuong trinh vo dinh 2” + y? =: tap cdc s6 thuc duong cé thé mo ta dudi dang tham sé = ucosy, x y -=usiny, - + ve (oe z= wu weR*, ve (0.5) Vay ta c6 két luan sau: Bai toan 1. Ching minh ring véi moi (u,v) voi u € Rt, v € (0. a) déu tén tai mot t ma s6 do cdc canh IA nhing so Pi(u,v) =ucosv, Po(u,v) =usiny, Py(u,v) =u va cdc tam gide d6 tng vi moi (u,v) voi u€ Rt, v € (0. 3) cho truée déu gic vuong, Giai. That vay, dé thay P,(u, v) > 0, Pa(u,v) > 0, Ps(u,v) > 0 tng voi moi (uw ueRt ve (0. 5) va ding thie sau luén luon ding (Pi(u, v)? + [Pa(u,e))? = [Pa(u,0))?. Tir do suy ra Pi(u,v), Po(u, v), Pa(u,v) 1& do dai cée canh cia mot tam gide canh huyén P3(u,v). Tigp theo, ta xét cdc bd hm sé mot bién trong lép céc da thie tao thanh cdc canh cia mot tam giée tng voi moi déi s6 trong mot mién cho truGe. a4 Bai toan 2. Chimg minh ring v6i moi « > 1 déu t6n tai mot tam gide ma sé do c4c canh 1A ahing sO Py(z) = 24 +29 422? +241, Ps(a) = 209 4 2? 420 41, P;(z)=a4-1 va cde tam giéc d6 tmg v6i moi x > 1 cho truéc déu c6 géc 16n nhat nhu nhau. Giai. Dat 2? +2+1=a>0,2r+1=b>0vae?-1=c>0thi |b—e| = |x? — 22] 0. Bai 2. XAc dinh cdc da thite T(x), U(x), V(x) bac khong qué 2, sao cho T(z), U(x). V(z) lun luon Jap thanh d6 dai céc canh cha mot tam gidc thudng img v6i moi x > 0. BAT PHUONG TRINH HAM LIEN QUAN DEN TAM | 4.1. Bat phuong trinh ham lién quan dén tam gide 42 Ham sé chuyén déi thit tu cdc yéu té trong tam gide 4.1. Bat phuong trinh ham lién quan dén tam gic Trude hét, ta nhac lai (khong chimg minh) mot s hé thc dac trung cho | ma moi hoc sinh bac THPT déu quen biét. Day 1a nhimg hé thite dac biét qu lien quan dén su rang bude ty nhién cia cdc yéu 16 canh va géc trong mot ta Bai toan I. Trong moi AABC ta déu c6: ting vi géc lén hon thi canh 16 Nhan xét 1. Diéu khang dinh ten cho ta mot ket luan tuong duong sau da Trong AABC khi A < B thi sin A < sin B. Va nhu vay, mac di ham sO f(x) = sin x khong déng bién trong (0,7) t hg thic kigu "déng bién” cho cap géc cba mot tam gidc. Bai toan II. Trong moi AABC ta déu c6: A+B cos A + cos B < 2cos 7 Nhan xét 2. Nhu vay, mic di him s6 f(x) = cos. khong 1a ham lém (c6 bac hai luén ludn am) trong (0,7) ta van c6 hé thite kigu "ham 16m” cho cap mot tam gidc. Bai toan III. Trong moi AABC ta déu cé bat ding thie sinA+sinB+sinC < _ Bai toan IV. Trong moi AABC ta déu cé bat ding thie 36 Bai toan V. Trong moi AABC ta déu c6 bat ding thie cot 4 + cotg 3 + core $ > 3v3. Bai toan VI. Trong moi AABC ta déu oo bat dang thie cos A + cos B + cosC < 3 Nhan xét 3. Diéu khing dinh cia cdc Bai toan II-V dé dang kiém ching due dua tren bat dang thuic Young W.H. quen biét d6i v6i lép ham c6 dao ham khong déi dau trong khoang (0,7). Tuy nhién, d6i v6i khang dinh cia Bai ton VI thi ta thay ngay rng tinh chat cla ham 6m khong cdn duge sir dung nhu mot cong cu co ban dé kiém ching tinh ding dan cia bat dang thie. Vay thi, mot van dé xudt hién tw nhién a: Vé téng thé, ta c6 thé mo t& duge hay kh6ng 16p cdc ham t6ng quat thod man diéu kién 1(A) + (8) +910) < 34(3), hode $(A) + 1(B) +10) > 3(F) v6i moi AABC? Sau day ta xé1 mot s6 minh hoa thong qua c4ch xay dymg cdc phuong trinh ham dé m6 14 nhiing nhan xét da néu 6 trén. Bai toan 1. Cho ham s6 f(t) x4c dinh trong khong (0,7). Chéng minh ring c4c diéu Kien sau day 1a twong duong: at Ja) + Fy) < 2B) ve,y.2 + € (0.7) a) Fa) +10) +4) <3 ) Vr,y,z,0+y +z € (0,7) (2) Giai. That vay, th (1) dé dang suy ra (2). Gid sit x > y > z thi ta cd 24 Eh yea(=-) (3) — f(z) + (3 Tu (1) va @) suy ra tiyte S(z) + f(y) + fe) + FC “)< se 01( AED) « Ava go(B) = fo(* — B) > fo(A) = go( A) Tigp theo, ta chtmg minh ring moi ham f(t) théa man diéu kien bai todn dang i= go(t) khi OAva J(B) 2 90(B) = fo(* - B) > fo( A) = go(A) = F(A). Bai toan 3. Xét ham sé sint khi O F thi (4) 6 dang | 3v3 sinA+sinB+1+cosC < 7 38 Dé yi rng v6i gée C th thi LteosC 2r (dau ding thie x4y ra khi tam gic déu). Ta chimg minh g(R) > 2g(r). Ta c6 f(x) > 0 va f(2c) > f(z) voi moi z > 0 (do f(z) g(2z) d6ng bién) suy ra LE > ae) hay g(2z) > 2g(x). Tu day suy ra g(2r) > 29(r). Mat khéc, (R) > g(2r) do R > 2r va g(x) déng bign. Suy ra g(R) > 2g(r). Bai tap Bai tap 1. Xét him sé sey = {eost Mi O 0 va dau ding tht xay ra khi va chi khi tam giée can. Mat khé dang tinh duoc D = 4r?{4Rr(R - 2r)? — (p? +17 — L0Rr —2R?)?} > 0 2 Vay nen (p? +r? —10Rr — 2R?)? < A(R 2r)> (3) Dau ding thic xay ra khi tam giéc cn. Ty (5) ta c6 p> 2R? + 10Rr — 1? — 2(R — 2r)\/R? — 2Rr (6) p? <2R? + 10Rr —r? +.2(R - 2r),/R? — 2Rr (1) Biéu dién dé thi. Tam gide (R,p,r) déng dang véi tam giéc (2 z 1). Goi 2 = R/r vay =p/r. Theo bat ding thie quan hé gitta R va r cia Euler thi z > 2. Tir bat ding thic (5), chia hai vé cho r? ta c6 bit ding thite tuong duong duéi bign méi 2,y (y? +1 - 102 — 22”)? < de(a — 2)8 (8) Bait ding thie nay lai twong duong voi hai bat ding thitc yP > 2a? + 10r — 1 — 2(2 — 2) fa? - 2 (9) y < 2x? + 10x — 1+ 2(2 — a) a? — 22 (10) Dau ding thie xy ra & (9) va (10) khi va chi khi tam gide can. Trong mat phang Oy truc chudn (8) la bat phuong trinh m6 t4 cho ta mot mién S bao gdm céc cap diém x, y) tho’ (8). Bé dé 1. Phép tong tmg (R,p,r) 4 B 2 1) la mot song Anh tir lop tam gide d6ng dang sang mién $ & (8). (Ban doc ty chtmg minh). Nhu vay thay vi nghién citu tam giéc tuomg tg véi bd ba (R,p,r) ta nghien cou 6 thi cia $8 (8). Do mién S 18 mién cong cé by lién tuc nén cho ta khang dinh chat cla ham {(R,r) va F(R, r) nhur (2) va (3) cha Dinh ly 1. Hay n6i khée di f(R,r) va F(R,r) 1a nghiém thudn nhat bac hai cla bat phuong trinh ham (Rr) SP? S$ F(R,r) v6i du ding thic xy ra cho tam gic cAn, 0.3. Lép céc dang toan phuong bac hai Dinh ly 2. Néu g(r,r) va Q(R,r) 1a hai dang toan phuong bac hai c6 hé sO thuc thod man bat phuong trinh ham (Rr) < (Rr) Sp < F(R,r) < AR) Voi moi tam gic (R,p,r) ma dau ding thie chi xay ra cho tam gide déu thi | 6 q(R,r) = L6Rr — 5r? Q(R.r) = 4R? + 4Rr + 3r? Chitng minh. Ta cb 4R(R —2r? = A(R — 2r?)?(R -r)? — 4r*(R - 2?) <4(R-2r)*(R-r)? Dau ding thite xay ra khi va chi khi R = 2r, we IA tam gide déu. Theo (5) ta: (p? +r? — 10Rr — 2R?) < 4(R -2r)*(R-r)? hay [a 16Rr —5r? 2 Thay x = 2 suy ra da + 26+ = 0. Ta phi c6 aa? +Bx+7>0, Yr> a ax? + Ba —da—~28>0, Ye >2 Gian uée cho « ~2 > 0 ta c6 az+2a+8>0,Ve>2 Lai c6 2a —2)V2? — 2a < (2~a)2? - (6+ B)at (4a +2644), Va>2 Gian ude cho x —2 > 0 ta 66 ave? — de < (2- ~(a+8+2), Ve>2 Suy ra a(a—4)a* + (4a? + 208 — da — 49)a — (2a + 8 +2)? > OVa > 2 Tir (14) cho z + +00 ta c6 @ > 0, choa + 2tacd da+f 20. Tir (15) cho x + 00 ta cd 2— a > 0, ttt (16) cho « + +00 ta 66 a(a ~+ suy ra =0, 8 =0, 7 =0. Danh gid tuong ty cho vé trdi (13). Vay q(R,r) = 16Rr —5r? va Q(R,r) = 4h? +4Rr +37? 1a nghiém tot nb bat phuong trinh (Rr) Sp? $ QWR,r), V(Rp,r) va ddu ding thie xdy ra khi tam gide déu. 44 0.4 Mot sé van dé dé thi Goi S 1a d6 thi cia (y? +1 — 10x — 227)? < 4r(x — 2)? Theo bé dé, ta c6 mot phép song 4nh tir tap g6m I6p cdc tam gic déng dang va d6 thi S. Ta nghien cau d6 thi S. Xét dutmg cong AB :y? = 2a? + 10x — 1+ 2(2 — 2)/2? — 22 AC sy? = 2a? + 10x — 1 — 2(a — 2) 2? — de AD :y? = 42? +4243 AE: y? =l6r-—5 Tat ca céc du’mg cong nay chung nhau tai diém A(2,3, V3). Tu cdc bat phuong trinh ham & trén ta c6 dutmg thing AD cao nhat va duing AE thap nhat. Dutng cong AB va AC cé do nghiéng A déu la V3. Tam gidc déu tng véi duy nhat diém A. DO nghiéng cha AD tai A 1a 10/3/9, dO nghiéng cla AE tai A 1a 8V3/9. Dudng thing y = 2x + 1, » > 2 la tiém cin cla AB va AD. Vay thi cdc diém AB va AC chinh 1a I6p tong duong cia cdc tam gidc can. 4p dung cho dang bac hai thay cho S la tap S’ bi kep bai hai bien AD va AE. Ta lai biét dau cha p — 2R —r (a 0, 1 duong, 1a 4m) cho ta phan loai (lam giée vuong, nhon, hay td) Nhu vay tam gidc vu6ng 1a cdc diém trén dudng y = 2a+1 cia tap S nd 1a dudng FG. Trong binh vé dudi day ky hitu u = 22 + 3V3 —4 vat = 5 =3V3 Diém F(1 + V2,3 + 2V2) dac trmg cho 6p tam gide vuong. Mién tao bdi BA, AF, FG tuong tm cho tam giéc nhon. Mién gita FG va FC 1a mién tam gide img dung. Qua A ké dudng thing song song véi Ox va FG, khi dé mién k hai dudng nay thod min 373 < y < 2x + (33 —4). Thay c = R/r va y=} 6 ngay bat dat thtic tuyén tinh 3V8r

q(R,r) = 16Rr — 5r? b) Cho m= 1 tacé p? < O(R,r) = 4R? +4 Rr + 3r? Dinh ly 4. Vi moi m € {-1,1), u 2 0. v > 0 va moi tam gide cx (R,r,p) thi mp < (m+ 1)?R? — 2(m? — 5m + 2)Rr + (4 — m)r? + ur(R - 2) +0(R- Dau dang thie xay ra khi va chi khi tam gide déu. Ghi chi. Cho m = $,u=0,v = 4 tacé p? < 5R? — Rr +9r? = 91 (R,r) Dinh ly 5. Cho m ¢ (-1,0),m € (0,1),u > 0 va moi (R,r,p) chim ring 2r l-m mp? < (m+ 1)?R? — 2(m? — 5m + 2)Rr + (4—m)r2+u (« ~ Chiing minh. Nhu Dinh ly 3. Ghi chi. Cho m= $,u = $ ta cé p® < 9R? — 23Rr + 39r? = yo( R,r) voi moi (Rr, Dinh ly 6. Néu f : (0,4) — (0,00) 1a mot ham so sao cho p > Rf moi bd ba (R, 7, p). Khi d6 dau ding thie xay ra khi tam gidc déu hodc tam g 66 ding mot géc lin hon 1/3 Ching minh. Gia sit tén tai AABC sao cho A < 1/3 < B < p= Rf(R)- Xét tam giéc A/B/C! ma cos SHE" = cos 258 — cos C48 cos 28 = cos 258" — cos OB! Khi 46 Se Rr py ees one C+ Bi. C'- B! C'+B' = 2cos (cos 5 = 00s ) C-B | =. += )oos “SS = =2 (cos 48 C-B CHB _ $B Goi cos $54 = x, cos $$" = y vir > y > 0. Ta cd Z asin A+sinB+sin€ + cos C-B 2 ) =2sin 2B (eo, 948 =Ar+y)V1l-y* : and) @inep a E 2 sin CAB (oop CAB eee Bi) =202-v) Vine? R 2 2 2 Do d6 ta 06 , P P RoR Suy ra Dan t6i mau thudn. Dinh ly 7. Néug : (0, } > (0,00) IA mot ham s6 sao cho p > Rg(f) ding voi moi (R,r,p) cha tam giéc, Dau ding thite xay ra khi va chi khi tam giéc déu hoac c6 ding mdt géc 1én hon 7/3. Chiing minh. Nhu Dinh ly 6. 0.6 Phuong trinh ham dang f(A+B,B) = f(A, A+B) Ky higu Rf, y] 18 vanh cae da thttc hai bién hé s6 thuc va R(x, y) a tueng céc phan thife hai bin hé s6 thie. Coi f € R(x, y) va xét phuong tinh ham f(a +b,b) = f(a,a +b) hay [a f(a.y) = f(z - ya) Ta xde dinh tu dang cau T: Fay > Fea) bo T(x) =2—y va T(y) =a. Goi G la tap bat bién cia T, ta 06 Ts) =0-y T(x) = -y 22) (23) Nhu vay T8(y) = y va T® = I (déng nhat). Ta co [R(x,y) : G] = 6. | T (a), T(z), T9(2), .., T9() € G. Va 6 ww ~ Ti(a)] = (w? ~ 2?)(w? — y*)(w? — @ -2)) Goi S; (j = 1.6) Ia cac da thie Viét cia T(x) (i = T,6). Ta 06 S.=0 S3=0 S5=0 Qe? —a—yt2y? Savy +o%(ae—y)P +y(2-y)? So = 2yP(w - y)? . S4= yt? Vay R(S1, S2, 53, $4, S5,56) = F(S2, Se) 2G. VaG C R(z,y) suy ra F G. Voi « lA nghigm cia (24) IA phuong trinh bac sau cé hé s6 tring R(S2, la nghiém cia phuong trinh bac hai c6 hai hé s6 trong R(S», Sg, 2) do 22° - 2y? — Sa = 0. Ta c6 (R(x, y) : R(S2, S5)] < 12. Vay (R(x, y) : R(S2, Se) {G : R(S2, S6)| = 2 suy ra P = ay(x — y)(x* — xy — 2ay? +5) va c6 G = R(S2, Ss, P) NhéGn xét. 1. Céc két qua Dinh ly 8 va 9 cho ta dau higu kiém nghiém tinh nghiem | bat phuong trinh ham kiéu (1). 2. Ty nhien phat trign ca bai todn cho dang bac cao 1a vain van dé cén ng! tiép . Xét bat phuong trinh ham cho da gidc, cho té dién tuong ty hay khong \. 4. Vie dua su nghién ctu bat phuong trinh ham vé nghién ctu S va S! bai nay [2 mot huéng mé ra hay. Mong ring sé c6 didu kién dé chting ti trao déi them vé van dé nay. TAI LIEU THAM KHAO {1 I. Bludon, Canad. Math. Bull. (8) 1995, 615-626 [2] Al. Lupas, “On Some geometric. Inequality”. R.M-T. 1/1984. [3] L. Nicolescu, “Inequalities in R,r” Bulutin. Math Romanial 1987 50 DONG DU VA PHUONG TRINH DONG DU Dang Hang Thang I. Dong du Déng du la mot khdi niém cue ky co ban va day sttc manh trong ly thuyét s6. Khai niém nay do nha ton hoc Dite Gauss, 1777-1855) 6ng vua todn hoc, mOt trong nhimg nha todn hoc Ji le nhat cla nhan loai dua ra N6 duge trinh bay wong t4c phdm "Disquistiones Arthmeticcae”cia Ong xudt ban nam 1801khi ong mdi 24 wii. 1.1 Dinh nghia Cho s6 nguyén duong n. Hai s6 nguyén a,b € Z dunige goi [A déng du theo modun n va ta viét a = b (mod n) néu khi chia cho n chting cho cing mOt s6 du. Dé thay ba khang dinh sau 1a twong duong 1. a=b (mod n) 2. a —b chia hét cho n 3.a=b+knvoikeZ Ky higu déng du gidng voi ding tic. 1.2 Tinh chat nham nhaén manh su kién rang déng du c6 rat nhiéu tinh chat 1, N& a = b (mod n),c = d (mod n) thi atc = b4d (mod n),ac = bd (mod n) 2. Véi moi ke€ N* a (mod n) + a = & (mod n) 3. ( Last gién uéc) Néu ac = be (mod n), (c,n) = 1 thi a= b (mod n) Néi rigng néu n IA s6 nguyén 16 ac = be (mod n),c # 0 (mod n) +a =b (mod n) Il. Hé thang du 2.1 Hé thang dw day ai Cho tp A = {a;,a3,...,0n}. Gid sit ri,0 <7 < n—1 [a 86 du khi chia a; cho r;. Néu tap céc s6 du {r1,r2,...,7m} tring v6i Lap — 1} thi ta ndi A JA mot hé thang dur ddy di (goi tat 1a HDD)( modun n). Tap A lap thanh mot HDD( modun n) néu va chi néu i ¢ j + a; # ay Néu A= {a1,a2,...,an} IA HDD mod n thi tir dinh nghia dé suy ra ¢ V6i moi m € Z 16n tai va duy nhl a; € A sao cho a; = m (mod n) © V6i moi a € Z tap A+ a = a) +0,0) 44,...,¢, +4 cing lap thanh HDD modn # Neu c €Z,(c,n) = 1 thi tap cA = cay, cag, ..., can cing Mp thanh HDE Vi dy 1 Cho hai HDD mod n A = {a},..., an} va B = {b1,.., bn}. Chimg m néu n la sO chan thi tp A+ B = {a; +4,...,4, + bn} khong lap thanh m mod n, C6 thé néi gi néu n Ia sé 1é? Gidi Ta nhan xét rang néu C = Di = n(n + 1)/2 # Oo nal n(n + 1)/2+n(n+1)/2 = n(n +1) 1ap thinh mot HDD. Néu n 1é thi chua két luan duge. Nghia la A + B c6 thé 1a HDD, cé thé | HDD. Ching han xét n = 3 Néu A= {1,2,3}, B = {4,5,6} thi A+ B = {5 HDD mod 3. Tuy nhién voi B’ = {5, 4,6} thi A+ B’ = {6, 6, 9} lai khong mod 3, Vi du 2 Cho hai s6 nguyen duong m,n va hai tap A = {a4,...,¢n} {b1,.., bm} trong dé A 1a HDD mod n ,B 1 HDD mod m. Chimg minh 1 (m,n) > 1 thi tap AB = {ajbj}i = 1,2,..,nj = 1,2,...,.m khong lap th HDD mod mn. C6 thé néi gi néu (m,n) Gidi Ta c6 nhan xét sau: Néu A = {a1,...,¢n} 1A HDD mod n va p LA ust 16 cla n thi trong A 6 ding n — n/p s6 khong chia hét cho p. That va =qntril <7 ). Ta 61 (a + be) = Dy ia + LD 0. VayA+ B= {ay +b1,...,0n +n 2 nm) og mn a(n 2ym-Z)o MIM P P P rp PP Day [a diéu vo ly vi p > 1. Néu(m,n) = 1 thi chuwa két uan dugeNghia lA A+ B 6 thé 1 th ong tk HDB. Vi dun = 2,m = 3 Nu A = {1,2},B = {5 = {5,7,9,10,14, 18} A HDD mod 6. Néu A = {1,2},B = {5 aa {5,6,7, 10, 12, 14} khong a HDD mod 6. Vi du 3 Mot s6 nguyen dutong T goi [a sé tam giéc néu n6 ¢6 dang T = k Tim tat ca cdc s6 nguyen duong n cé tinh chat: Tén tai mot HDD mod n tam gidc. Gidi Ta chtmg minh s6 n cé dang n = 2". i) Néu n = 2%, Ta xét tap A = {Tai-i}Ly 6 d6 T, = B(k +1)/2. T HDD mod n vi néuTp;-1 = Taj~1 + (i — j)(24 + 2j — 1) chia het cho n. V 06 uéc 1é nén i — j chia hét cho n. Mau thudn. Dao lai gid sir t6n tai mot HDD A mod n gém n sé tam gidc voi n = m> 1 s6l&. X@ tap B= {T;}",. Ta ching minh B A HDD mod m. lay z € {1,2,...,m}. Vi A a HDD mod n nén tén tai sO tam gide Ty, € T; = x (mod n) + T, = 2 (mod m). Gid stk =i (mod m),i € {1,2,.. 46 k(k + 1) = i(é +1) (mod m). Vim Lé nén tir dé suy aT; = T, = 2° Vay B la HDD mod m. Nhung diéu nay [A mau thudn vi Tn = m(m + (mod m), Tin-1 = m(m — 1)/2 = 0 (mod m) Chii thich Vi du 3 c6 thé phat biéu du6i dang mot bai todn vui nhu sau: Mot lép g6m 7 hoc sinh dimg thanh vong tron dé chuyén béng ngugc chiéu kim déng hé theo quy tac sau: Lép truuéng (coi 1 hoc sinh mang s6 mot) bd qua hoc sinh bén canh( hoc sinh mang s6 hai) chuyén béng cho hoc sinh mang s6 ba. Hoc sinh mang s6 ba 6 béng, bd qua hai hoc sinh ké tiép ( mang s6 bénwr va s6 nam) chuyén béng cho hoc sinh mang s6 s4u. Hoc sinh mang s6 sdu c6 bong , bd qua ba hoc sinh ké tip ( mang s6 7,8,9) chuyén béng cho hoc sinh mang s6 10 va ctf tip tuc nhu thé, Ching minh rang néu n khong c6 w6c 1é thi ludn t6n tai ft nhét mot hoc sinh khong bao gid nhan duge béng. ‘That vay bang quy nap dé dang chtmg minh duge em thir mang s6 i nhin duge bong khi va chi khi t6n tai k € N* dé T, = 4 (mod n). Nhu vay tt cd moi hoc sinh déu c6 béng khi va chi khi t6n tai mot HDD mod n gém cdc s6 tam gidc. 2.1 Hé thang dw thu gon : Cho B = {bi,...,be} 18 mot tap gém k sO nguyén va (b;,n) = 1 v6i moi i = 1,2,...,k. Gidu sit by = gin + ri,1 <17j 3 8 s6 nguyen t6 c6 dang p = 3k +2. a) Chimg minh ring tap A = {23 — 1,3° -1,...,p* — 1} 1a he thu gon me b) Chimg minh rang II?_, (i? +i +1) =3 (mod p) Gidi Hién nhien méi phan tir cia A déu khong chia hét cho p. Gid sit i3—1 (mod p) > i = j3 (mod p) > * = j* (mod p) >. Theo dinh ly kt! = j9k+1 =] (mod p). Tit dé suy rai=j (mod p) +i=j. Vi o(p) = p— 1 =A] nen A 12 he thu gon mod p. b)Vi B = {1,2,...,p—1} la he thu gon mod p nén ta c6 IP_,(i3 —1) = (p- TP_y(i—1)I_9(i? +i +1) = (p—1)! (mod p) (p—-1)E_p(? +i+1) = (mod p) + IP_y(i? +i +1) (mod p). Suy ra T7_, (#2 +i +1) =3 (me Dinh ly Euler Cho n [a s6 nguyen duong va a € Z sao cho (a,n) = 1. a%") =1 (mod n) Mot cdch tng quét voi moi sO nguyen a € Z ta cd a® =a") (mod n) Chitng minh Lay mot hé thu gon B (mod n). Gid sit B = {by,b2, -..,bm m= $(n). Vi (a,n) = 1 nén tp aB = {aby, aby, ..., bm} cling 1 he thu go n). Thanh thir (ab,)(ab2)...(abm) = B-bp-wbm (mn0d n) Suy ra aby by..bm = 1-b2..bm (mod n) a") =1 (mod n) b) Ta phai ching minh T= — a9) = gh 4") (qh") — 1) =0 (mod n) Gia sit n c6 phan tich tiéu chudn n = p?...pft. VGi mdi i cO dinh ta chimg chia hét cho pi‘ .Dé cho gon ta dat p = p;,t = Néu (a,p) = 1 theo phan a) ta c6 a") — 1 chia hét cho p'. Vi ham nhan tinh nén 4(n) chia hét cho (p*). Suy ra a") — 1 chia hét cho a*P") — a") — 1 chia hét cho p*. Vay T chia het cho pt. Néu pla. Khi d6 a"~") chia hét cho p"~#), Mat khéc n—6(n) > t bdi nhit ¢ sO khong nguyén t6 véi p 1a p!,p?, ...,p*, Suy ra a") chia hét cho | chia het cho pt 34 Tém lai véi moi i = 1,2, cho n. H6 qua ( dinh ly Ferma) Néu p la mot sO nguyen t6 va a € Z sao cho (a,p) = Khi d6. k ta déu c6 A = 0 (mod pi!) . Suy ra A chia hét a?-1=1 (mod p) V6i s6 nguyén a bat ky thi a? = a (mod p) ‘That vay néu p la s6 nguyén 16 thi ¢(p) = Dinh ly Wilson Cho n > 1 la s6 nguyén dutong. Khi dé 7 1a sO nguyén 16 khi va chi khi (n-1)!=-1 (modn) Chutiing minh Gia sit p la s6 nguyen 16. Néu p = 2,3 thi két luan ding. Xét p > 3. Xé tp A = {2,3,...,.p— 2}. Voi méi k € A t6n tai duy nhat h/,1 Ski (k-1)(k +1) 0 (mod p). Suy ra k = 1 hodc — 1 Mau thuin. Dé thay (k’)! = k. Vay cdc phan tir cla A duoc ghép thanh (p —3)/2 cap (k, k') vu6i kk’ = 1 (mod p), Thanh thir 12—3k = 1 (mod p). Do dé (p - 1)! =1(p- 1) = -1 (mod p). Dio lai gid sir (n — 1)! = -1 (mod n) van = ab voi Njbja; jal 1 (mod ni), Njb; = 0 (mod my)néui # j. Vay a = a Ta 06 vOi mbit do Nib; (mod ni). Né&w b =a (mod n) thi b= 0 =a; (mod nj). Neue lai néu b thda man (1) thi b — a chia héit cho n; véi moi i. Vi ny,ng,..., ng 40i mot nguyen t6 cing nhau nen suy ra b— a chia hét cho n hay b= a (mod n). Vi dy 1 Tim s6 nguyen duong nbd nhat 6 tinh chat: Chia 7 du 5, chia 11 du 7 va chia 13 du 3, Gidi Ta c6 ny = 7, TAL = 77. Tacé Nib = Vay 11.13 = 143;ng = 11, No = 7.13 = 91;n3 = 13 by = 1 (mod 7) > by . Tuong tr by = Ab a = (143)(—2)(5) + (91)(4)(7) + (77)(3)(—1) = 1430 + 2548 — 231 = Tat cA cdc sé théa man cé dang b = 887 + 1001k. Vay 887 Ia s6 cin tim, Vi du 2 Chimg minh hop s6 véi moi n. Gidi Xét cdc s6 Feema F, = 2?" + 1. Ta nhan thay cdc sé Fy doi mot ng cing nhau. That vay gid st m > n va p|Fn, p| Fim. Ta cS g tn tai sO nguyén duong k sao cho un = 2" 2" = (Qyen" Ma 2? = -1 (mod p). Suy ra 1 = —1 (mod p). Vo ly vip ke. Ta da biét Fs c6 phan tich nguyen t6 1a Fs = (641)(6700417). Dat p = 6 6700417 Theo dinh ly thang du Trung hoa tén tai sO nguyen duong k w maa(Fy, F2, F3, Fi,p,q) sao cho k=1 (mod Fm),m=1,2,3.4 k=1 (mod p) 1 (mod q) ‘Ta ching minh k 1 s6 cdn tim. That vay gid sit n = 2b vdi b 1a so le. Néu thi up = 2k +1 = 2" 41 = (2?")'+150 (mod Fm) Va Un > k > Fim. D 1a hop so. Néu m = 5 tacé u, = 2k +1 52% 41 = (2”)+1=0 (me Un > > p'do d6 up, LA hgp s6. Néu m > 5 ta cé n= 2%c voi c la s6 chén. 2 = 2") = (F-1)°=(-1)'=1 (mod g) do dé. eb lS Vi tn > k > q nen un [a hop sé. Sé chinh phuwong(mod n) Dinh nghia Cho s6 nguyén duong n. SS nguyen a duge goi 1a 6 chinh (mod n) néu t6n tai x € N sao cho Un = 7 41=-14+1=0 (mod gq) z?=a (mod n) R6 rang mot s6 chinh phuong sé Ia s6 chinh phuong (mod n) véi moi n. TD mot s6 khong chinh phuong 6 thé 1 sé chinh phuong theo mét mod n nao d¢ han 2 I s6 chinh phutong (mod 7) vi 3? = 2 (mod 7). BAi tofn: C6 t6n tai chang s6 khong chinh phuong a nhumg 1a s6 chink (mod n) véi moi n? V6i mdi sé nguyén duong n cho truuéc ta mudn tim mot tiéu chudn dé 1 mét s6 a khi no 1a s6 chinh phuong (mod n). L Trudng hgp n=p la sé nguyén td. Hién nhien néu pla > a = 0 (mod p) thi a [a sO chinh phuong (mod p) ta chi xét (a,p) = 1 56 Dinh ly Cho p 1a sO nguyen 6 i) Néu p = 2 thi moi sO a 1é déu [A 86 chinh phuong (mod 2). ii) Néu p> 2. Khi d6 JA s6 chinh phutong (mod p) khi va chi khi a’-D/2=1 (mod p) Q) a 1a s6 khong chinh phuong (mod p) khi va chi khi a?-/2 =_1 (mod p) @) Chang minh Gia sir a 1a s6 chinh phuong (mod p). Vay t6n tai tén tai c EN sao cho z?=a (modn) Do (a,p) = 1 nén (x,p) = 1. Ti d6 af“)? = 2-1 = 1 (mod p) theo dinh ly Ferma. Dao lai gid sit c6 (2). Voi mbi k € {1,2, ..,p—1} t6n tai duy nhat k’ € {1,2,..,p— 1} sao cho kk’ = a (mod p). Néu t6n tai k = k’ thi ta c6 kk’ = k? =a (mod p) suy ra a Ja s6 chinh phuong (mod p). Néu tri lai thi tap {1, 2, ....p — 1} duoc chia thanh (p-1)/2 tap con hai phan ti {k, k/} ma tich cba ching déng du véi a(mod p). Suy ra (p—1)! = a®-)/? = 1 (mod p). Nhwung theo dinh ly Wilson (p—1)! = —1 (mod p) .Vay 1 = —1 (mod p) Mau thudn do p 18. Do dinh ly Ferma v6i méi s6 nguyen a khong chia hét cho p hoe a-2)/? = 1 (mod p) hoac a-))/? =-1 (mod p). Tir dé suy ra (3). Hé qua Cho p [a s6 nguyén 16 1. Khi d6 « Tich cda hai s6 chinh phuong (mod p) Ia s6 chinh phuong(mod p) @ Tich cia hai s6 khong chinh phuong (mod p) 18 s6 chinh phutong(mod p) © Tich cia mot s6 khong chinh phuong (mod p) véi mot s6 chinh phuong(mod p) a sO khong chinh phuong (mod p) (1) IA sO chinh phuong (mod p) khi va chi khi p = 4k +1 Chimg minh suy ra truc tiép tir tieu chudn tren. Dinh ly Néu p 1a s6 nguyén 16 1é thi trong tap S = {1, 2, ...,p—1} e6 (p—1)/2 s6 chinh phuong (modp) va (p — 1)/2 s6 khong chinh phuong (modp) Chitng minh V6i mdi i € 51 = {1,2,..., (P—1)/2} goi ry € SIA sO (duy nhat) ma # =r; (mod p). Ta thay ring rj # rj néui # j. That vay néur; = 1; 3 i? -j? 20 (mod p) (i— j)(é + j) = 0 (mod p). Nhung diéu nay khong xdy ra vii — j,i +j déu khong chia hét cho p do 1 < |i ~j] p/Mirk+i/dAeizk+l. Vayn=k T néu p = 4k—1 thi tap tat cd cdc s6 chan trong khoang (0, p) 1a {2i}, 3 2 062i > p/P i>k—-1/4 idk. Vayn=k. Tido suy a2P-)/? = 1 ( khi va chi khi k = 2¢ p= 8+ 1. Sau day IA mot 4p dung hay. Vi du ( Thi HSGQG 2004 bing A) Ky higu $(n) 18 téng cdc chit s6 cia gid ti nhé nhat cha S(n) khin chay tren cdc boi cua 2003. Gidi Dat p = 2003. p 1a sO nguyen 16. Ro rang S(n) > 1 vi 1O* khong « cho p. Gia sit tén tai n 18 boi cia p va S(n) = 2. Suy ra t6n tai & dé 10° (mod p). Chi ¥ rang 2° = 1024 = 10? (mod p) nen (2*)? = 210k = 107" = (10*)? =-1 (mod p) Vay —1 1a sO chinh phuong (mod p). Mau thudn vi p khong c6 dang 4k+1. Tiép theo ta ching minh t6n tai 7 1a bdi cia 3 ma S(n)=3. Ta 06 107 = 21 + 2.10700 = 21001 — 2(P-1/2 = 1 (mod p) vip #8t+1. Vay n =2.10™ + 1 1a boi cia p va $(n) = 3. Thanh thir gic ahét cla S(n) khi n chay trén céc boi chia 2003 1a 3. Ta xét bai todn tigp theo sau day: Tim tat cd cdc s6 nguyén t6 p lé (p, 3) cho 3 1A sO chinh phuong( mod p) Dinh IyCho p [a s6 nguyen t6 18 (p,3) = 1. Goi n fi 6 ca 56 1 boi ed trong khoang (p/2;p). Khi d6 3-D/? = (-1)" (mod p) 58 Chitng minh Gid sit r1,79,...,7m 12 tap tt c& che s6 boi cha 3 nim trong khoang (p/2; p),81, $2, «.-» Sm IA tap tat ca c&c s6 boi cha 3 nim trong khoang (0; p/2) vaty, ta, ..., tn Ia tap t&t c& cAc s6 bdi cha 3 nam trong khoang (p; 3p/2). Ta cé m+n-+h = (p—1)/2. ‘That vay m+n +h la s6 céc boi cha 3 nam trong khoang (0;3p/2) tie 1a s6 cdc 86 4 V6i 3i < 3p/2 1 3[r+s= t—p-+ 3|t+r = 2p déu din dén 3[p). Vi (p -1)/2}-Vay srontn(— nlrb =P)ttn 9) = (254): 8 S)eSmPteotgtreonty (=1)" 3): (mod p) Nhung SSP Paty aty = 3D? ( $ *\), Vay suy ra 30-1? = (-1)" (mod p) Hé qua Cho p = 6k + 1 JA s6 nguyen 16 . Khi d6 30-07? = (-1)* (mod p) ‘Ty dé suy ra 3 1a s6 chinh phyong (mod p) khi va chi khi p= 12¢ +1 Chitng minh Gia sit p = 6k + 1. Tap tt cd céc s6 chia hét cho 3 trong khoang (0,p) 1a {3i},i = 1,..,2hTa 06 31 > p/2 i >kt1/6eidk+1. Vay n= k, Tuong ty néu pk — 1 thi tap tat cd cdc s6 chia hét cho 3 trong khoang (0,p) Wa {3i},i = 1,...,2k — LTa c6 3i > p/2 ei > k-1/6ri>k. Vayn=k. Teds suy ra 3-1)/2 = 1 (mod p) khi va chi khi k = 2¢ p= 12141. Vi du Tim tat cd céc s6 nguyén duong n sao cho 3" — 1 chia hét cho 2" 1 Gidi R6 rang n = 1 théa man.” Xét n > 1 va gid sit 3" ~ 1 chia hét cho 2" —1 Néu n chin thi 2" —1 chia hét cho 3 nén 3” — 1 chia hét cho 3. Vo ly. Vay n lé. Név p lA mot u6c nguyén 16 bat ky cla 2" — 1 thi suy ra 3° = 1 (mod p) + 3°47 (mod p). Vin +1 chén nén 3 [a s6 chinh phuong (mod p).Theo dinh ly trén p 06 dang 12t +1. Do moi w6c nguyén 16 cha 2" — 1 06 dang 12¢ +1 nén 2" —1 c6 dang d6. Néu 2" — 1 = 12t—-1 + 2" = 12¢ + 2" = 0 (mod 3). Mau thuan. Néu QP 1 = 1241 2" = 128425 2) = Gt + 1. Mau thudn vi Gt +1 1A S618. Vay n = 1 1 sO duy nhit théa man dé bai. Vi du 1 Cho p > 3 [a sO nguyén 16 c6 dang p = 3k +1. Ching minh ring TE, (i? +i+1) =0 (mod p) Gidi Déu tién ta ching minh rang néu p = 3k + 1 thi -3 JA sO chinh phuong (mod p). That vay k phdi chin k = 2] suy ra p = 61 +1. Néul = 2t 4 p= 121+1 — (p—1)/2 = Gt 1a sO chan nén (-1) 1a s6 chinh phuong (mod p) va 3 la s6 chinh phuong (mod p). Thanh thir (-3)=(-1)3 [a sO chinh phuong (mod p). Né&vl = 2t+1 4 p= 12t+7 > (p—1)/2 = 61 +3 1a sO 18 nén (-1) 1a 86 khong, chinh phuong (mod p) va 3 cing la sO chinh phuong (mod p). Thanh thir (3)=(1)3 14 s6 chinh phuong (mod p). Do d6 t6n tai € N, x lé dé x? = ~3 (mod p). (Ta o6 thé gia sir x 1A s6 18 vi n€u @ chdn ta thay x boi 2 +p). Gid sir e=2k+1 > (2k4+1)? +3 = 4(k? +k 41) 30 (modp) 3 P+ k+ (mod p). Gia sir i (mod p) v6i 1 p/2}, B= {ralre < p/2}. Ta 06 1°), = {re A}"h(rpeB}e —1)"Hgreap(p — re) gr,en}rk (mod p) Ta c6 véi r, € A thi (p — rg) € (0;p/2). Them vao d6 véi ry € A,r; € (p—ri) # rj. That vay néu (p— 7) = 7) Orit) =p > ait (mod p) 3 i+9 (mod p). Piéu nay khong thé vii + j pi} Ao = {(i, H)lai < pi} As = {(i, j)|qi = pi} Tac6 Ag =. That vay gi = pj + pli + p 1. va ld 86 8. Khi dé a 2 s6 chinh phuong (mod n) khi va chi khi: i) @ = 1 (mod 4) néu s=2. ii) a = 1 (mod 8) néu s > 3. Chitng minh i)V6i s=2 thc la n=4, Néu t6n tai z € Nsao cho 2” = 1 (mod 4) vi v6i moi s6 18 x 2? = 1 (mod 4). Dao lai néu a 1? (mod n). Vay a la s6 chinh phuong mod n. ra x’ a (mod 4) + 1 (mod 4) ““ii) V6i s > 3.Néu tén tai z € N sao cho a = 2? (modn) > a= (mod 8) .Vi véi moi sO 1é x 2? = 1 (mod 8) nén 1 (mod 8). Dao a= 1 (mod 8) tie Ra = 8 +1. Véi s = 3 n=8 thi hién nhien ali phuong (mod 8). Xét s > 3.Nhu vi du | da chi ra t6n tai he day dit (mod 2°~? 86 tam gidc tic 1a t6n tai k € N Sao cho k(k+1)/2 = t (mod 2°-3) + 4k(k (mod n) + (2k + 1)? = 8¢ +1 =. (mod n).Vay a Ia sO chinh phuong m Dinh ly Gia sit n = p* v6i p 1A s6 nguyen (6 18. Khi d6 a 1a sO chin mod n khi va chi khi a 1a s6 chinh phuong mod p. Chitng minh Hién nhién néu a 1A sO chinh phuong mod n thi a I phuong mod p. Gi sit a 1a sO chinh phuong mod p. Ta ching minh bing khang dinh sau: V6i méi k t6n tai 2, € N sao cho 2? =a (mod p*). Véi k: dinh ding vi a 1a s6 chinh phuong mod p. Gia sit ding voi k te 1A tén tai x ah = a+ tpk voit Z. Dat wey) = xy + hp* voi h la s6 nguyen duong 2hxy, = —t (mod p). SO h nay tén tai vi (p, 2z—) = 1. Vay ap, = of + 2hayp* + p™* =a+t(2hay +t)p* + p* =a (mod p*) bai vi 2hz; +¢ chia hét cho p va 2k > k+1. N6i ring t6n tai x, dé 2? =a hay a 1a sO chinh phuong(mod n). Ky hiéu Jacobi Ky hieu Jacobi 1A sy mé rong cba ky hiéu Lagendre 14 mot sO nguyen duong 1é v6i phan tich tieu chudn n = ITH_1p;. ( Céc pj 06 nhau). V6i (a,n) = 1 ta dinh nghia ky higu Jacobi nhu sau = (2) DE thay ring néu a [a s6 chinh phuong mod nti (2) =1 vi (8) = nguyc lai khong dting. Ching han (35) = (3) (2) = (-1)(-1) = 1 nhuumg 14 s6 chinh phuong mod 15 vi 2 1a khong Ia chinh phuong mod 3. Tuy nhiés Jacobi cing c6 ddy di cdc tinh chat nhu ky hiéu Lagendre. Cu thé Dinh ly Gia sir n 1a s6 nguyen Ie, @ 1A sO nguyen véi (a,n) = 1. 1. Néu a= (mod n) thi (2) = (4) ( 4 (= nena @=cyeron ¢ 6. ( Luat tuong hd ) Néu n,m la cdc s6 nguyen 1é nguyen 16 cing nhau (3) @)-carveme 64 Vi du Cho m,n Ia céc sO nguyen duong sao cho n +1 chia hét cho 4m. Ching minh rang —m 1a s6 khong chinh phutong (mod n). Gidi Gia sit —m 1a 86 chinh phuuong mod n. Say ra 1— (2%) = 1 wi theo gid thiet n = 4km — 1 nen ( sit m = 2%b voi b lé. Ta c6. m ie b ()= (=) (2) ® a) Ta chtmg minh (2) = 1. Néu s chén thi hién nhién. Néu s 1é suy ram = 21 + n= 4km—1= 81-1 (2) =(-1)-Y8 =1- (F) =1. bh ching minh (2) = 1. Tacé theo luat tong hé (?) = (YCRCH) (F) = (ane (2) SB) = 1 (8) (@) = (-1)0-/? = -1. Gid n do (n — 1)/2 Ié. Lai 06 do n = —1 (mod b) - (#) = (54) = (-1)@-Y? vay b\ ca ( \ = (-1)' =1 do b — 1 chan. Ti a) va b) ta c6 (7) = 1 mau thudn véitr (6). Vi du Chimg minh ring phuong trinh 4ayz = 2+y+t? khong c6 nghiém nguyén duong. Gidi doyz =a ty +t? + l6xyz = 4a + dy + 40? l6ayz? = doz + 4yz + 420? & 42t? +1 = (daz —1)(4y2 -1) @ (Q2t)P = z (mod n) 8 d6 n= 4yz —1. Ta c6 n-+1 chia hét cho 4z va —z 1A sO chinh phuong mod n. Trdi v6i vi du 2. C&p cha m6t 56 Cho sé nguyen duong n. Néu a 1a mot s nguyen véi (a,n) thi ludn t6n tai s6 nguyén duuong k dé a* = 1 (mod n) ( ching han k = gn). SO nguyén duuong k bé nhat théa min a* = 1 (mod n) duge goi 1a ctip cba af mod n). Dinh ly i) Gia sit cp cita a mod n 1a h. Khi d6 a* = 1 (mod n) khi va chi khi k chia hét cho h. ii) Nu a c6 c&ip h (mod n), b 6 c&p 1 (mod n) va (h, 1) = 1 thi ab c6 ep Al (mod n). iii) Cho c4c s6 ny. ng, ....m, d6i mot nguyén 16 cing nhau va n = nj...n, Gid sit v6i mbi i, hy 1a cp cha a (mod p;). Khi d6 cép céa a (mod n) A h=BCNN Ura, Fay.» Fax). Ching minh i) Gid sit a* = 1 (mod n). N&uk = gh+r,1 v Chiing minh Ta c6 h\d(2*) + h = 2t,t < s—1. RO rang néu u>s thia- het cho ni do 46 k= 1. Néus >uthih>2-+¢>1Tacd a® —1 = (a? -1)(at +1)..(a" +1) Néu u <5 v thi tir ding thitc () suy ra h a v+t—1 NG chia hét cho 2° khi va chi khiv+£-1>s—£>s8+4+1—v>2. Vayt t=s+l—o. Dinh ly Cho p A sO nguyen t6 18 (a,p) = 1 van =p’. Gid site a(mod p) va a” — 1 = pq, (gp) = 1. Goi h 1a cp cia a (mod n). Khi db r nduuds rp" névu s thih = 1. X « Gia sit h = p'g, (q,p) = 1. Néug > 1 thidoa = 1 (mod p) néna’—1 = (0 vi (b,p) = 1. Vay a —1 =1 (mod n) trdi gid thiét h 1a bé nhat. Vay h = 7 . Ta c6 b6 dé Bé dé Véi moi n EN a” —1=p"A,(A,p) =1 Chimg minh quy nap theo n. Véi n = 0 ding. Gia sit ding vi n. Dat b = al ye a P-L =(b-1Y(P I +- +1) oR Vio = 1 (mod p) va p 18 nén dé thay B chi chia hét cho p ma khong chia p®. Do dé a?! — 1 = p"*"414, (A, p) = 1.B6 dé duge chig minh Theo bé dé ta c6 a —1 = p't"A,(A,p) = LVaytt+u>s+t>s- Vay ¢ bé nhdt At = s-useh=p*. 66 Tré lai tru’ng hop r bat ky. Khi 66 a" = 1 (mod n) + a! = 1 (mod p) +h = Ir. Dat b= a". Via" = b! dé thay cap cla mod p) IA 1 va 1 1a cp cia b (mod n). Vay theo tring hop dac biet r = 1 4p dung d6i véi b ta c6 jaf 1 nurs ~ ) p= névucs Vay h= rl. Dinh ly duge chimg minh. Tém lai viéc tim c&p cha mot s6 theo mod n duge quy hoan toan vé tim cp theo modun nguyén 16. Vi du Xét s6 Feema F = F, = 2°"+1,n > 1. Chung minh ring F 1a s6 nguyen 16 khi va chi khi BPW gy chia hét cho F. Gidi Dé thay F khong c6 dang 12k +1, Do dé néu F 1A sO nguyén t6 thi 3 1 so khong chinh phuong (mod F). Vay 3°°-))/? = —1 (mod F) tie 1a 3("-2)/2 4.1 chia hét cho F. Dio lai gid sit 3(F-D 2-1 (mod F). (ay Ty (11) suy ra 37-1 = 1 (mod F). Goi h la cp cia 3 (mod F). Khi d6 h|F—1 = 2” - Vay k= 2t,t <2". Néut < 2"—1 thi h|(F—1)/2 do d6 3*-0/? = 1 (mod F) mau thudn v6i (11). Vay t = 2" h= F-1. Vihjo(F) nen F -1/6(F) > F-1=(F). Vay F 1 s6 nguyen 6. Phuong trinh déng du Cho f(x) 1a mot da thitc hé sO nguyen va m 1a mot sO nguyen duong. S6 nguyén a duge goi 1a nghiém cla phuong trinh déng di f(z) =0 (mod m) (2) néu f(a) =0 (mod m). . R6 rang néu a [A nghiém cia (12) thi véi moi b = @ (mod m) b cing [a nghiém cla (12). Do d6 hai nghiém déng du (mod m) duoc déng nhat v6i nhav. N6i cach khdc ta chi xét céc nghiém phan biét (mod m). Ging nhu v6i phuong trinh dai sé ta quan tam dén sO nghiém cia mot phuong inh déng du. Dinh ly Cho da thie f(x) = anz"™ +--+ a1x +.a9 hé s6 nguyen . Xét phuong tinh déng du S(z) =0 (mod p) (13)' trong d6 m = p IA mot s6 nguyen 16. Néu phuong tinh (13) c6 n +1 nghiém phan biet (mod p) thi moi hé s6 a;(é = 0,1,2,...,.) déu chia hét cho p. Noi ring khi 46 f(a) =0 (mod p)¥Va eZ Chitng minh Chimg minh quy nap theo n. Véi n = 1 khing dinh ding. That vay gid sit a,2; + a9 = 0 (mod p),a;x2 + a9 = 0 (mod p) va 21 = az (mod p). Tr¥ hai vé suy ra ay(2 — 22) = 0 (mod p) + a; =0 (mod p) -+ a9 = 0 (mod p) Gia sit khing dinh ding v6i moi da thic bic k < n.Gid sit phuong trinh (13) 6 n +1 nghi¢m phan bit 21,....2,4; (mod p). Xét da thite g(x) = f(x) — a,(2 — 2})..(e — 2p). Ta 06 deg(g) f(any1) = 0 (me On (Zn41 — T1)-\(Tn41 — Zn) =O (mod p) + an = 0 (mod p). Xét da thé F(x) = anv” = anya"! +++» +ayu tag. Ta cd deg(h) p—1 ma |Al < k, |B] < p—1—k nén du bang xay ra, Vi du Cho p [a sO nguyén t6 lé. Voi mdi k nguyén duong,ky higu 3(k) = Tim t&t cd cde & dé Sy, chia hét cho p. Gidi N&u k chia hét cho p — 1 thi i = 1 (mod p) + 3, =p—1 (me k khong chia hét cho p gid sit k = (p—1)l+h,1 dayx = db, (mod dm) + 0 (mod m). ii) Gid sit zo 1A mot nghiém ca (15). Khi d6 a,29 = by (mod mj). Dat 2, = 29+ km, (ke = 0,1, ...,d—1). Khi d6 a,x, = 0,29 + kaymy = a129 = b) (mod my). Vay 2, a nghiém cia (15). Cc nghiém nay phan biét (mod m). That vay 2; = 2; (mod m) 4 im, = jm, (mod m) & i= j (mod d) > i= j. Néuz 1a mot nghiem bat ky thi yx = by (mod my) ~ a;(x — 20) = 0 (mod my) + # = 29 + Im, = a9 + (k+td)my = 2% +tm-— © = 2; (mod m) Vi du Chimg minh ring v6i moi n> 5, phuong trinh az — P(x) =a? +1582? —11a+38=0 (mod 3”). (16) phuong trinh cé ding 9 nghiém phan biét . Gidi Trusc hét ta ching minh nhan xé 1; Néu a = 1 (mod 9),a! = a +3"-2b thi P(a)+3"bh (mod 3"*) voi (h,3) = 1 That vay dé thay do 3n — 6, 2n —3,2n -4 > n+1 nen P(a’) = P(a) + 3°""(a? + 102a - 37) Vi a? + 1024 — 66 = 3 (mod 9) suy ra a? + 102a — 37 = 3h vii (? ‘Thay vio ta c6 diéu phai chimg minh. Buée 1:Phuong trinh (16) cé it nhat mot nghigm a va a = 1 (mod 9) minh bing quy nap. Véi n = 5 cé nghiém a = 19. Gia sit khing dinh ding IA tén tai ay voi P(an) = 0 (mod 3"). Dat P(a,) = 3"v. Dat ana1 = an Ta sé tim b dé P(an+1) = 0 (mod 3"+1). Theo nhan xét 1 P(an41) = P(a)+3"bh (mod 3"*') 3°(v +bh) (mod 3°*1) Vi (6,4) = 1 nen chon duge 2 dé v + bh chia hét cho 3 do dé P(an (aod 3 1). Them vao 46 an41 = an = 1 (mod 9) Buéc 2: Phuong tinh (16) c6 it nhat 9 nghiém phan bigt. Theo bude trinh (16) c6 it nhat mot nghiém a vi a = 1 (mod 9). Dat "2k = 0.1.2 a, =a 8) Tit nhan xét 1 ta cO P(a,) = P(a) + 3"bh (mod 3°41) + Plax) = J (mod 3”) Ta c6 néu a; = a; (mod 3") +i = j (mod 9) +i = j. Vay nay phan biet. Bude 4: Trude hét bang quy nap ta chimg minh néu nhan xét 2: Néu P(a (mod 3") thi a = a’ (mod 3"-*), Voi n=3 ta kiém tra duge nhan xét sit ding véi n. Néu P(a) (a’) (mod 31) + P(a) = P(a’) (mm a’ = a+ 3"~% theo gid thiét quy nap. Theo nhan xét 1 Pla’) = P(: (mod 3") — 3[bh + 3/b + a! = a (mod 3-1), Nhan xét 2 duge chime Gia sit a’ 1A mot nghiém bat ky ciia (16). Theo nhan xét 2 a! = a +3"~ Gia sith = 91-+k(k = 0, 1,...,8) Suy ra’ = a+3"-?k = a, (mod 3"). V trinh (16) c6 diing 9 nghiém phan biet. Vi du Cho p IA sé nguyen t6 18. Chimg minh ring véi moi n > 1 phe = a?-!_1=0 (mod p") 6 dting p — 1 nghiém phan biet. Gidi Chimg minh bing quy nap theo n. Véi n = 1 Khing dinh dir khing dinh ding voi n.Xét phuong trinh P-1_1=0 (mod p"*!) Gia sit {a4, ..., ap_1} A p—1 nghiém cia (17). Ta c6 a? -1 = pihi.Véin duy obit ty € {1,2,-.,p} thda man (p~1)ail+h; = 0 (mod p) vi (p—1)ae hét cho p. Dalby = aj-+tep". Ta ed BP! — 1 = (a; + typ )Po! = a?! 144 (mod p"*!) Ta c6 a?! — 1 + (p— Lajtip® = p(h + (p- Lait) =0( Do vay d; 1 nghigm eda (18 ). Bao lai néu bj = bj (mod p?!) = (mod p") + 4 = j. Vay phuong trinh c6 it nhat 9 nghiem. Bay gids gid sir (18). Khi d6 0 cng 12 nghi¢m cla (17). Vay t6n tai aj, 406 b = a; + tp" Ta c6 P-} ~ 1 = (a; + tp)P} = ab! — 1 + (p — 1) ajtp” = 0 (mod p"). Suy ra (p ~ l)ajtp” +h; chia ht cho p. Vay t = & (mod p) +4 = 4 +pl b= a; + tip" + pt) — b =a; (mod p™)), Vay phuong trinh (18) ¢6 ding p-1 nghiém I by, ..., bps Sau day 1a mot vai img dung cia phuong trinh déng du. Dinh ly Cho p Ja mot sO nguyén 16 1é. S6 nguyén duong A JA cfip cla mot sé a (mod p) khi va chi khi h B ude cba p~ 1. Chiing minh Ta chi can ching minh : néu hjp~1 thi h 2 céip ca mot s6 a (mod p). Trude hét xét trudng hop k = g° voi g [A s6 nguyén t, Goi AC {1,2,..,p— 1} 1 tap céc nghiém cba phuong trinh 2" 0 (mod p) va BC {1,2,...,p ~1} a tap c4c nghiém cia phuung trinh 2?”""' —1= 0 (mod p). Tacé BC AVAB#A (do JA] = h = p®, |B] = p*-}). Lay a € A,a ¢ B. Ta khang dinh rang caip cba a(mod p) la A. That vay gid sir [1a c&p cla a(mod p). Vi a" = 1 (mod p) nén Ih=p' a l=pt,t 7 sao cho g* (mod p) + p —1|i— j Mau thuin. Dinh ly sau day mé rong vi du cho k bat ky (khong nhat thiét A use cla p— 1) h ly Cho k 1a s6 nguyen duong va a € Z, (a,p) = 1. Gid sit d = (k.p ~ 1). ]6 phuong tinh g (mod p),> g 7 =1 Khi «gy 6 nghiém Khi va chi khi a®-N/4=1 (mod p) (20) Néu c6 nghiém né sé c6 ding d nghiém. Chitng minh Digu kin cin: Gid sit (19 ) 6 nghiém a. Khi dé alPD0/8 ax Hl? ANE _ (gpI) h/a 1 (mod p) do dinh Iy Feema. Diéu'kien di: Goi g 1& can nguyen thiy (mod p). Tén wi b € S = {1.2,....p—1} sao cho a = gb. Bude 1: Ta khing dinh d[b. That vay a1 g/d x1 (mod p). Vid Id can nguyen thiy nén suy ra b(p — 1)/d chia het cho p we Ia db, Buc 2: Theo vi du thi phuong trinh ku = b (mod p— 1) e6 d nghiém ph mod (p = 1). Noi each khie goi UC S' sao cho ku = b (mod p — 1) thi || Bude 3: Tap {a = g',u € U} 1 d nghiem phan biet (mod p) cla phon dang xét. That vay ta 66 o uel —+ku=b (mod p—1) +g (mod p) + a* =a (mod ‘Tiép theo ta chimg minh ching phan biét. Gia situ, u’ € U sao cho g = 9" thi w=! (mod p—1) (do g la can nguyen thay ). Vay u =u’, Bude 4: Néu x 1a nghiém cba (1) thi x = g" (mod p) trong dé u € UL ‘That vay tén tai u € S sao cho x = g" (mod p). Ta cé tk=a (modp) og=g’ (modp)ekusb (modp—1)eu¢ Dinh ly duge chtmg minh xong. Ta c6 thé m& rong khéi nigm sé chinh phuong (mod p) nhw sau: Cho k > nguyén dung va sO nguyen t6 18 p. SS nguyen a goi [A sO k-phutong (mod p) tai x € N sao cho =a (mod p). Tir dinh ly tren ta suy ra Dinh ly a la so A-phuong (mod p) khi va chi khi alp-v/e (mod p) tic 1A @ 1A nghiem cla phuong trinh déng dur 2P-Did (mod p) 5 d6 d=(k,p—1). Theo vi du phuong trinh (21) c6 ding 254 nghiém phan biet. Vay Dinh ly C6 ding 25+ s6 k-phuong 6 d6 d = (h,p — Vi du Xé&t k = 3,p > 3. Neu p = 3s +1 thi d= (3,35) = 3. Do dé phuong (mod p) néu va chi néu a® = 1 (mod p) va cé dting s sO lap phuong N&u p= 3s +2 thi d= (3,35 +1) = 1. Via?-! = 1 (mod p) nen moi sé lap phuong (mod p). PHUONG TRINH PELL Dang Hang Thang 1. Phwong trinh Pell Phuong trinh Pell 1 phuong trinh cé dang et ay=1 @ Pay 1A mot phuong trinh rat néi tiéng cd nhiéu tg dung trong viec gidi nhiéu bai todn sO hoc hay va kh6. Trong chuong nay khi n6i dé nghiém ta luon hiéu d6 1A nghiém nguyén duong. 1. Dinh ly 1 (vé su t6n tai nghiém) Phuong trinh (J) cé nghi¢m nguyén duong khi va chi khi d IA s6 khong chinh phutong. Chitng minh Néu d = m? thi (1) ws thanh 2 — my? = 1 (e@—my)(e+ my) =1loa—my=r1+my=lor=1y=0 Dio lai gid sit d [4 s6 khong chinh phuong.Ta can c6 céc bé dé sau B6 dé 1 Cho a [A mot sé vo ty. Khi dé 06 16n tai vo sé cap sO nguyén duong (h,k) véi & > 0 théa min Hed | Re Chitng minh Trude hét ta chimg minh nhan xét: V6i moi s6 nguyen duong g t6n tai (h,k) nguyén duong véi 1 < k v6i moi (hk) € A. Bay git 7 €N sao cho 1 o. Nhung } > b> la — 2 > thuin vdi (3).Bé dé duge chimg minh. Bé dé 2 Tén tai vo s6 cap s6 nguyen duong (z, y) thoa man jo? — dy?| <1 +2Vd Chiing minh Theo bé dé | tén tai vO sO cap s6 nguyen duong (2c. y) thea Suy ra Vay <14+2Vd Ta bude vao chuting minh dinh ly. Theo b6 dé 2 tén tai vo sO cap sé ngu (x,y) théa man Va uie la rong doan I = [-1 —2Vd; 14 2Vdj 06 vo 6 cap s6 nguyen duong ( tén wi bE I dé v6i vO sO cap s6 (x,y) ta c6 ju? - dy?| <1+ v dy = Goi H = {(2,y)|a?—dy? = k} vavéi 1 0 Név v = 0 thi u = £1 Suy ra (2) — wVA)(02 + Vd) = tk £(21 ~ ys Vd)(es + mV), Vay a2 + wd = 2 + iV > 2) = 22. = yp Ta 6 mau thudn. Dinh ly J duge chimg minh . Chi y: Nghiém bé nat cla phuong trinh Pell c6 thé rat Ién. Thi du phuong tinh x? — 61y* = 1 c6 nghiém nhé nhat 1a a = 1766319049, b = 226153980, phuong tinh a? — 109y* = 1 ¢6 nghiem nbd hai A a = 158070671986249, b = 15140424455100 Bay gid’ chting ta sé tim cOng thife m6 td tt cd cdc nghiém cia (1). Dinh ly 2( Cong thie nghiém) Gia sir (a,b) 18 nghiém nho nat cia phuong tinh a? —dy?=1 ngbia 1A b 1A sO nguyen dutong bé nhat dé 1 + dd? 1a s6 chinh phuong (theo dinh Iy 1 thi Ju6n tén tai) Xét day (a,) va (ys) cho béi hé thie tray héi sau &o = 1k) = A, ay 49 = Zaring) — By (29) Yo = 0.41 = b, Yn42 = 20Yn41 — Yn G0) Khi 46 (2r,,. y,,) 1 tat ca cdc nghiém cia phuong trinh Pell (1). Chiing minh Phuong trinh dac wung cha day (9) la 2? — 2ax +1 = 06 & = a —1 = db do dé c6 hai nghiém IA Ay = a + bVd,d2 = a ~ bVe Te diéu kién ban déu dé tim duge (a + ba)" + (a — bva)" 2 I= a+ bya)" (a- byaj* a Ta Tu dé By + yd = (a + bVE)", an — nV = (a — dvd)". GL wv = ( va)" va = ( fay" 2 — dy2) = (a? ~ db?)" = 1. Dé ching minh quy nap ring (z,) va diy tang cic sO nguyen duong. y) a mot nghiém bat ky cua (1). Ta sé ching minh khang Ton tai n dé etyVd=(atbvd) = 25 tynVd Phan chimg: Gia sit wai lai véi moi n ta c6 2 + yVd ¢ (a + bVa)". Khi dé wr nguyén duong sao cho (a+bVd)™ yVdatm > YmVd nen u > 0. Lai c6 (u — vv ad)(u + vad) Ll0. Vay (u,v) 1a cia (1) do dé6a< u,b 2 sao cho tam giéc c6 d canh 1A x — 1, 2, + 1 c6 dign tich [A mot sO nguyen. Gidi Goi S 18 dign tich tam gidc. Theo cong thifc He rong ta tim duge 0" (a? — 4). S= Fey 3(a? — 4) —> 16S? = Ta cd néu S € Zthie chin. Dat « = 2y suy ra S? = 3y?(y? — 1) - yV3(y? = 1) — 30-1) = zz + B(y? 1) = 92? > y?- Nguge Ii néu (y, z) 1 nghiém cia phuong trinh Pell 2 sh thi dé thay @ = 2y,y > 1 thda mn diéu kign dau bai. Nghigm nho nhat ci (2.1). Vay tat cd céc nghiém ea (13) A day(yn) cho biti yo = Ly =2.Ynt2 = Ynst = Yn Suy ra nghiém cia bai todn 1 day (x,) véi n > 1 cho bei = ay = by erpe = den pen 864,14 ‘Vi du 2 Tim tat ca cfc nguyén duong T sao cho sé tam gi: chinh phuong. Gidi T(T + 1) = 2y? —s AT? 447 41 = 8y? +1 — (27 +1)? — By? = 1 Dat 2T + LSuy ra (x, y) 1d nghiém cia phuong trinh c TU) fa mot so -8y=1 (34) Dio lai néu («,y) 1 nghigm cia (14) thi x 18 do d6 T = 75! théa man diu bai Nehiém nh6 nhat ca (14) 14 (3;1).Theo dinh ly wen ae trinh (14) ¢6 nghi¢m Ia day (c,) xde dinh bai tq = 1.0) =3,2n42 = 6tn41 Bq Khi d6 v6i 2, = 2Tp +1 ta C6 2Tp42 +1 = 6(2Trs1 + 1) — (2T, +1). Tir d6 day (Tx) xde dinh béi To = 0,7) = 1,Tn42 = 6Tn+3 — Tr +2 la day cin tim. D6 Ia cdc 86 1,8, 49,288,... Vi du 3 Tim tat cd cdc sé nguyén duong n sao cho tung binh cong cia n sO chinh phuong dau tien lai A mt s6 chinh phuong. Gidi Ta 06 , : —— V4 243% 40-40? _ (nt+1)2n4)) n 2 6 . Vay ta c6 phuong trinh (n + 1)(2n +1) = 6y? —> (4n + 3)? ~ 48,2 = 1. Dat a = dn +3 ta c6 phuong trinh 2? — 48y? = 1. Nghiém nhé nhat 1a (7,1). D theo quy luat a9 = 1.8 = 7.8ng2 = Mtn41 — Bp. Bing quy nap dé thay Voi n chén thi ay, = 1 (mod 4) , Voi n Ie thi 3 (mod 4). Vay ne = asa =3 Ja day can tim.Sau day ta thiét lap tutng minh cong thie truy hdi cha Ny. Dal Up = 2-41. TAC oae+3 = Marge 42-241 = 1414241 22x) — ek = 19622041 — 14a, — 2o¢41 = 195@2n41 — Zoe41 — TOm—1 = 194 aRG1 — Tan. VAY ung = 194Urg 1 — URVAY drys +3 = 194(4nz 43 +3) — dn, — 3 Ta d6 Mpeg = 19441 — MH + 144 V6i mg = 1,m = 337, ng = 65521... Vi du 4 Tim tit ca cdc sé nguyén duong n sao cho ci 2n + 1 va 3n + 1 déu la s6 chinh phuong. Gidi Vi (2n+1,3n+1) = 1 nén cd 2n-+1 va 8n+1 déu 1a 86 chinh phutong Khi va chi khi (2n+1)(3n+1) = y? Suy ra (12n +5)? —247? = 1 Dat r= 12n+5 1a dan ii phuong uinh Pell 2? — 24y? = 1 Nghiém nhé nhat Dé chutmg minh 2, = 5 (mod 12) khi va chi khi & = Waa — ae 2k+) = l0r2¢42 — Ta¢41 = O(a Rs) — 2x) — Ts) = 100K.) — 9922441 —22e41 — 22-1 = Supe — upVay 12ryo + = aeon41— aap. VAY upse = Sun 43 — B(12n.41 + 5) — 12my, — 5 Te d6 uz Vay ura2 = N42 = VN — MH + 40 voi my = 0.) = 10,7 Tir d6 suy ra mot 3960, i todn hay sau day: Chung minh ring ngu n {a mot s6 nguyén duong sao cho cd 2n +1 {4 sO chinh phuong thi n phai chia het cho 40. I.Phuong trinh Pell loai 2 Phuong trinh Pell loai 2 1a phuong trinh x—dy=-1 (Dd chinh phuong d = m? thi phuong tinh (1) wi thanh (my—. z)=14 my—x2=my+a2=1-— x2 =0 Vay phuong trinh khong c6 nghié b) Néu d c6 u6c nguyen t6 p = 4k +3 thi phuong trinh ciing vo nghiém. 1 gid sir (x,y) 1a nghiem. Khi dé a? + 1 = dy? + pix? +1. Vip cé dang 4 theo mot két qua quen thude | chia hét cho p. Ta cé mau thudn, Digu nguugc lai khong diing. Néu d khong c6 ude nguyén t6 dang 4k+3 thi trinh c6 thé vin vo nghiém .( Xem vi du 5) Tuy nhién, néu d 1a mot sé nguyé digu nguge lai cling dting. Cu thé ta cé Dinh ly 3 Phuong trinh Pell loai 2 c6 nghi¢m Khi va chi khi d # 4k +: Chitng minh Gia sit phuong trinh c6 nghiém. Theo b) d # 4k +3. Ng d#4k+3. N&u p = 2 phuong trinh x? — 2y? = —1 c6 nghiém (2, y) = 1 (mod 4). Xét phuong trinh Pell x — dy? = Ta goi dé 1a phuong trinh Pell len két véi (1). Goi (a, b) 1 nghiém nhé nhat c Ta c6 a? — 1 = db®. Néu a chan thi b I do dé L? = 1 (mod 4) (mod 4). Diéu nay khong xdy ra. Vay a Ié va b chan. Gia sit a = 2a, +1, = 6 (a—1)(a +1) = db? = aj(a, +1) = db?. Dod a 86 nguyen 16 va (a1, a1 + nen ta c6 a) =v. Néu ay =u, a, +1 = dv? 3 uw? — de’ (u,v). Néu a, = du?,a, + rmv>asta+l=v' xdy ra. Dinh ly sau day cho ta mot diéu kign cain va dit dé (11) c6 nghiem, Dinh ly 4. Goi (a,b) 1A nghiém nh nhat cia phuong trinh Pell lén két Khi dé (I) c6 nghiém khi va chi khi he a=24dy? b= 2ey Chitng minh. Gia sit (2x0, yo) 1 nghigm cla (16). Ta e6 a? ~ ab? — dyp) = 1. Vay ta c6 2% — dy = 1 hoac xh di —1. Néu trutng xay ra thi (29, yo) [A nghiém cia (1) do d6 wy > a = 23 +dyj > wo. Mau thu phai c6 2% — dy = —L tie A (xp, yo) IA nghiem cia (ID). = =1. Vay (I) 66 “6 nghigmn nguyé 3 v? — du? = 1. Vay (v,u) Ia nghiem cia ¢ >a = 2a; +1. Mau thudn. Vay kha nang nay 6 nghiem. 78 Dao lai gid sit (II) c6 nghiém. Goi (wo, yo) I nghiém nhd nha cia (Il). Ta sé chimg minh ring (29, yo) chinh 1A nghiém cia (16). That vay dat u = 22 + dyg,u = 2p, yo. Ta c6 u2 — dv? = (23 — dyg)? = 1. Vay (u,v) Ia nghiém cia (16). Suy ra u2a,v > b. Ta ching minh u = a,v = b. Gid sir wai lai tie Au > a,v > b, Ta 66 (a~ bya) < (a—Bvad)(a+ b/d) = 1 do 5 = ant wva) < zo tmvd oe (37) (a2 — dw) +. (owe bag) va 0. That vay 1 >0 © ay > bend <> (db? + 1)yg > (dye — 1b? & yf > —0?. Diéu nay ding. Néu s > 0 thi (s,v) la nghiém cia (Il) do dé s + tV/d > 29 + yoVa. Mau thudn vdi (18). Néu s < 0 thi (—s, t) 1a nghiém cita (II) nén suy ra —s +4/'d > x9 + yoVd. Mau thuin véi (19). Dinh ly duoc ching minh. Vi du 5 Ching minh ring phuong winh 2? — 34y? vo nghieém . ( Chi y ring 34=2.17 khong c6 uée nguyén 16 dang 4k+3) =-L ‘That vay phuong trinh x? — 34y? = 1 c6 nghiém nh nhat 1a (a;b) = (35:6). Xét he Tw phuong trinh thi nhat cla hé suy ra (ay) = (2:1). Tuy nhién (131) khong thda phuong trinh thir 2, Vay phuong trinh 2? — 34y? = —1 vo nghiém. : Nhén xét. Hé (16) néu c6 nghiém thi c6 duy nha nghiém . Do dé theo chimg minh tren nghiém duy nhat d6 chinh 1 nghiém nhé nhat cua (11). ‘That vay gid sit (wo, yo), (21,41) 1A hai nghi¢m cia (16). Khi d6 ta 6 29 + dug + QeoyoVd = 22 + dy? +2014: Vd = a+ bVd. Suy ra (wo -+yoVad)? = (ay +41 VE)? + zo+yoVd = 21+4; Vd. Do dla sé khong chinh phuong nén ta rit ra zg = 2), yo = 1 Tir d6 suy ra nghiém nhé nhat cia (I) ludn bé hon nghiém nhé nhat cia (1).Nhan xét nay gitp ta tim nghiém nhé nhat cla phuong trinh Pell (1) nhanh chéng khi ma nghigm ma nghiém nhé nat dy lai rat én. Vi du 6 Tim nghiém nhé nhat cha x? — 29)? = 1 Phuong trinh x? —29,? = —1 cé nghiém nhé nhat 12 (70;13). Vay nghiém nhé nhat cha phyong trinh Pell dang xét A a = 70? + 29(13)? = 9801, b = 2(70)(13) = 1820. Dinh ly sau cho ta cong thie nghiém. Dinh ly 5. Gid sir he phuong trinh (16) c6 nghiém va (u,v) 1 nghiém duy nha cha né. Xél day s6 nguyen duong (2). (yn) xde dinh boi x9 = ua =u + Bduv?, 242 = 2otn41 — Bn (40) yo = By = de? + 8u7v, Yee = 2ayn41 — Yn (ly Khi d6 (2rp, yy) 1A Wt ed cdc nghigm cia cia (M1). Chiing minh Diu tien ta chimg minh ring = (utevd)yr*t Phuong trinh dae trung ciia day (8) 1A 2? -2ar+1=0c6 A =a? -i= d6 6 hai nghiém 1 Ay = a + bd, Ag = a —bVd. Do d6 tp = Ci(a + Cola - bVd)" = Cy(u + Vav)?" + Ca(u — Vav)” . duce C; = “43¥4 Cy = “24 Do vay tn = Tuuong tr ~via (ut vv)! — ( m= ai Suy ra (22) va (23), Nhan (22) véi (23) vé véi vé ta duge 22 — y2Vd = we Vadyent! a1, Dio lai ) IA nghiem cia (ID). Xét so dyv) + (yu +a) sttvd vii s (x? ~ dy?)(w? ~ dv?) = (-1)(-1) (D. Do dé tén tai n € N sao cho sttvd = (a+dVay""! = (a + yVd)(u + vd) = (u + vVd)yPr?? atyVd= (uted! = an +ynVd > (x,y) = (2n, Yn) yen (a + yVa)(u + vv) aut dyv,t : Vay (s,¢) 14 nghiém cla phuong tr Vi du 7 Tim tat ca cdc s& nguyen duong n cé tinh chat n? + (n+ 1)? [a s phuong. Gidi n? + (n +1)? = y? @ (2n 4 1)? +1 = 2y? & (2n 4:1)? — 2y? = - x = 2n+1 ta cé phuuong trinh 2? ~2y? = —1. Theo dinh ly tén nghiém 1a to = 1,01 = 7, 2h 42 = 6rey1 — tq. TH d6 2nyyg + 1 = 6(2rg41 +1) -2 Cac $6 cin tim duge cho béi diy (nj) sau day ng = 0,n1 = 3, M42 = Gree — Me +2 20; 119 ... I.Phwong trinh Pell chiva tham sé n Phuong trinh Pell chia tham s6 n 1 phuong trinh -dp=n (ID trong d6 d 1A sO nguyen duong khong chinh phuong cdn 7 IA s6 nguyén. Dinh ly 6. Phuvong tinh (II1) hode vo nghiem hode ¢6 vO sé nghiem. Chiing minh. Gia sit (MD) c6 nghigm (en, yn) te BA 80 Goi (a,b) Ja nghi¢m céa phuong trinh Pell (1). Khi dé t @ dan (45) Nhan (24) véi (25) ta duge (2% — dy) (a? — di?) = m4 (ana + dyad)? ~ dln + 00)? Dat In41 = ya + dyad (46) and + Yaa (47) Uns = Ta thay (27941, Yn41) TA nghiém va tp < tp41,9n < Yn4i. Vay cong thie () cho ta vO sO nghiém cia (III) néu (1) c6 mot nghiém khéi dau (21, 91). Cha ¥. Néu ta chon nghiém khdi dau 1 nghiém bé nhat thi trong trudng hop n= 1 cOng thttc (26) va (27) cho ta tdt cd cdc nghiém. ‘That vay : V6i n = 1 theo cong thite (11) (a+ 6d)" 2m + nV. Suy ra Ing + imsiVd = (a+ dbVa)") = (a+ bv)" (a + bVd) = (an + ynVd)(a + bd) = (tna + dynb) + (nb + yna)Vd Vay y+) = Ina+ dynb, Yn4y = nb + yna. V6i n = —1 theo c6ng thite (22 ) ta c6 (x9 + yoV)?"4! = an + yn V/d. Suy ra trait ynsiVd = (20 + ov)? = (210 + yoV'd}" (29 + yoVd)? = (a, + unvVa)(a + 2) = (2,6 + dumb) + (@nb+ yna)Vd > (via = 2} + dyg, b = 2x40.) Vay Tug) = A+ dyy.b, Ynsi = Ind + yma Tuy nhien véi diéu nay khong ding. Day (26) va (27) khong nhdt thiét véi can t4t cd cdc nghiém cia phuong trinh (IID). Thf du sau day minh hoa cho diégu 46. Vi du 7. Gidi phuong trinh oy = (48) Xét phuong trinh Pell tong tng x? — 5y? = 1. Dé thay nghiém nh nhat cia phuong tinh Pell 1a (a,b) = (9,4) va nghiém nho nhat cua phuong imh (48) 1a (2.41) = (1;1). Do vay cong thie (26) va (27) cho ta day nghiém (27, yn) sau day Tn41 = 9% +20Yn, Yrgr = Aon + Yn Cu thé dé 1a cae nghiém sau (1:1), (29:18), (521 (9349. 4181). 33) Tuy nhien a6 chua vét hét nghiém, Ching han r6 rang (4;2) 1d nghi¢m trinh nhung khong c6 mat trong day tren. Bay gid ta chimg minh tit cd cdc (vn. Yn) cha phuong trinh (48) duoc xic dinh bi quy twat sau zel mw=i, 1) True hét ta ching minh (2a, yn) 1a nghiém. Cach 1 Ta cé wh —Btny itn +22 —tnatte1 +22 = 2 x Vay 224) —3¢n¢1tn +02 = 03 — 32,22 +02 =5 Tuong ty y24) — Bynsitn +92 = 08 — 3yrye ty = 802 +5 - Bante = 32n(32n — 4 802 4545-22 — 22) = 70? -— 22 1 +10 2 42.2 9 Ynti = Tn — Yar ~ 2 Tir (49) va (50) ta suy ra ast — Sung ta = TCR = Sua +) — (2h — Bang phuong ph4p quy nap suy ra voi moi n=1,2,... a2 oy +4=0 Cach 2 Ta chimg minh bing quy nap ring Ban + 5Yn In. = — Bq +34 Unt = = ay, voi n = 1,2 cOng thie (51) din; kya — Tee. Thay bidu did Gia sir (51) ding chon = k,n = Tk+2)Tr+1 theo (51) vao ta thu d Bap42 + Suk +2 2 5 )+ 5 Buen — oe) = Ching minh wong ty Yes = Tir dé 82 2) Nguoe lai gia sir (up, vp) La mot nghiém bat ky cha (48). Ta phai ching minh rang tén tai k sao cho (uo, vo) = (we; Ye) Néu up = 1 4 v9 = 1. Vay (uo, v0) = (1,41). Gid sit up > 119 > 1 Sug —5vy , _ 3v0 ~ uo 2 2 w= 21 Vi ug — Sug = —4 nén uo, v9 cing tinh chan lé, Vay uj, v1 nguyen. Hon nitau; > 0,u1 > 0. That vay uy > 0 < 3up > Sup & Gus > 2g & 9(5ug — 4) > Wg & 20u3 > 36. Ding vi vp > 1. Ta c6 v1 > 0. 3u9 > up 99 > ug & 98 > Sug—4 Ding. Ta cing kiém tra dé dang ring u? — 5v? = —4 va uy < ug va 3uy +50 up = 3 (33) . m= — (34) Néu uw > 1 + vy > 1 thi ta lai xay dung duge day (ug, v2) IA nghiém cha phuong trinh(48) v6i uz < wy. Qué trinh nay phdi ket thiic & bude thi k va tw c6 (ux, ve) = (1,1) = (#0, yo) Vi Un + 3Up rs a nen (ui, Ue-1) = (#1441) > (e—2, U-2) = (2,42. (wo, U0) = (eve) Tom lai tat cd cdc nghiém cha (48) I (1; 1)(4; 2)(11; 5)(29; 13)(76; 34)... Dinh ly 7. Gia sir phutong tinh (IID) 6 nghiém. Néu (29, yo) 1A nghiém nhd nhat cua né thi 2 », —na ue < max {ne a } (35) Chitng minh X61 s6 u = axa — dbyo.v = ayo ~ bao . D8 kiém tra u? — dv? = n. Gid sit tri lai yo > nb?,y > 0 That vay v > 0 ayo > bry = Py} > ba} = (1+ db?)yy > Vay & yf > O(a} — dw) = nb, Bat ding thée duing.u > 0 = a2a2 > db2y2 = 23(1 + db?) > d?b?y3 = af > —db?(23 — dyg) = —ndb? <> dy§ > —n(db? +1) & dy > —na? Bat ding thie ding. Vay (u,v) la nghiém cia (111). Mat khéc tir hé phuong trinh bac nhat axxo — dbyo rot ayo = v( dn [a xp. yo ) gidi ra ta duoc a = au +dbv. yo = bu tar — mp > u. yo > v. Digu nay mau thu’n v6i viée chon (9, yo) 18 nghiém nbd nhat. Dinh ly 8 Gid sit phutong trinh (Il) c6 nghi¢m. Gia sit (a1, 4)... A cde nghiém cia (IM) théa man bat dang thite Om, Bm) Va 2 2 4,2, ~na v} < max fna?: Sp i=12.m (56) t d Xm day sau day: day thit i G=1,2...m) (14.4. Yna) Xe dinh boi 29,4 =Or: Yo = 4 Tns1s = nga + dyn id Untia = Trad + Yn se Khi d6 céc day (2p, Ya) S€ vét hét nghiem cba (IT). Chitng minh Ta da ching minh cic sO hang cia day tren [A nghiém cla Neuuoc lai gia sit. Ngugc lai gid sir (up, vp) A mot nghiém bét ky cia (48). T. ching minh rang tén tai #5,% sao cho (uo, v9) = (k,is Yk.) Néu v9 < max {nb?; =} thi t6n tai i dé (19, v0) = (24, 81) = (2204, YO.) ta.c6 k = 0. Gid sit trai lai. Két s6 wy = aug — dbvo, v1 = aug — bug. Dé kie u} — du} = n. Ngoai ra vi up > nb?, up > =8% nen ly luan nhu trong ching dinh ly 7 ta 6 w > 0,01 > 0 do d6 (uy, 01) A nghiém cia (IM) va uu = ua + duyb v= wb+ 110 Tir d6 vy < vp. Néu vy > nb?,v, > =B% thi ta lai xay duwmmg due day (ua, nghiém cla phuong tinh(48) véi vz < v1. Qué trinh nay phai két thic & bude vGi vg < max {nv?; = \. Khi d6 tn tai i dé (ux, ve) = (aa, 81) = (20,4, Yo; Una = Atty + dnb Un—1 = Unb + Una (tk—1, Ve=1) = (@1,i, Yr.i) > (Ue—-2, VE-2) = (2,4, 92,5 J > (uo, V0) = (2K, Dinh ly duge chimg minh Vi du Ching minh ring phuong trinh 2? — 34y? = Gidi Phuong tinh 2? — 34y? = 1 c6 nghi¢m (a,b) c6 nghi¢m thi nghiém nhé nhét (29, yo) thoa man y3 < GX = 36,029 + yo Thir cdc s6 1,2,3,4,5,6 ta déu khong tim duge nghiém. Vi du 8. Tré lai phuong trinh 2? — 5y? = —4. Phuong winh 2? — 5y? = nghiém (a,b) = (9;4)C4c nghi¢m (a; ) théa man 6? < 484 = 64,8 + 6 < (1, 1)(4;2)(11, 8). Vay ta e6 ba day sau 0,1 = 1,401 = 1, nti = 92,1 + 20yna, Yntia = 4en1 + Ina 0,2 = 4, Y0,2 = 2, 2n41,2 = 92,2 + 20Yn,2, Ynti,2 = 4tn,2 + 9n,2 G03 = 11,403 = 5, %n41,3 = 9n,3 + 20Yn,3, Ynt13 = 42n,3 + na Ba day nay vét hét nghiém cia phuong trinh da cho . Sau day ta sé bign luan sy c6 nghiém cla mot s6 phuong trinh Pell c6 tham I. Bién luan phuong trinh v—we=an Bai todn dat ra 1a: Tim diéu kién cn va di ma n phai théa man dé phuong trin c6 nghi¢m nguyén duong. Ky hiéu D 1a tap hop céc s6 n nhu vay. Dinh ly 9 Gid sit n = p 1a s6 nguyen t6. Khi d6 p € D khi va chi khi hoac p = +1 (mod 8) Chitng minh Gia sit phuong trinh c6 nghiém (2,y) va p = £3 (mod 8). chia hét cho p thi y chia hét cho p. VE trai chia hét cho p? . Vo ly. Vay 2? (mod p) + a?~1 = 2@-Y/2yp-1 _, 2(P-1)/2 = 1 (mod p) Mau thudin v6i két « biet. 84 Dio lai: Neu p=2 thi (57) 06 nghiem (25 1).Xét p = -t1 (mod 8). Khi d6 t6n ti a i N théa min a? = 2 (mod p),-Xét tap {x + ay} trong d6 x =91, y ,m voi m. = Tel Tap nay 06 (m+ 1)? s6 va vip < (m+ iu nén tn lai RE # (a2, 92) dé g1 Fay = 22 +042 aS = (er — 22) = aly — 3) (mod p) + (a ~ are Oe Sti)? God p). Date = |e. - aly = |v — wl . Tac6 2? < m? a? > a? — 2y? > —2y? > —2p, 3 (w+ 2y)? -Ac+y)P =p. Datta=rt+%barctyta 6a, 5) 1 nghiém cia (57). BO dé 1 N&ineD,meD thinme D. Chitng minh Gid sit (a? — 2b?) = n, ( — 2d?) = m. Khi d6 nm = (a? — 2b*)(c? — 2d”) = (ac + 2bd) — 2(bc + ad)? Didu nay ching minh nm € D. B6 dé 2 Gia sir q 1A sO nguyen 6 q = £3 (mod 8). Néu x? — 2y? chia hét cho q thi x,y déu phai chia hét cho g va do d6 x? — 2y? chia hét cho q? Ching minh Phan ching. Gia sit tr4i lai. Khi d6 2, y déu khong chia hét cho p. Ta c6 2y? (mod p) 3 2P7} = QP-D/2yP-1 _, 20P-10/2 = 1 (mod p). Mau thudn v6i két qua da biét ; Néu p = £3 (mod 8) thi 2°-1)/2 = -1 (mod p). Dinh ly 10. Gia sit n c6 phan tich tiéu chudn n= 2" Tp} Tg 6 d6 ¢ = £1, pj 1A céc s6 nguyen 16 dang 8k +1, q 1 céc s6 nguyen 16 dang 8k 3 Khi 46 n € D néu va chi néu t; la sO chan véi moi i. Chitng minh Gia sit t; 1A sO chin v6i moi i, Dat m = e2"TIpy. Ta cd € € Dr2@QE Divi € D do dé theo b6 dé 1m € D. Vay tén tai x,y nguyen duong sao cho x? — 2y? = m.Vi t; la s6 chan véi moi ¢; nén Ugi = h?. Thanh thir n= mh? = (wh)? — 2(yh)?. Ching 16 n € D. Dio lai gia sin € D tie 1a tén tai w,y sao cho 2? — 2y? = n. Gid sit g 18 use nguyén t6 clan, q = 8k +3 vag c6 s6 mii 6 B96 18. Kki d6 2? — 2y? = q°b voi (bq) = 1. Theo b6 dé 2 x = gri,y = gy > 23 — 2y? = q°?b néus > 2. Sau mot sO hitu han bude ta din dén 2? — 2y? = qb, Theo bé dé 2, ta c6 mau thudn. Il, Bién ludn phvong trinh oe -3y =n (58) Baj todn dat ra 1 ; Tim diéu kign cn va di ma n € Z phdi théa mn dé phuong trinh (58) 06 nghiém nguyén duong. Ky higu C 1a t4p hop cdc sO n nhu vay. Dinh ly 11. Gia sit n khong chia hét cho 3. Khi d6 n € C néu va chi néu a) 1 (mod 3) b) Trong phan tich tiéu chudn cita |n| = MIp#IIgi* voi pj = +1 (mod 12), qi = +5 (mod 12) thi t; [a s6 chin véi moi i. Chitng minh Diéu kién cAn: a)Gia sir n € C. Tén tai x,y nguyen duong sao cho —3y? =n «2? =n (mod 3). Vay n=1 (mod3). n byTa c6 bd dé sau Bé dé 3: Néu q 2 s6 nguyen t6 , g = +5 (mod 12) va x? -3y?=0 (2 thi x, y déu chia hét cho q va do dé 2? — 3y? chia hét cho q” That vay Gia sit trai lai. Khi d6 2, y déu khong chia hét cho p. Ta c6 az? = 3y? (mod p) + a?7! = 30-D/2yp-1 _, 9-0/2 = 1 (mod p) -Mau thudn v6i két qua da biét : Néu p = £5 (mod 8) thi 3-1/2 B6 dé 3 duge chimg minh. Gia sit q = qj A ube nguyen chan, q = 12k+5 Vag = qj C6 s6 ma la sO lé. Kki d6 x? — 3y? = qb v6i (b,q) = 1. Theo b6 dé 3 x = gri,y =: x} ~ By? = q°-?b néus > 2. Sau mot sO hi han bute ta din dén 2? — 3y2 Theo bé dé 3, ta c6 mau thudn. Diéu kién di: Gia sir cdc diéu kien a) va b) duge thue hién. Theo b) n = trong d6 hodc |h| = 1 hoac|h| = Mp; & d6 p; = £1 (mod 12). Néu |hj = 1: Vin =1 (mod 3) 3 Ah = 1 4 n = m? phuong trinh c6 n (2m,m). Xéth # +1. Vin = 1 (mod3) + A = 1 (mod 3) ( do mé (mod 3)). V6i mdi p; tn tai a; sao cho a? = 3 (mod p,) = +1 (mod 12) 1 a; sao cho a? = 3 (mod p;). Theo dinh ly Trung hoa tén tai a sao cho a (mod p;) voi moi i. Ti d6 a? = 3 (mod p;) Vi © a? = 3 (mod h). X {x + ay} trong d6 x = 1,2,...,u;y = 1,2,...,u voi u = [Wh]. Tap nay 06 (u sO va vik < (u+1)? nén tén tai (1,41) # (w2,y2) dé oy + ay = 22 (mod h) + (21 —22) = a(yz—y1) (mod h) -+ (2, — 22)? = a?(y2—y.)? (me Dat x = |zy-—a2|,y = |n—yl. Por 2 ta cd x Ba} — y? = 31m + 2? — By? = 32m. Sau mot s6 hitu han bude ta di dén a? ~ 30? = w2 — 3y2 =m 86 Theo dinh ly 11 ta c6 m théa man diéu kién a) va b) Gia sir & 18. Twong tu nhu trén ta c6 ¢ = 321 > 322 —y? = 3'-1m > 2? —3y} = 3*-2m, Sau mot s6 hitu han buéc ta di dén 2? — 3y2 = 3m > a, = 3u > 3u? —y2 =m y? — 322 = m. Ta c6 —m phai théa man diéu kién a) va b) cla dinh ly 11 te Ia m théa man diéu kign a) va b) cba dinh ly 12 Diéu kien di:Gia sit n théa min céc diéu kién a) b) + Néu k chin, k = 2t . Theo dinh ly 11 t6n tai x,y sao chp 2? — 3y? = m4 (3¢z)? — 3(3'y)? = 3%m =n. Vay nec. Néu k 18 k = 2¢41. Theo dinh ly 11 t6n tai x,y sao cho 2? — 3y? = —m + 8y? — 2? = m > (3¢ty)? — 3(3¢a)? = 341m = n. Vay n € C. Dinh ly 12 duge ching minh. Bai toan téng quat: Gidi va bién luan phuong tinh 2? —py=n V6i p IA sO nguyen t6 Ta da bién luan suv c6 nghiém cia phuong trinh trén v6i p = 2 va p = 3. Trudng hop p 1a s6 nguyen t6 bat k theo ché ching t0i biét con dang bd ngd. IV. Phwong trinh Pell téng quat Phuong trinh Pell téng quat 1a phuong trinh Av? -By=n (IV) Trong trumg hop n = 1 ta c6 két qua sau day Dinh ly 13. Cho phuong tinh Aq? — By =1 (59) v6i AB khong chinh phuong va A > 1 Goi (a, b) 1A nghiém nhé nhit cia phuong tinh Pell két hop a? ~ ABy? = 1. Xét he a =Az? + By? b =2ny a) Néu hé trén c6 nghiém thi né c6 nghiém duy nhét. Goi (zo, yo) 1 nghiém duy nhat 46. Néu Axg — By} # —1 thi phuong trinh (59) c6é nghiém va (zo, yo) chinh 12 nghiém nhé nhét cia n6. b) Dao lai néu phuong trinh (59) c6 nghiém va (20, yo) 1& nghiém nhé nhét cia né thi (0, yo) 1&4 nghiém duy nhat cha hé tren. c) Gid sir phuong trinh (59) c6 nghiém va (zo, yo) 1& nghiém nhé nhét cla n6. V6i moi s6 tu nhién 7 t6n tai duy nhat (2m, Yn) thea (toVA + yoW BY??? = ay VA + yn VB Hon nita day (an; yn) vét hét nghiém cia (59) Doc gid ty chtmg minh, bing phuong phép tuong ty nhy chtmg minh dinh |: Chi §: Ta c6 thé mo ta nghi¢m duéi dang day truy héi nhu sau Ta cé (oVA — yoVB)"*! = an VA — ynVB Suy ra (zoVA + yov'B)**? + (toVA ~ yo B)?"*! ty = OVE SVE ov eve 2VA tn = Ci( Aa} + By} + 2aoyoVAB)" + Co( Aa} + Byg — 2xoyoVAB)” = Ci(a + bVAB)" + C2(a — bVAB)" Tir dé (vq) [A day truy héi caip 2 @n42 = 2aT_41 — Tn v6i cc gid tri ban déu zo, 21 = Az} + 3Baoy2. Tuong ti (yn) [a day truy héi ep 2 Ynt2 = 24Yn41 — Ym v6i céc gi tri ban ddu yo, y1 = By + 3Ayo23. Vi du Giai phuong trinh 32? — 2y? = 1 Phuong trinh Pell két hop x? —6y? = 1 c6 nghiém nhd nhat (a,b) = (5, 2) Ph tinh dang xét c6 nghiém nhd nhat (20, yo) = (1,1) Vay tap nghiém (tn, yn phuong trinh duge cho béi cong thtéc Tn42 = 10tn41 — In v6i cdc gid tri ban dau zp = 1,2, = Ax} + 3Bxpy3 =34+6=9 va Ynt2 = 10Yn+1 — Yn v6i céc gid tri ban d4u yo = 1,y1 = By + 3Ayox} =24+9=11. Dinh ly 14. Phuong trinh (IV) hode vo nghiém hode c6 vo s6 nghiém. Chiing minh. Gia sit (IV) c6 nghiém (an, yn) tite 1a At — BR =n Goi (a, b) 1A nghiém cia phuong trinh Pell két hop x? — ABy? = 1. Khi dé tac a ~ ABP =4 Nhan (40) véi (41) ta duge (Av? — By?)(a? — ABO) =n 4 A(tna + Bynd)? — B(Atnb + yna)? =n Dat Tn41 = Inat Bynd Uns = Atnd +40 88 Ta thay (2n41,Un41) IA nghiém va ty < tn41,¥n < Umer. Vay cOng thite (42) va (43) cho ta vO sO nghiém cia (IV) néu (IV) c6 mot nghiém khéi déu (ay, 41). Bang phuong ph4p tuong ty nhu dinh ly 7 ta c6 dinh ly sau Dinh ly 15. Gia sit phuong trinh (IV) c6 nghiém. Gid sit (a1, 91), -..; (am; Bm) 1a tft cA c&c nghiém cha (IV) théa man bat ding thite =na? BB max {ano =} i=1,2,...m (4) Xét m day sau day: day tht i (=1,2,...m) (2tn,,Un,i) x4c dinh boi Boi =O, Yo, = Bi Bayi = Tria t+ Byngd Yatii = And + Unie Khi d6 c4c s6 hang (:¢m, yn) ca t&t cd cdc day tren sé vét hét nghiém cia (IV). Chai ¥. C6 thé m6 td du6i dang day truy héi cp 2 nhu sau: Dé cho gon, 6 dinh i dat p= Eni, Yn = Yn. ta 06 Zo = Q4, 2) = 20; + BbG;, yo = Bi, m1 = a8; + Aboy va Tn42 = In410 + Byn gid Bynyib = ABB’ a, + Bbayn In42 = Fn410 + ABYy + a(tn41 — Ta) = = 2atn41 — (a? — ABY) tn = 20%n41 —2n ‘Vi du Gidi phuong trinh 2? — 2y? = 7. Phuong tinh 2? — 2y? = 1 c6 nghiém nhé nhat (a,b) = (3,2).Ta 06 6? < 7.2? = 28. Vay & € {1,2,3, 4, 5}, Ta thay c6 hai nghiém théa man dé Ia (3; 1); (5;3). Vay tt cA cdc nghiém cia phuong trinh dugc m6 ta bdi hai day (¢n,1,Yn,1) Va (an,2,Yn,2) xéc dinh bai cOng thite sau 20,1 =3, yo,1 = 1 {ee = 32n,1 +441 Yasir = 2tn1 + 3yn1 va 2,2 =5,yo2 = 3 {ee = 3002 + 4n,2 Ynti2 = 2ztn2+3yn2 Day dau cho cc nghi¢m (3; 1)(13;9)(75, 53), ... .Day thtt hai cho céc nghiém (5,3) (27,19) (157, 111),.... Cling ¢6 thé mo t& hai day tren bing day truy h6i cp 2 nhu sau . = to = 5,21 =27 any2 = 62n41 41 = 19 yn42 = bYns1 — Yn 3,01 = 13 tpg? = Otny1 — tn 2M =9 Yn42 = 6Yn41 — Yn Zn LIEN PHAN SO VA UNG DUNG Dang Hang Thing 1. Lién phan sé hitu han Biéu thitc c6 dang ag + ———— a+ ag tot ; ay + — an 1 trong d6 a9, a1, ...,4m IA c&c 86 thue a1,...,4n # 0 duge ky higu 1A [a9; a3, ...,¢ dinh nghia dé thay : : 1 {205 a1, ..-) x41] = a0 + —————_—— far} .a2; ..., 241] Néu ap € Z va a1,...,@n 1A céc sO nguyen duong thi ta néi (a9; a1, ..., an] 1A me phan s6 hitu han 6 do dai n. R6 rang mot litn phan s6 hitu han la mot s6 h Nguoc lai ta c6 Dinh ly 1.1 Méi s6 hia th c6 thé bigu dién duéi dang mot lién phan s6 hit Chitng minh Gia sit x = a/b trong d6 a,b € Z va b > 0. Dat ro = a,r- Thuat chia O co lit cho ta oe OC3>C5>-+- Co Cy. Ta lai c6 (cing theo dinh ty trén) (-1)2"-1 Gam GQ2m—1 Vay Caj-1 > C2j—1421 > C2j+2i > Cri Com — Com—1 = <0 Cam < Com-1 Il. Lién phan sé v6 han Dinh ly 2.1 Cho ap, ay, a2,... lA diy vo han céc s6 nguyén véi a; > 0, Dat Oe = 205 1 oy OR) 92 Khi d6 t6n tai giuéi han lim Qj =a k-400 Ta goi a IA gid wi cha lien phan sO vo han [ap; a1, a2..,] va viet = [a9; 01, a2,...] Chitng minh Theo dinh ly trén ta c6 C1 > C3 > Cs > +++ > Con-1 > Congr > 0° Co < C2 < Cy < ++ < Can-2 < Con < + Hon nita day (Cop41) 1 day gidm va bi chan duéi b3i Co con day (Cox) tang va bi chan trén boi C, . Vay t6n tai lim Co41 =, lim Cy, = a2 kos00 k-400 Ta cdn chimg minh oy = a2, That vay theo dinh ly tren 1 q2k+1 92k Cox — Cre = Dé thay (bang quy nap) q, > k. Do d6 im Corp ~ Cor = 0 a1 = 22 Dinh ly duge chimg minh. Dinh ly 2.2 a = (ag; a1, a2, ...] IA mot s6 vo ty, Chitng minh Phan ching : Gid sit tri lai a = a/b € Q.Theo dinh ly tren Con 1. Nhu vay tat cd cdc $6 ay, a2,... déu 1a s6 nguyén d Dé kiém tra duge = [a9; 01, 42, .., 4, e+] Tu dé Ok+1Pk + Pkt (erg + G41) _ ~(Pides1 — Pe-19%) © (Qk419 + 98-441) = (ay? © (C419 + G41) I a-k= Vi 419k + Qb41)% > O19 + G1 = Geer SUY Ta la-Ch| < Wek +L Vay a = [a9; a1, @2,...]- Tigp theo ta ching minh bigu dién IA duy nhdt. Gid sit a = [a9; a1, a2, (t0; bi, b2,...]. Vi Co = ao, C = ap + 1/a) va gidn phan 1é Ién hon moi gin chan nén ag --] Suy ra [ax42, 0443, --] = [be-+2, 443, --]- Do d6 bang quy nap suy ra az = be Vi du 2.1 Biéu dién V6 thanh lien phan sO vo han, Ta 06 V6 +2 ond _ L = V6+2 1={ 7 ] ers 6+2 1 vO+2_ 5 = Qy aa 2 94 Vay a3 = a1 do d6 a3 = a1, a4 = ag,..., Vay V6 = [2;2,4,2,4,2.4, «J. Cha y: Tins ly thuyét lign phan s6 ta tim Iai mot ching minh khéc cia bé dé 1 ( Xem chuyén dé 2 ; Phuong trinh Pell). Cho @ 1a mot s6 v6 ty. Khi dé c6 t6n tai vo s6 cap s6 nguyén duong(h, m) thoa man Aa mm ‘That vay Theo ( 66) ta c6 Ja — Ck] = Ja — pie/ge| < 1/gege41- Vi ge < Gk41 nen suy ra L la = pe/ge| < % Vay c6 v6 s6 cap sO nguyen duong (h,m) ma ja — h/k| < 1/m? IIL. Lién phan sé v6 han tudn hoan Ta goi lin phan s6 vO han [a9; a1, a2, ...] A tan hoan néu day (an) 1A tuén hodn ké 1 mot chi sO no dé utc Ia: tn tai s6 nguyen duuong m va k véi moi n > mta 66 On = On¢e. SO nguyen duong k duge goi 1d chu ky. Trong trudng hop dé ta viet [03 01, 425 ..+5 @m—1; Gon Gms Gm tka] Bai toén dat ra Ja dac trung tat cA cdc s6 vo ty cd biéu dién lién phan s6 vo han tudn hoan. Ta c6 khéi niém sau Dinh nghia $6 v6 t¥ @ goi 18 s6 vo ty bac hai nu n6 1A nghiém cda mot tam ‘thitc bac hai véi hé s6 nguyen. Vi du 3.1 S6 vo ty a = 24 V3 IA 86 vo ty bac hai vi no 1a nghiém cia 2?—d4do+1=0 Bé dé 3.1 S6 thuc « néu va chi néu c6 tén tai cdc sO nguyén a,b,c voi b> Ova khong chinh phuong,c # 0 sao cho _atve c a Chitng minh Gia sit a 1 s6 vo 19 bac hai. Khi d6 t6n tai céc s6 nguyen A, B,C sao cho a 1a nghiém cla phuong trinh Ax? + Br +C =0. Vay -B+ VB? —AC 2A Dat a = —B,b = B? — 4AC,c = 2A hoac a = B,b = B? — 4AC,c = —2A Nguoc lai néu at+vo e thi a [A s6 vO ty va né 1a nghiém cia phuong tinh bac hai c?x? — 2acr +a? —b = 0 B6 dé 3.2 Néu a [A 6 vo ty bac hai thi (ra + s)/(ta + u) cling 1a s6 vo ty bac hai néu r, s,¢,u 1a cdc sO nguyen. Chiing minh Gia sit Tinh ton cho ta rats _ (ar+cs)(at+ cu) —rtb+ (r(at + cu) — t(ar +cs)) Vb tatu (ai Fou)? — Dinh nghia S vo ty duoc goi 1a lién hop cia a va ky hiéu 1a a! Bé dé 3.3 Néu sé v6 ty bac hai a 1a nghiém cia phuong trinh Ax? + Ba + thi lin hop cia né cing 18 aghiém cia phutong trinh dé. That vay aya" = 2a/c = —B/A, (a)(a') = a? — b/c? = C/A Bang phép tinh ta dé thay Bé dé 3.4 Ta cé céc hé thiée sau (at By o's oh (08y = o's" ()-5 Ta c6 dinh ly co ban sau day do Lagrange tim ra Dinh ly 3.1 S6 vo ty a c6 bigu dién lien phan sO tun hoan khi va chi khi 86 vo tyu bac hai Chiing minh Trude hét ta chimg minh ring néu a ¢6 biéu dién lien phan s hon thi n6 a sO v6 ty bac hai. Gia sit Gel en 08 | Omori oace) Dat B= (Om; Omi; = Gal Khi d6 8 = am, am415 0 @m+e-f3] do 46 + Phen 7 ee o trong d6 pe/ de Va pe—1/Ge—1 1A hai gidin phan cudi cita [aym, dmn:1) +4 dmsx) TY thie (1) suy ra 8? + (qn-1 — Pk)B — Pk-1 = 0 Vay 3 1 s6 vO ty bac hai. Ta lai c6 a = [a9; a1, a2, ..., @m—1, 3] do a6 BBmn—1 + Pm—2 BQm—-1 + Im-2 do dé theo bé dé ta c6 a [A 86 vO ty bac hai. Vi du sau day minh hoa céch tim sO vo ty bac hai t biéu dién lin phan s hoin cia né. 96 Vi dy 3.1 Tim a biét ring # = [3,3]. Ta c6 x = [3;y] voi y = [1,3]. Ta 06 y= [1;2,y} do d6 1 3y+1 =1 evi nr s SEs Suy ra 2y? — 2y—1=0. Viy>Onen y= (14 V3)/2. Vic =341/y nenta tim 2 _44+v2 duge ea3t 1+ V2 2 Dé chting minh phan nguge lai ta céin bé dé sau BO dé 3.5 Néu a 1A s6 vo ty bac hai thi n6 06 thé biéu dién dudi dang auPtvd Q trong 46 P,Q, dla cdc s6 nguyén sao cho Q|(d — P?). Chitng minh Ta c6 a = (a+ Vb)/c. Nhan cd tit va mau véi |e| ta duge a = (alel+ Vibe?) /cle|. Dat P = alc|,d = be?, Q = cle| = +c”. Khi dé d—P? = c?(b~a?) .) nhu sau (Po + Vd)/Qo. chia hét Q = +c. Gia stt @ = ap 1a sO vo ty bac hai. Ta xay duumg day (ao, a7 Theo bé dé trén ta c6 céc sO nguyén Po, Qo va d sao cho ag Qo|(d— P3). Ta dat ap = [ag] va xéc dinh P, = a9Qo — Fo, Q1 = (d= P3)/Qo, a1 = (P, + Va)/Q1 . Tip d6 dat a; = [ay]. Mot cdch téng quét néu cd Pr €Z, Qe € Z,Qx\(d- Pz) P, _ arva ax = [ax] Ta'sé dat Pas = Qi — Pes Qevr = (d- PR )/Qes Pra + va; One = Fant V0) On41 = [On4] Qe Khi d6 tinh todn cho thay Qnia = (A= PR)/Qi + (2a,Ph — aA) do 45 Quer €Z vA Vi QerQe = (4~ PR, ;) nen Qesa(d— PZ,,)- C6 thé ching minh duoc rang a = [ag; a1, 42,...] va hon nila day (aj) x4c dinh nhy trén 1a tudn hoan. Vi du 3.2 Khai trién lien phan s6 cla s6 a = (6 + V8)/4 —8, ao = (6+ 28) /4, a9 = [ao] = 2 Tacd Po = 6, Qo = 4,d = 28, 4|(28-67) va Py = 24-6 = 2,Q) = (28 = 2”) /4 = 6, a1 = (2 + V28)/6, a1 = [ay] =1 Pp = 1.6 —2-—4 = 4, Qo = (28 — 4”)/6 = 2,09 = (4 + V28)/2, a2 = [a9] = Py = 4.2—4 = 4,Q3 = (28 — 4”)/2 = 6,03 = (4 + V28)/6, a3 = [ag] = 1 Py = 1.6 —4 = 2,Q4 = (28 — 2")/6 = 4,04 = (2 + V28)/4, a4 = [ay] = 1 Ps = 1.4 —2=2,Qs5 = (28 — 27)/6 = 4,05 = (2 + V28)/4, a4 = [ay] = 1 Ta thy Pi = Ps, Qi = Qs do d6 a; = as va day tudn hoan chu ky 4. Ta c¢ _ =205000 Ti€p theo ta mun tim diéu kien dé sO vo ty bac hai e6 biéu dién Lien phan s« hoan ngay ti du , utc 1a diéu kién dé t6n tai s6 nguyen duong k sao cho a, = v6i moi n > 0. Ta cé dinh ly sau Dinh ly 3.3 SO vo ty bac hai a c6 biéu dién twin hon ngay tir dau néu néua>lvai-l Lva-1 0 théa man |sa—r| < |gx0 ~ pa] this 2 gk41- Chitng minh Gia sit trai lai 1 < 8 < qeya- Xét he phuong trinh PRE + Peoiy =P KE + Qh+1y = 8 Suy ra (Pi1dk — Pk Qk+1)Y = 74k ~ SPR Vi pe4idk — PeQe41 = (—1)* nén y= (-1)*(r4x — spr) Tuong tu ta cé © = (-1)*(speva — rge41) Ta nhan xét rang 2 4 0,y # 0. That vay néu c = Ohi spear = roxy. Vi (peti, qk4i) = 1 nén geyi]s—+ s > ge41 tri gid thiét. Néu y = 0 thir = pya,s = que do 46 |sa-—r |lqeo — pil > geo ~ pel Mau thuan. Tiép theo ta chttng minh zy < 0. That vay y <0 gx =s—qyiy>032> 0. Néu y > Othi Vi gery > Gear > $ acd Ge =s —Geiry<032<0. Mat khéc ta lu6n c6 py/gk < @ < pesi/ge+1 hode Pryi/de+1 O nen o(qeoe — pe) Va y(ge+12 — Pe+1) 66 cing dtu vay |soa —r| = |x||qxa — pe] + |yllae41@ — Peoal 2 |allqx0 — pel 2 laa ~ pel Diéu nay mau thudn v6i gid thié.B6 dé duoc chimg minh. Dinh ly 4.1 Trong s6 cdc s6 hitu ty r/s xp xi s6 VO ty a voi mau s6s 0 thi phuong trinh (68) cé mot nghiém 13 z= (-1)lgn-asy = (-1)"Pra Néu b < 0 thi phuong trinh (68) cé mot nghiém 1a 2 (-1)" Gnas = (1)! Vi du 5.1 Gidi phuong trinh 3420 — 123y = 15 Gidi Vi (342, 123) = 5 nén phuong tinh da cho tong duong voi L142 — 41 Ta biéu dién sO 114/41 thanh lién phan sé 100 Ta cé 114 = 2.41 + 32 41 =1.32+9 32=3.9+5 9=1544 5=1441 4=41 Do vay 62 oe = 2; 1,4] 3 = 213,114) Ta c6 n = 5,pa/g4 = [2,1,3,1,1] = 25/9. Vi b = —41 < 0 nén mot nghiém cia phuong trinh 114¢ — 41y = 1 1A ¢ = q = 9,y = 25. Suy ra mot nghiém cia phutong tinh 114% — 4ly = 5 A 2 = 5.9 = 45,y = 5(25) = 125. Nghiém téng quat cla phuong trinh d& cho [a x =45+41t y = 125+4114t voit eZ. b) Phuong trinh 2? — dy? = +1 B6 dé 5.1 Cho d a s6 khong chinh phuong. Gia sit Pe, Qu, ax, ax Ia cdc sO xéc dinh trong vigc tim Khai trién lien phan s6 cba Vd ( xem bé dé 3.5 ). oy = AAV) 9 tay] Qn Prat = xQk— Pr» Quer = (d— Phs1)/Qe, Ok = eae ax41 = [ary] + Gia sit py /gy 1A gidn phan thit k cla Vd. Khi d6 Pk ~ adh Chitng minh Vi Vd = ag = (a0; 41, 22.55; 4-441] nen theo dinh ly ta c6 11k Va = ay = Sethe + Pho OR + k—-1 Vi on41 = (Prgt + VE)/Qu4r ta 06 Va — Petit Va)pe + Qe 1Pk-1 (Pera + Veer + Qin %-1 7g = (Pride + Qu si%—1)Vd = (Pe4iPe + QesrPe1) + PVE Tur dé suy ra ( dovd ¢ Q) ng = PeviPe + QeriPe—1s Peridk + Qerie-1 = Pre Tir d6 ( nhan phuong trinh ddu véi q,, phuong tinh tht hai véi pz, rdi rir cho ta duoc Ph — gg = (Peae—1 — Pk-19k)Qe+1 = (—1) Qasr Dinh ly 5.1 Gid sit chu ky cia biéu dién lien phan sé cia V/d a r. Goi pp, gidn phan thir k chaV/d. Néu r chan thi x = pia, y = er—1, (t = 1,2,...,) any cha phuong trinh Pell x? — dy? = 1. Néurlé thi 2 = porpi,y = Qoer—1, (t= 1, 1& nghiém cia phuong trinh Pell 2? — dy? = 1. Chiing minh Vi Jd = 0+ Vd/1 nén Qo = 1 Quer =Qo =1 Vk. Th dé 1 ta c6 Pir—1 ~ bya = (—1)" Qin = (-1)" ‘Thanh thir néu r chan thi pj,-;—dqp,_) = 1 vk € Nnéurlé thi thi p3,,_,~dq3, 1,(t=1,2,...)4 B6 dé 5.2 Q; #1 voi m s2y... VA Qy = 1 khi va chi khi k chia hét c Chimg minh Gia sit tén tai i dé Q; = —1. Suy ra a; = ~P,- Vd. Vi biéu dién lien phan sO tun hoan ngay tir déu nén —1 < (ai)! = —P; + Vd < aj = —P,—Vd>1. Suy ra Vd Py Vi ox = (ax, a¢41,--] bi hoan ngay tu ddu nén —1 < (a,)' = P, - Vd <0 + Vd-1 < Py < Vd Tha [Vd] = Py = ag. Suy ra a, = P, + Vd = [Va + Vd = Bao, a1,..., a7] . Ta cé 493 41, «5 O41, OR] = [205 21, «.., @k—-1, 200, 1, Ar] = (40; 41, « boi cia chu ky r. Dé chimg minh day [a tt cd cde nghi¢m cia phuong trinh Pell ta cin cde sau B6 dé 5.3 Cho a IA mot s6 vo ty va r/s 1a sO hitu ty ti gidn voi r > Ov ja —r/s| <1/(2s?) Khi d6 r/s phai !& mot gidn phan cia a. Chiing minh Gia sit r/s khong Ia gidn phan khi d6 t6n tai k sao cho SS < Fett 102 Theo bé dé 1 ta cé lore — px| < [sa - | = sla —r/s| < 1/(2s) Suy ra lo — pr/ge| < 1/(2sqe) Vi ring [spe —rgx| > 1 nen ta 6 : a Vay 1/2sqx < 1/2s? + 2sqy > 2s? + gy > 5 trai véi (5). Bé dé 5.4 Gia sit x, y 1A cdc s6 nguyen dutong sao cho 2? — dy? = n va |n| < Vd. Khi d6 x/y 1a mot gian phan cia Va. Chitng minh Xét truudng hop n > 0 Ta c6 (« + yVd)(«@ — yVd) =n + @ > yd 4 0 < 2/y— Va. Lai c6 ®_gienvd uy a? — dy? © y(a+ ya) n oo y(2yvd) eavceeee QyrJd 2y? Theo bé dé 5.3 thi z/y 1a mot gidn phan cha Vd GiA sit n <0. Khi d6 ep —(1/d)a? = —n/d Ta c6 —n/d > 0,—|n|/d < 1/Vd Vay theo buée truée y/a 1a mot gidn phan cia 1/Vd. Nhung khi d6 a/y = 1/(y/2) 1& mot gin phan cha 1/(1/(Vd) = Vd Dinh ly 5.2 Cho phuong trinh Pell a? —dy*=1. Goi r 1a chu ky cia biéu dién lien phan s6 cla Vd . Néu r chin thi tt cd cde nghiém cia phuong trinh Pell 12 © = Per-1,Y = Qkr-1 . Néur Ié thi tét c& cdc nghiém cba phuong trinh Pell A. @ = parr—1,y = Qar-1, 1 € Ne Chitng minh Gid sit (x,y) 1A nghiém cia phuong trinh Pell. Theo bé dé 5.4 tai idé x = pi,y = qi. Tir dd Ph da? = 1. Tur bd dé 5.1 rit ra (—1)* Qin) = 1 > Qigd = £1. Vi Qua ¥ 1 nen Qian va i lé. Theo bé dé 5.2 ta nit ra tén tai ki + 1 = kr +i = kr —1 va kr chan. TI thine néu r 1é thi k chain, k=2t. Xét phuong trinh 2? — dy? =-1 (5.1) Ta c6 két qua sau. Dinh ly 5.3 Phuong trinh 2? — dy? = —1 c6 nghiém khi va chi khi chu cia biéu dién Lien phan sO cha V/d 1a sO 18. Trong trudng bop ay cdc nghiem od 1A @ = pear-r—1) ¥ = U2tr—r—1) VO t=1,2,... Ching minh: Tit bé dé 5.1 dé thay néu chu ky r cia biéu dign lien phan s¢ Va 18 8618 thi & = pragrp—1ys¥ = 2er—r—1) VOit=1,2,... 1 nghiem. Gia sit (x,y) 1A nghiém cia phuong trinh (5.1). Theo bé dé 5.4 t6n tai D=piy=q. Tr d6 ; Pi — dg? = -1. ‘Te bd dé 1 rit ra (—1)? 4 Qiga = -1 > Qin = 41. Vi Qigs # —1 nen Qiyn va i chin. Theo bé dé 5.1 tn tai k € N sao choi +1 = kr +i=kr—1vak ‘Thanh thir néu r chn thi kr luon chdn do d6 phuuong trinh vo nghiém. Trong truvng hop r lé ly luan tuong tw nbuv trong truéng hop phuong trinh a? — dy? — 1 tit cd cdc nghiém phai cé dangx = per—1,y = der—1 trong d6 kr 1 ia khi k 18 hay 2 = p(aep—p1) ¥ = Qatr—r—1) VOI t=1,2, V.I Phan tich mét sé ra thiva sé Cho s6 nguyén duong n.Ta c6 nha sau : Néu tim duoc hai s6 nguyen duong théa man x? = y? (mod n) véi x — a-+y khong chia hét cho n. Khi d6 u = (x—y,n) vav = (+y,n) Ia céc ude k tdm thutng ( tic 1A khong bing 1 hodc n cian), That vay ta c6 (x — y)(@ + y) hét cho n. Vi x-y kh6ng chia hét cho n nén u # n, Neu u =1 suy ra e+ y chi cho n, trai gid thiét. Tuong tv # 1. « Gid sit pg, gk, Pe, Qe A c&e 86 06 duge khi tinh cdc gidn phan cla Vd. dinh ly ta c6 —1)*"Qk41 (mod n) Néu ta tim duge k lé va Qi+1 = s? 1a sO chinh phuong thi ta c6 thé ding h u=(p, —5,n),v = (pe + 5,7) 1a cdc ude cha n néu ching khéc 1 na chinh n. vay thuat todn nhu sau: - Trong day Q; voi k chan ta nhat ra céc sO chinh phuong. - Gid sut Qy = s?, k chain : xét cée sO py +5. Kiém tra xem 06 s6 nac hét cho n khong -Néu ching khong chia hét cho n thi ta ding thugt todn O co lit dé tim (pe + 8,7), v = (py — 8,7). Khi d6 u,v chinh 1a cfc thita s6 cha n Vi du 6.1 Phan tich 1047 ra thia sO. Ta c6 Qi = 13,Q2 = 49 = ©6 py = (-1)?Q2 (mod n), py = 129 Vay (129 - 7,1037) = (122,103 61, (129 + 7, 1037) = (136, 1037) = 17Ta c6 61.17=1037. Vi du 6.2 Phan tich ra thita s6 1000009. Ta c6 Q: = 9,Q2 = 445,¢ 873, Qq = 81 = 9" Tuy nhien ps + 9 = 2000009 +9 chia hét cho 1000009. Ta lz 104 tuc tim céc sO Q, chinh phuong ma k chin, Ta tim duge Qig = 16 = 42. Khi d6 pyz = 494881. va (494881 — 4, 1000009) = 293, (494881 + 4, 1000009) = 3413. Thanh thir 1000009 cé6 hai u6c phan biét 1a 293 va 3413. Ta cling thay (293)(3413)=1000009. MOT SO PHUONG TRINH DIOPHANT PHI TUYEN Dang Hing Thang I. Phuong trinh Pitago Phuong trinh Pitago 14 phuong trinh sau eyya2 Bo ba s6 nguyén duong (x,y, z) théa man (1) duge goi 1A mot bo ba Pitago. Ro ring néu (c,y, z) [A mot bo ba Pitago thi véi moi d € N*, (de, dy, dz) 1a mot bO ba Pitago. Vi thé ching ta chi cin tim céc bo ba Pitago (x,y, z) (w,y, 2) = 1. Mot bo ba Pitago nhu vay goi 1A mot bd ba Pitago nguyen thiy. BG dé 1 Néu (c,y, z) 1A mot bo ba Pitago nguyen thay thi 2, y, z doi mot ng) t6 cling nhau. Hon nita, z,y khong cing tinh chn lé va z Ie. Chitng minh Gia sit (x, y) > 1. Néu pla sO nguyen t6 v6i px, ply thi p2|a?+: 2? + plz. Diéu dé trai voi gid thigt. Vay (c,y) = 1. Tuong tr (x, z) = 1,(y,z) Vi (¢,y) = 1 nen «,y khong thé cing chin. Gia sit ching cing 18. Kh a? = y? = 1 (mod 4) + 2? =2 (mod 4). Diéu may khong xdy ra. Vay 2,y KI cing tinh chan lé. Do dé z Ié. Vi vai trd cia (@,y) binh ding nén khong gidm téng quét ta gid thiét y chéin Dinh ly 1 Bo ba (z,y, z) 1A mot bo ba Pitago nguyen thiy (véi y chan) né chi néu né cé dang c=m—r? y=2mn z=m int tong dé m,n Ja cde sO nguyén duong m > n, (m,n) = 1 va m,n khéc tinh cha Chitng minh Gia sit (x, y, z) 18 bO ba Pitago nguyén thity. Ta 06 y? = 2? —: (z +a)(z—2). Via,z lé va y chan nen z +a = 2,2 —a = 2t,y = 2h, Thay ta duyoc h? = It.Ta c6 z =1 + t,2 =1—t. Theo bé dé (x, z) = 1 (I, t) = 1. tén tai m,n sao cho 1 = m?,t = n? + h = mn. Vay 2,y,z 6 biéu dién da Hon nifa vi z 18 nén m,n khéc tinh chén 18. Néu 06 s6 nguyen t6 p véi p|m, pl p'|m?, p*|n? > pla, plz Mau thudn. Vay (m,n) = 1. Pio lai néu (x,y,z) cd dang trén thi dé kiém tra chiing 1a mot bd ba Pitago ching minh né 1A mot bd ba Pitago nguyen thiy ta chi cin chimg minh (2, z) ‘That vay gid sir 6 sO nguyen t6 p voi p|x,p|z. Suy ra pla + z = 2m?, plz — Vi z lé nén p Le. Vay plm?, p[n? — plm, pln > (m,n) > 1. Mau thuan . Vi dy Lay m = 2,n = 1 ta duge bd ba Pitago nguyén thiy nhd nhat (3. Lay m = 5,n = 2 Khi d6 theo cong thie trén ta thu duoc bd ba Pitago nguyen (21, 20, 29) 106 Il. Phuong trinh Feema . Ben Ié cia mot cuGn sch s6 hoc cla Diophant xudt ban yao nam 1637, ngudi ta da tim thay Fecma da viét nhu sau :” Phuong trinh a" +y" = 2" Khong c6 nghiém nguyén duong véi n > 3. Toi da tim duge mot céch chtmg minh tuyét di¢u diéu khang dinh nay nhumg vi lé séch qué nhd nén khong thé trinh bay duge”. Phuong trinh 2” +4" = 2" duge goi la phuong trinh Fecma. Khang dinh:” Phuong trinh <" + y" = z" khong c6é nghiém nguyén duong véi n > 3” duge goi 1a dinh ly lén Fecma. Dinh ly nay sau méi duge chimg minh d4y di nam 1995 béi Wiles. Ngudi ta khong tin Fecma da ching minh dugc dinh ly nay mot cach chinh xéc. Dinh ly 2 Phuong trinh aityt=2? (my Khong c6 nghiém nguyen duong. Tir dé suy ra dinh ly 1én Fecma dung véi n = 4. Chitng minh Gia sit phuong tinh (2) c6 nghiém. Goi (zo, yo, 20) Ja nghiém sao cho zo 1a nhé nhat. Ta c6: i) (xo, yo) = 1. That vay néu tréi lai goi p 1a witoc nguyen 16 chung cla xo, yo. Ta ©6 pila} + uo = 23 — P?|20 > #0 = pei, wo = PUL, 20= Pa > wt + uf = 2}. Vay (x1, yx, 21) IA nghiém véi 2, < 2p. Mau thudn. ii) Vay (23, y, zo) 1a mot bo ba Pitago nguyen thiy. Gia sit yo chan, xo lé. Khi d6 theo dinh ly 1 ta cé x3 =m? —r? (72) ye = 2mn zy =m? +n? trong d6 m,n la cde s6 nguyén dutong m > n, (m,n) = 1 va m,n Khéc tinh chin 18. Ty (3) suy ra (zo, n, m) lap thanh bd ba Pitago nguyén thiy. Lai theo dinh ly 1 a =a? - 0? n= 2ab maa? +b (73) trong dé a,b IA céc s6 nguyen duong a > b, (a,b) = 1 va a,b kh4c tinh chan Ie. Gid sit yo = 2ys. Ta c6 th (3) yB = dy? = 2mn = dab(a? +0?) 4 ye = ab(a? +b?) = abm. Lai cé (a,b) = 1 (a,m) = (b,m) =1> a =a2,b=i,m= mj. Thay vao (4) ta dugc m} = a4 + bf. Vay (a1,bi,7m1) Ja nghiém cia (2) v6 mi < m3? =m < m? +n? = 2p. Mau thudn, Hé qua Dinh ly 16n Fecma ding voi n = 28,5 > 2. That vay suy tir 2 + y= 22° > (2?) 4 (y2?)4 = (224, Ménh dé Néu dinh ly Fecma lén ding cho moi s6 nguyén t6 1é p thi né din: V6i moi n > 3. Chitng minh Theo dinh ly wen ta chi can ching minh véi tudng hop n c6 ué nguyen 6 lé p. Gid sit n = mp. Khi d6 2” + y" = 2" > (2™)P + (y™)P = (2) Mau thuan vi dinh ly Fecma lén ding cho moi s6 nguyen t6 lé p. Ole da chimg minh dinh ly Fecma 16n vdi n = 3, Diricle voi n = 5 nam 182 va Lame v6i n = 7 nam 1825. Nam 1993 di chimg minh dinh ly Fecma v6i moi s nguyen t6 p < 4.108. Dinh ly 3 Phuong trinh khong c6 nghiém nguyén duong. Chitng minh Gia sit phuong trinh (5) ¢6 nghiém. Goi (x0, yo, 20) IA nghiér cho zg 1a nhé nat, Ta c6: i) (vo, yo) = 1. That vay n€u tréi lai goi p 1A uttoc nguyen t6 chung cia zp, y £6 piled — yg = 23 + p?l20 + a0 = per, yo = Pyi, 20 = pa + af — yf = 22, (#1, y1, 21) A nghiém v6i 2 <2. Mau thudn. ii) Ta c6 (x8)? = (3)? + 29. Do d6 (ug, zo, 23) IA bd ba Pitago nguyen thay a) Néu yo Ié. Theo dinh ly 1 t6n tai m, m lA cdc s6 nguyen duong m > n, (m,n ya m,n khac tinh chan 1é sao cho 43 = m? ~ n?, 22 = m? +2. Suy ra mt — (zoyo)?. Vay (m,n, coyo) [A mot nghiém cia (5), Nhuumg m? < m? +n? m n,(m,n) = 1 va m,n Khéc tinh chan 1é sao cho 43 = 2mn,a2 = m?- Vay (rm, n, xo) IA mot bd ba Pitago nguyén thity.Theo dinh ly 1 t6n tai cc sO ng duong a > b,(a,b) = 1 va a,b khdc tinh chan lé sao cho zo = a? + 8? con a? — bn = 2ab hodc m = 2ab,n = a? — B. Trong moi trudng hop ta luc mn = 2ab(a? — b?) —+ yf = 2mn = dab(a? — b) -> y? = ab(a? — b*). Vi (a,b) nén (a, a?—b?) = 1; (b,a?—b?) = 1. Vaya =a?,b = bi, 0?—B? =r? af 04 : Vay (a1, 1,7) 1a nghiém cia (5). Nhuung a; < a? +b} = a+b b, (a,b) = 1 va a,b khac chan [é sao cho 2y3 = 2ab,a3 =a? +0? +a=r?,b=s? + af=r44s4 ws dinh ly 2. Vi du 2 Ching minh ring phuong trinh 24 + 4y4 = 2? khong 6 nghiém ng duong. Gidi Gia sit phuong trinh c6 nghiém. Goi (zo, yo, zo) IA nghi¢m v6i 2 nha.Tuong wr nhu tren ta c6 (z9,yo) = 1.Gid sit zo chin xg = 2h. Thay 16k* + dug = 22 + zo = 2h,4k4 + yf = bh? Vay (yo,k,h) 1 nghien h < 2h = 2. Mau thudn.Vay zo 1é. Ta c6 (23)? + (248)? = 23. Do zo (xo, yo) = 1 nen (22, 242) = 1. Suy ra (x2, 2y2, 29) IA bd ba Pitago nguyén thay d6 t6n tai cc s6 nguyen duong a > 6, (a,b) = 1 va a,b khéc tinh chiin 18 sac 2yg = 2ab, 23 = a? — b? Sa =r?,b = s* of = rt — s4 wi voi dinh ly 3. Vi du 3 Gidi cdc phuong trinh: 1. at -Qyt=1 2. af —2yt = 108 Gidi 1. vt — 2yt = 1 a + (y2)t = (y4 + 1)?. Te dinh ly 2 suy ra phuong winh vo nghiem. N . a4 — 2yt = -1 & (y?)t— at = (yt — 1)? . Tir dinh ly 3 suy ra ta phi c6 (yf -1) =0 + y=1-+a%=1, Vay phuong tinh cé nghiem duy nhat r=y=l. Il. Phuong trinh kiéu Feema Xét bai todn sau: Cho s6 nguyén duong n > 2. Tim tat cd c&c s6 nguyen duong (a, b,c) phan biét sao cho a”, b",c" 1A mot cp s6 cong. Baj toan nay tuong duong véi tim nghiém nguyén duong x 4 y cia phuong trinh a” fy" = 22? (75) Phuong trinh (6) goi JA phuong trinh kiéu Fecma(Fermat-like equations). 1. Xét trong hop n = 2. Phuong trinh (6) w thanh 2? + y? = 227. Ta chi cén xét bd nghiém (2, y, z) nguyén thily tte 14 (x,y,z) = 1. Ta cé a, y khong cing chfin.Gia sit x 1é. Suy ra y Ie. Vay vé trai dng du 2 mod 4 vay z lé. Néu plz, ply > plz 3 p= 1. Vay (x,y) = 1. Gid sit a > y.Dat u=(x+y)/2, (@@-y)/23wW4v=2. (ey) =le=uteysu-v 3 (uv)= Vay (u,v, 2) 18 bd ba Pitago nguyen thay. Vay u=m? —n?(=2mn) v = 2mn(= m? — n?) z=m +n? trong d6 m,n Ja cdc s6 nguyén duong m > n,(m,n) = 1 va m,n khdc tinh chin 1é hay z= m —n? +2mn= (m+n)? — 2n? z= m= + rn? Vi vai trd x,y binh ding nén ta két luan nghiém 1a a= (m+n)? —2n? y= (min) — 2m? zem tn? 8 d6 m,n la céc s6 nguyen duong nguyén t6 cing nhau va m,n khéc tinh chin le. V6i m = 2,n = 1 ta duge bd ba sO chinh phyong (1,25, 49) lap thanh mot caf s6 cong. 2. Xé truudng hop n > 2. Fecma da chuting minh rang ba s6 dang a*, b*, ct khong thé lap thnhf c&p sé cong. Euler chuting minh ring 3 sO dang a, b3,c? khong thé Jap thanh cap s6 cong. Méi day (1997) Merel va Darmon da chimg mint voi mgi n > 2 khong t6n tai ba sO nguyen duong (a,b,c) phan biét sac a”, B",c" 1a mot cap sO cong. Chui thich Nha toan hoc Euler di phéng doan ring phuong trinh Fecma « big attytyctadt ciing khong c6 nghiém. Tuy nhién nam 1988 nha todn hoc Elkies da chi ra phéng doén tren 1a sai bing céch chi ra mot nghi¢m sau day 26824404 + 15365639" + 18796760" = 20615673". IV. Biéu dién sé nguyén dwong thanh téng cdc binh phwong Ce nha todn hoe cha nhiéu thdi dai di quan tim t6i bai todn bidu dién sO nguyen duong thang téng c4c binh phusng (tng cdc sé chinh phuong). phant,Ferma,Buler, Gauss va Lagrange IA nhimg nha toan hoc da c6 déng gép 161 bai toan nay. 1. Biéu dién sé nguyén dudng thanh téng cua hai binh phuo Ta xét bai toén: Sé nguyen duong nao biéu dién dugc thanh téng cla hai phuong tic 1a V6i n nao thi phuong trinh 2? + y? = n c6 nghiém «,y EN. Dinh 1y 4 Cho p IA s6 nguyén t6. Khi d6 phuong tinh 2? + y? = p c6 ng ty nhién khi va chi khi p khong c6 dang 4k + 3. Chitng minh Gia sit phuong trinh c6 nghiém (x,y). Né&u p = 4k+3 + ple,p pp. Mau thudn. Nguge lai gid sit p khong c6 dang 4k +3. Neu p = 2 thi | la nghiem. Xét p = 4k + 1.Vi —1 1a s6 chinh phuong(mod p) nén tén tai a € a? = —1 (mod p). Dat q = [/p]. Xét (q+ 1)? 86 sau {x + ay}, a = 0,1,..49 . Vi (g+1)? > p nen t6n tai (21, y1) # (2, ya) sa0 cho 2 +ay, = @2- (mod p) + (2, — 12) = a(y2 — y1) (mod p) + u? = av? = —v? (mod z w+? =0 (mod p) 6d6 u= |ay —29| << VB; v=|u—ml 1 1a sé nguyén duong v6i phan tich ti¢u chudn n= QTp} Taft trong d6 pj = 1 (mod 4), q =3 (mod 4). Khi d6 n biéu dién duge thanh téng cia hai binh phuong khi va chi khi t; chin v6i moi é. Ching minh Diéu kien cin: Gid sit n biéu dién duge thanh téng cia hai phuong va gid sir q 12 ude nguyen t6 clan, g = 4k+3 vag c6 86 mi t la s6 18. K n=22 +y? = gb voi (b,g) = 1. Theo bd dé2 2 = n,y=qu > att+y= 110 néut > 2. Sau mot s6 hitu han buéc ta din dén x} + yz = gb, Theo bé dé 1, ta c6 mau thudn, Diéu kién di:Goi D 1a tap cdc s6n biéu dién duge thanh tng cia hai binh phuong . Gia sir ¢; IA sO chin v6i moi 4. Dat m = 2"Tp$. Ta c6 2 € D, pi € D do dé theo bé dé 2 me D. Vay tén tai x,y nguyen duong sao cho x? +4? = m.Vi t; IA s6 chin V6i moi t nén Tg! = h?. Thanh thir n = mh? = (wh)? + (yh)®. Chimg t6 n € D. 2. Biéu dién sé nguyén duong thanh téng céa ba binh phwong Ta xét bai todn: SO nguyén duong nao biéu dién dugc thanh téng cia ba binh phuong tic 1 V6i n nao thi phuong trinh «? + y? + 2? =n c6 nghiém 2, y,2 EN. Dinh ly 6 Néu n cé dang n = 4"(8k +7), m,k EN thi n khong biéu dién duge thanh téng cia ba binh phuong. Chitng minh Gia sir wai lain = 4"°(8k +7) = a? +y? + 2”. Ta théy 2? a € {0;1;4} (mod 8) do dé n = 2? + y? 4+ 2? = a € {0,1,2,3,4,5,6} (mod 8). Né&u m > 0 thi n = 0 (mod 4) do d6 2? + y? + 2? = a € {0,4} (mod 8) > 2? +y? +22 =0 (mod 4). Viz? =a € {0;1;} (mod 4) wy? ays2s (mod 4) + @ = 221, y = 2y1,z = 2m 3 22 + y? + 22 = 418k +7). Tigp tuc nhu thé ta din dén x2, + y?, + 22, =8k+7=7 (mod 8). Mau thuin. Diéu ngugc lai cling ding ufc 1a néu n khong c6 dang n = 4(8k +7),m,k EN thi n biéu dién duge thanh téng cia ba binh phuong. Ching minh khé nén ta cong nhan. Nhu vay ta c6 dinh ly sau Dinh ly 7 SO nguyen duong n biéu dién duge thanh téng cla ba binh phuong khi va chi khi n # 4""(8k +7) v6i ky m EN Tiép theo ta xét bai todn :Biéu dién duge thanh t6ng cia ba binh phuong, trong dé c6 hai binh phuong tring nhau tic 1a tim n dé phuong tinh n = «? + 2y? c6 nghi¢m tu nhién. Bé dé 4 Néu pix + 2y? va p 1a sO nguyen 16 p = a € {5,7} (mod 8) thi pix ply That vay néu wai lai thi (x, p) = (y,p) = 1 2? = —2y? (mod +)z?-1 = (—2)@-D/2yP-1 _, (—1)(®-1/29P-1/2 = 1 (mod p). Néu p = 8k +5 thi 2 IA se khong chinh phuong (mod p) con p ~ 1/2 = 4k +2 chin do d6 (—1)®-1)/?gp-1/2 —1 (mod p). Mau thudn. Néu p = 8k +7 thi 2 1a sO chinh phuong(mod p) cor (p — 1)/2 = 4k +3 18 do d6 (—1)®-Y/29P-1/2 = _1 (mod p). Mau thuan. Ky hiéu D 1a tap hop cdc s6 n dé phuong trinh n = x? + 2y? c6 nghiém ty nhién Dinh ly 8 Gia sit n = p la sO nguyen t6. Khi d6 p € D khi va chi khi p = ¢ hodc p = 1,3 (mod 8) Chitng minh Gia sit phuong tinh cé nghiém (2, y) va p = £5,7 (mod 8). Thec b6 dé 1 x chia hét cho p , y chia hét cho p. VE trai chia hét cho p? . Vo ly Dao lai: Néu p = 2 thi'2 = 0? +2.12.Xé p = 1,3 (mod.8). Khi d6 —2 1a s¢ chinh phuong (mod p) ( Suy ti 2 14 chinh phutong (mod p) khi va chi khi p = +) (mod 8)). Vay t6n tai a € N théa man a? = —2 (mod p). Xét tap {x + ay) wrong d6 x = 1,2,...,m:y = 1,2,...,.m voi m = [Vp]. Tap nay c6 (m +1) s6 va vip < (m+ 1)? nén t6n tai (1,41) # (x2, y2) dé ay + ay, = xp + ay, (mod p) + (1 — 22) = a(y2— 1) (mod p) + (1 — 22)? = a?(y2 11) (mod p) Date = |r — aal,y = ly - yl. eb a? cm < gy 2 mot so hitu han bude ta din dén x} + 2yz = gb, Theo bé dé 2, ta c6 mau thudin Vi du 14 = 1? +2? +3? Ja tng cia 3 binh phuong nhung 14 = 2.7 va7 = £ c6 sO mi 1é nén phuong trinh 14 = 2? + 2y? vo nghiem. 3. Biéu dién sé nguyén duong thanh téng cia bén binh phu Néu mot so khong biéu din thanh téng cia hai hay ba binh phuong thi li c6 thé thinh téng céa bon binh phuong hay khong? Cau trd ldi 1a: Moi sé’ ng duong déu biéu dién duoc thanh tong cia bdn binh phuongD. Dé Ia di néi tiéng cia Lagrange. Sau day ta sé chimg minh dinh ly nay. Ky higu D 12 tap hop céc s6 n sao cho n biéu dién duge thinh téng cla bér phuong tic 1a dé phuong trinh n = 2? + y? +z? +0? cé nghiem ty nhien. BO dé 1. N&ume D,ne€ D thi mne D. Ching minh Gia sit m = a? +0? +c? +.d?,n =e? + f? +9? +h?. Khi dé hang ding thie sau mn = (ae + bf +.cg + dh)? + (af — be + ch — dg)? +(ag — bh — ce + df)’ + (ah + bg ~ cf — de)? Vay mn € D. Chang han 7 = 214 124.124 11,10 = 3! + 11 + 0° + 0° uni 4p dung con trén cho ta 70 = 7? + 1? + 2? +42, Sau day ta sé chimg minh véi moi sO nguyén Wp tacé pe D. Vi2=1)4 0° + 0° € D nen ta chi can xét p > 3. BO dé 2 Cho p 1a s6 nguyen t6 18. Khi d6 t6n tail a? +y? = kp > kp = 2? +y? +17 407, Vay kp €D. Laicé kp =a? ty}

mip > m=p— m2 p( dom > 1 va p nguyen 16). Mau thudn . +Vayl4Y2 4 224.7% = m?(X2 + ¥2 + 2347?) = m?kp > XP4Y2 42247? = kp Vay kp ¢ D & k € M. Mau thuan véi viée m 1A sO nho nhat cha M. Ti dinh ly 2 va bd dé 1 ta dt ra Dinh ly 11( Lagrande) Moi s6 nguyen duong déu biéu dién duge thanh tén bén binh phuong. Tiép theo ta xét bai todn : SO nguyén duong nao biéu dién duge thanh tén; b6n binh phuong, trong d6 c6 ba binh phuong tring nhau tic 1a tim n dé phuong n=? + 3y? c6 nghiém ty nhién. Bé dé 1 Néu plc? + 3y? va p 1a sO nguyen t6 p = 2 (mod 3) thi plz ‘That vay néu trai lai thi (xp) = (y,p) = 1 + 2? = —3y? (mod +)a? (-3)@-D/2yp-1 _y (—1)(@-10/23p-/2 = 1 (mod p). Vi p = 3k-+2 nen p = 12 hodc p = 12k +11. Néu p= 12h +5 thi 3 [A sO khong chinh phuong (mod f p—1/2 = 6k +4 chan do dé (—1)(?-1)/2gP-1/2 = —1 (mod p). Mau thudn. p = 12k + 11 thi 3 A sO chinh phuong(mod p) cdn (p — 1)/2 = 6k +5 18 (-1)@-0/23P-1/2 = —1 (mod p). Mau thudn. Ky higu D 1a tap hop cdc s6 n dé phuong trinh n = x? + 3y? co nghiém tw 1 Dinh ly 12 Gia sit n = p [A s6 nguyen t6. Khi d6 p € D khi va chi khi ; hoac 1 (mod 3) Chitng minh Gia sit phutong trinh cé nghiém (x,y) va p= 2 (mod 3). Th dé 1 @ chia hét cho p , y chia hét cho p. VE tréi chia hét cho p? . Vo ly. Dio lai p=3thi3 = 0? +3.17.Xé p= 1 (mod 3). Khi d6 —3 Ia s6 chinh phuong (1 ( Suy tir 3 1a chinh phuong (mod p) khi va chi khi p = £1 (mod 1)2). Vay t a € N théa min a? = —3 (mod p). Xét tap {a + ay} trong dé x = 1,2,...,m 1,2,...,m v6i m = |/p]. Tap nay c6 (m+ 1)? s6 va vi p < (m+1)? nent (1,11) # (@2,y2) dé x1 + ayy = 22 + aye (mod p) + (21 — 22) = alye (mod p) -+ (v1 ~ 22)? = @?(y2 — yn)? (mod p). Dat ¢ = Jey — 22],y = [yr .Tacé 2? 2{p. Mau thudn, Néu 2? + 3y? thi ¢ = 3z 3 92? + 3y? = 3p 3 y? +322 =p peED. Bé dé 2Néune D,meD thi nme D. Chiing minh Gia sit n = a? + 3b*,m = ce? + 3d”. Khi d6 nm = (a? + 3b?) (c? + 3d?) = (ac — 3bd)? + 3(be + ad)? Diéu nay ching minh nm € D. Dinh ly 13. Gia sit n c6 phan tich tieu chudn n= 3p Iqit 6 d6 p; IA cdc s6 nguyen t6 dang 3k + 1, q; 1A cdc s6 nguyen t dang 3k +2 K n€ D néu va chi néu ¢; 1 s6 chan voi moi i. Chitng minh Gia sit t; 14 s6 chin v6i moi i. Dat m = 3"IIp*. Ta c6 3 € T D do dé theo bd dé 1m € D. Vay tén tai x, y nguyén duong sao cho 2?+3y? = t 1 sO chain v6i moi ¢; nén TIgj = h?, Thanh thitn = mh? = (xh)? +3(yh)?. ( toneD. Dio lai gid sit n € D wie 1A tén tai x,y sao cho x? + 3y? = n. Gid str q | nguyén t6 chan, q = 3k +2 vag c6 sO mii t 1a sO Ie. Kki dé 2? + 3y? = q 114 (bg) =1. Theo bé dé 2 2 = qay,y = gy > 2? + 3y? = 57d néus > 2. Sau mo s6 hitu han bude ta din dén xz + 3y2 = qb, Theo bé dé 2 suy ra vé tr4i chia hét cho @. Ta c6 mau thudn, 4. Bai toan Waring: Biéu dién mét sé thanh téng cc lay thita k. Vao thé ky 18 nha todn hoc Anh Waring di phdng dodn ring, moi sO nguyen duong déu biéu dién duge thanh téng cia 9 lap phuong céc sO ty nhién va déu biéu dién duge thanh tng cia 19 lily thuda 4 cdc s6 wr nhien, tic 1A Voi moi n € N* cdc phuong tinh ai+ad---tagan titabt---tafp=n luon cé nghiém tr nhién. Ong da neu gid thiét sau Cho s6 nguyén duong k. Khi dé c6 tén tai 86’ nguyén duong m ( plu thudc vao k) sao cho moi so nguyén duong n déu biéu dién duoc thanh téng cla m $6 , moi so 6 dang c*, x € N nite 1a: Tén tai sé nguyén duong m sao cho véi moi s6 nguyén duong n phuong tinh 6 nghiém ty nhién. Nam 1906 nha toan hoc 18i lac Davit Hilbert di ching minh duge phéng dodn tren, Ching minh ca Ong cyc ky phife tap. Goi g(k) 1a 86 m nhd nhét c6 tinh chét tren téc 1A moi s6 nguyen duong n déu bidu dién dugc thanh téng cia g(k) s6, mdi sO 6 dang 2*,x € N va tn tai s6 n ‘khong biéu dién duge thanh téng cia m — 1 s6 , méi s6 c6 dang x*,c EN. Ta cd 9(2) = 4 (vi s6 7 khong biéu dién duge thanh téng cia 3 binh phuong va moi s6 nguyen duong 7 déu biéu dién dugc thanh téng cia bon binh phuong).Dén nay ngudi la da ching minh duoc 9(3) = 9, 9(4) > 19, g(5) = 37 va voi 6 < 471600000 thi o(k) = [(8/2)*] + 2 ~2. Vin con nhiéu cau héi mé xung quanh ham g(k). V. Biéu dién mét sé thanh téng cac binh phuong nguyén duong 1. Téng cia hai binh phwong nguyén dwong ‘Mot s6 c6 thé 1A téng cia hai binh phuong song c6 thé mot s6 bing 0 ( ching han 9 = 37,16 = 4? nhumg khong thé c6 9 = a? + #16 = 2 + d? véi a,b nguyén duong.Dinh ly 1 06 thé phat biéu cdch khéc Ia: Dé mot sO nguuyén duongn biéu dién duoc thanh téng cha khong qué hai binh phuong nguyén duong diéu kién cAn va di 1a trong phan tich tiéu chudn cia n cdc ude nguyen t6 dang 4k +3 phai c6 sé mii chan. Bay gid ta di tim diéu kién cla n dé n6 biéu dién duge thanh téng cla ding hai binh phuong nguyén duong. Dinh ly 14 Dé n dé n6 biéu dign duge thanh téng cla ding hai binh phuong nguyen duong ( ttc 1a dé phuong tinh n = x? + y? cé nghiém nguuyen duong diéu kién can va di a © Néu n [8 sO chinh phuong thi n phai c6 ft nhat mot ude nguyén 16 dang 4k+1 ¢ Néu 7 1a s6 khong chinh phuong thi trong phan tich tiéu chudn cia nc: nguyen t6 dang 4k +3 phai c6 sé mii chan. Chitng minh 1. Gid stn = m? = a? + a,b E N* va na Pg? vOi gi dang 4k +3. Xétq = qi 4 a= qay,b= qh; > a +B} = 2?*Tlg?*-*. § sO hitu han busc ta di dén c? +d? = 277 = 47 Qdy + cf +d? = 47-1. Sau mot sO hitu han buée ta di dén u? + v? = 1 voi u,v Mau thudn. Dio lai gid si pin,p p=@Ptardb—+p? Nt + n= (kA)? + (kB)? 2. Diu kién cin hién nhien do dinh ly 1. Ngugc lai , do n Ia sé khong phuong va trong phan tich tiéu chudn cla n cdc ude nguyén 16 dang 4k +3 5 s6 mii chin nén n = mA v6i h = pypo...pz , pi 1a cdc s6 nguyén t6 phan biét c6 dang 4k +3. Theo dinh ly 1 h = a? +8, Do hi IA tich cdc sO nguyen 16 ph nén a,b #0. Vay n = (ma)? + (mb)?. 2. Téng cia ba binh phuong nguyén dudng Dinh ly 15 Dé phuong winh n = 2? + 2y? c6 nghiém nguyen duong dié can va di 4k +13 plm 3 m= pk + n = pPk*. a? +b)? = (a? — b?)? + (2ab)? = A? + B?,, # Néu nla s6 chinh phuong thi n phai 6 it nhét mot uée nguyén 16 dang | hay 8% +3 e Néu 7 [a sé khong chinh phuong thi trong phan tich ti@u chudn cia nc nguyén t6 p= 2Vp=8k+5Vp=8k+7 phai c6 sO mi chin. Ching minh 1. Gia sit tai lai n = m? = a? + 26?,a,b € Nt va n= Pg V6i gi = 8K +5 V 8k +7. Xét q = qj. Theo bé dé 1 a = qay,b = gh; > a} - 2°rT1g?-?, Sau mot sO hitu han bute ta di dén c? 4 2d? = 27" = 4" 4 c= 2 2d; > c} + 2d? = 4°71. Sau mot sO hitu han bude ta di dén u? + 2v? = u,v € N*. Mau thudn. Dao lai gid sit pln,p = 8k +1V 8k +3 > plm 3 m= pk >n=p? 6 p = a? + 2b?, + p? = (a? + 207)? = (a? — 207)? + 2(2ab)? = A? + 28%, Nt n= (kA)? +2(kB)? 2. Diéu kién cn : Do dinh ly 1 thi trong phan tich tigu chudn cha n c nguyén 16, p = 8k +5 V 8k +7 phai c6 sO mii chan, Gia sit so mii cia 2 | s.Tuong ty nhu trén sau mot sé hiu han bude ta di dén c? + 2d? = 2°. Véi s suy rac chin do dé d chin. Vay ¢ = 2cy,d = 2d; + c? + 2d? = 2°-?, Sau hdu han bu6e ta di dén u? + 2v? = 2 voi u,v € N*, Mau thudn. Nguge lai , do n 1a s khong chinh phuong va trong phan tich tiéu chudn cdc ude nguyén 16 q dang q = 8k + 5,q = 8k +7 va q = 2 phai cé s6 mii ch n=m?h vi h = pip...pk , pi lA cdc sO nguyen t6 phan biét pj = 8k +1 VE Theo dinh ly 1 h = a? + 26%, Néu b = 0 thi h = a? vo ly do h LA tich 6 nguyen t6 phan biet. Néu a = 0 + h = 26? vo ly do h le. Vay a,b © N* va n= (ma)? + 2(mb)?. Dinh ly 16 Dé phuong trinh n? = 2? +2? + 2? c6 nghiém nguyen duong diéu kién cAn va di Bn 2° van #£ 5.2° Chitng minh Diéu kien cin: Nun=2 39 #& =a yo230= 2ay,y = 2yi,z = 221 3 4°} = 2} + y? +23. Sau mot sO hitu han bute ta di dén L=a2+y? +22. Mau thudn. Néu n = 5.29 > 25.48 = a? ty? +2? 3 2 = 291, y = 2y1,2 = 2a 3 25.49 = 22+ yf + 2}.Sau mot sO hitu han bute ta di dén 25 = 22 +4? + 22. Phuong tinh nay khong c6 nghiém nguyén duong. Diéu kien di: Vi n # 2° nén n c6 ube nguyén t6 18 . © nc6 mot ude nguyén t6 18 p #5. Vay n= ph 4 n? = p?h®. Ta chi cin ching minh p=X?+Y?4Z? voi X,Y,ZEN* +Néu p = 3 (mod 4) thi p = 8k +3V p = 8k +7. Néu p = 8k +3 thi theo dinh ly 2 p=w4u+e, voi u,v,tEN Néu cé mot s6 ching han t = 0 th) p = u? +. v? vo ly vip = 3 (mod 4). Vay u,v,t € N*. Do vay pP=(wtv—?) + (2ut)’ + (2vt)? = xX? +¥? +2? Viplenenw tv? AP + X,Y,Z40. Néu p = 8k +7 thi 2p # 4°(81 +7) do d6 2p =u? + eu? +t? 4p? = (u? +0? — #7)? + (Que)? + (2ut)? ae ops (4) + (uv)? + (vt)? = (p—@)? + (uv)? + (vt)? = X?4+¥?427, voi X,Y,ZEN* + Néu p= 1 (mod 4) . Theo dinh ly p? = a? + B?, a,b € N*. N&u cd hai sé a,b déu khong chia hét cho 5 thi p? = a? + 6? = 0,+2 (mod 5). Mau thudn. Vay phai c6 mot sO chia hét cho 5 chang han a = 5m. Khi d6 p? = 25m? +P = (3m)? + (4m)? + = X74 ¥2 4 Py n chi c6 uée nguyen 16 18 p¥#5. Vay n= 5'2°, Ta c6 n £5.28 +t >24n = 25m — n? = 625m? = (20m)? + (12m)? + (9mP 3. Téng cha bén binh phuong nguyén duong Ta biét rang v6i moi sO nguyen duong n thi n biéu dién duoc thanh téng cha bén binh phuong. Tuy nhién khong nhat thiét n biéu dién duge thanh téng cia bén binh phuong duong. Vi du Ching minh ring n = 2° véi s 18 khong biéu dién duoc thanh tén b6n binh phuong duong. That vay néu 2° = 22 + y2 42? 42 thi z,y,2,t chin. Dat c = 2x Qyy,z = 224, t = Wty 4 2°-? = a} + y? + 22 + t2. Sau mot sO hiu han bude 2=a? +0 +c? 44? > 4. Mau thuin, Dinh ly 17 Phuong trinh n? = 2? + y? + 2? + ¢ c6 nghiem nguyen duor va chi khi n # 1,3. Chiing minh Dé ching minh 1 va 9 khong biéu dién duge thanh téng ci binh phuong duong. Bay gid ta ching minh V6i moi s6 nguyen duong n # 1 phuong trinh n? =a? + y? 427427. NGu n= 2k +n? = 4k? = heh PR? Gia sin > 118. Dé thay chi cn chi ran c6 mot ude nguyen t6 p biéu dién thanh téng cia bon binh phuong duong. © nc6 mot ude nguyén 16 p > 26 > p* — 676 € N*. Ta c6 p? — 676 =1- (mod 8). Do d6 p? — 676 = a? + +c%, Név a,b,c > 0 > pF +P 4+c% Nrab>0,c =0 3 p? = 10? 424 40? +22 a>0,b=c=0 3p? = 6748? +24 +02. © Moi uéc nguyen t6 cia n nhé hon 26 : Khi dé hoac +n c6 ude nguyén 16 p thugc tap S = {5,7, 13,17, 11,19, 23}. Khi d p € {5,13,17,11,19} + p 4 8k+7dod6p=art+bh4c%, N&ux O;c = 0 > p? = (a? + b?)? = (a?)? + (b?)? + (ab)? + (ab)?. Néu a,b, thi khong thé 3 s bang nhau ching han a # b. Khi dé p? = (a? +b? + (2ab +c”)? + (a? — B)? + (ca — ch)? + (ca - cb)*. Con néu p € {7;23} thitac6 7? = 1? +4? 44? 4.47, 23? = 1? +47 +16? +n chi c6 uéc nguyén 16 p = 3. Khi dé n= 3°,s >23n=9m> 81m? = (2m)? + (2m)? + (3m)? + (8m)? Dinh ly 18 Dé phuong trinh n = z? + 3y? c6 nghiém nguyén duong diéu kic va di 1 # Nun 18 sO chinh phuong thi n phai cé it nhat mot uée nguyen t6 dang 3) # Néu nA sé khong chinh phuong thi trong phan tich tiéu chudn ca n uéen 16 p =3,p = 3k +2 phai cd sO mii chin. Chitng minh 1. Gia sit trai lai n = m? = a? + 367, a,b € Nt va n= 3g? v6i gq; = 3k +2. Xét q = qi. Theo bé dé 1 a = gay,b = gh, + a} +30? = 31 Sau mot sO hitu han bude ta di dén c? + 3d? = 37 = 9 + ¢ = 3c,d=% cf + 3a? = 97-1. Sau mot s6 hiu han bude ta di dén u? + 3v? = 1 véi u,v Mau thudn. Péo lai gid sit pln,p = 3k +1 3 plm 3 m= pk 3 n = pre? p =a? + 3b2,~+ p? = (a? + 307)? = (a? — 30”)? + 3(2ab)? = A? + 3B, / Nt + n= (kA)? + 3(kB)? 118 2. Diéu kién cAn : Do dinh ly 1 thi trong phan tich tieu chudin cia n cdc use nguyen 16, p = 3k + 2 phai c6 s6 mf chan, Gid sit s6 mf cia 3 I sO 1é s.Tuong ur nhw trén sau mt sO hitu han bude ta di dén c? + 3d? = 3°. Véi s > 3 ta suy ra 3]c do d6 3]d chin. Vay c= 8c1,d = 3d; + cf + 3d? = 3°-7, Sau mot s6 hitu han bute ta di dé u? + 3v? = 3 véi u,v € N*. Mau thuan. Nguge lai , do n 1& sO khong chinh phuong va trong phan tich tiéu chudn cha n cdc udc nguyén t6 g dang g = 3k + 1 va p = 3 phai c6 s6 mo chin nén n = mh h = pipo...pk , pi 1A céc sO nguyén 16 phan biet pp = 3k +1. Theo dinh ly 1 h = a? + 36%, Néu b = 0 thi h = a? vo ly do h Ia tich cdc s6 nguyen 16 phan bigt . Neu a = 0 + h = 38? vo ly do h khong chia hét cho 3. Vay a,b € Nt va n= (ma)? + 3(mb)?. Cu6i cing ta 06 Dinh 1¥ 19 Moi s6 nguyen duong n > 169 déu biéu dién téng cia nam binh phuong duong. Chititng minh Ta c6 n — 169 = a? +b? +e +d?) ‘acre © N&va,b,c,d>O0thin=12 404 424a ‘ © a,b,c>0,d=0thin=1248+404R 47 © Né&u a,b >0,c=d=0 thin = 12? 44743740740? a4) « N&va>0,b=c=d=0thin=10?48?427 41244? © N&wa=b=c=d=0 thin= 169 = 274+ 2? +5746? + 10? Bang cAch thir tryc tiép véi cdc sO n < 169 ta thay tit cA chi tit ra cAc s6: 1,2,3,4, 6,7, 9, 10,12, 15,18, 33 1a biéu dién téng cia ndm binh phutong duong. Vay Dinh ly 20 Moi s6 nguyen duong n ¢ {1, 2/3) 4/6) 7,9)10, 12} 15,18)33} déu bigu dién téng cia nam binh phuong duong. Sau day ching ta sé 4p dung cdc két qua tren dé khdo s4t mot ham s6 hoe duge dinh nghia nhu sau: V6i méi s6 nguyen duong n > 5 goi S(n) 1A s6 nguyen duong k lén nhat 6 tinh chat: V6i moi 1 < k < S(n) thi n® biéu dién téng cla k binh phuong duong. Ta cé dinh ly sau Dinh ly 21 1 néu n khong c6 uoc nguyen t6 p = 4k +1 2 néu n = 5.2° nr? —14 néu tri lai Chiing minh Truoc hét nhan xét ring n? tuon biéu dién téng cla 1 binh phuong duong. Vay S(n) > 1Gid sit n Khong c6 voc nguyén t6 p = 4k +1. Theo dinh ly thi n? Khong biéu dién téng cia 2 binh phuong duong. Vay S(n) <2 S(n) =1. Gia sit n = 5.2°. Theo dinh ly 1 ( vi 5 1A woe nguyen t6 dang 4k +1) nén n? bidu dién téng cia 2 binh phuong duong. Theo dinh ly 2 n? khong biéu dién téng cia 3 binh phuong duong. Vay S(n) <3 S(n) = 2. Bay gid xét céc n con lai tie 1a n # 5.2% va n c6 uoc nguyen 16 p= 4k +1. Vi n c6 ude nguyén 16 p = 4k + 1 nén n? biéu dién téng cia 2 binh phuong duong. Vi ncé ude nguyén 16 p = 4k +1 suy ran #28. Vin # 5.2°,n ¢ 2° nén theo ly 2 n? bigu dién téng cia 3 binh phuong duong. Ta cé t6n tai a,b € N* sa n? =a? +02. Theo dinh ly Pitago n 06 dang n= d(m? +n?) n? = (mm! + n* + (mn)? + (mn)?) = (dm?)? + (dn®)? + (dn)? + (dmn)? Vay n? biéu dién téng cha 4 binh phuong duong. Ti€p theo ta ching minh véi moi 5 < k < n? — 14 thi n? bidu dién tén k binh phuong duong, nhuung khong biéu dién duoc thinh téng cia n* — 13 phuong duong. That vay vik < n?-14 > n?-k+5 > 19. Dau tie k fn? — 28 + n? —k +5 £33. Theo dinh ly phuong trinh n? —k +5 = c6 nghiém nguyen duong a;. Vay 5 n= Soa? $141.41 = ae hay n? biéu dién t6ng cla k binh phuong duong. Néu k =n? — 28+ k > 6. \ +143? +3? +27 427 42? 42? hay n? biéu dién téng cla k binh phuong duong. Gia sir trai lai n? biéu dién téng cha & binh phuong duong voi k = n? — 13 t6n tai k sO nguyén duong 2}, ...,7, sao cho k ua a x nr —k= (x? -1) ist k 13 = }°(e? - 1) Z =e at Vi0 3 Ia sO nguyen 16 dang 4k +3,(g;p) = 1 Khi dé n ¢ K. N6i rieng, 06 vo 6 s6 nguyén duong chin va vO s6 sO nguyén duong Ié n ¢ K Chitng minh Gia sir trai lai n € a v= Vet+iP > om 1 Pens? tn(nt led cee) 4 aion ») 2 (6x)? + (6y)? = 6(n +1) (62? + 6ne + n(2n + 1)) + =6p*gA> w+vr=pM voiu=6s,v=6y,M =69A Paya (nt(Ptned Ta cé plu? +v? -+ plu = 6x (do p = 4k+3), Lai c6 2n+1 = 2p%q—1 khong chia hét cho p do dé n(2n +1) khong chia hét cho p, Vay A = 60? + 6na + n(2n + 1) khong chia hét cho p. Vip > 3 nén (M,p) = 1. Lai c6 w2 +0? = p'M 3 u=pui,v= pry > uy + v? = p*-?M. Sau mot s6 hitu han bude din dén p(a? + 6?) = M. Mau thuan. Dinh ly 23 C6 vo s6 s6 nguyen duong 18 n € K. Chitng minh Ta xét téng : ; (z-jP+27+ > (et5) voia>k 1 ial Me P é ii Ta c6 p la téng cha n = 2k + 1 binh phuong lien ti€p bat dau ti sé (x — k)?. Dang thituc trén tuong duong véi P= (2k +1) (?+ Her) qm Dat k = 6a? — 1 <> n= 12a? — 1,2 = 2a(12a? — 1) +a vao (8 ) ta duoc P = (12a? — 1) (4(12a? — 1)?a? + 40?(12a? — 1) + a?(12a? — 1) = a?(12a? — 1)?(48a? + 1) Néu (b,a) 1a nghiém nguyén duong cia phuong trinh Pell 0 - 480? =1 ta suy ra P =a"(12a? — 1)?0? va 1S rang z = 2a(12a? — 1) +a = a(24a? — 1) > 24a? -1 > Ga? —-1 =k VE 1a sO chinh phuong tie [An = 12a? —1 € K néu chon (b, a) 1A nghiém nguyen d cla phuong trinh Pell 6? — 48a? = 1, Vi phuong trinh Pell 6? — 48a? = 1 c6 y nghiém va n = 12a? — 1 [a 6 1é nén dinh ly duge ching minh. Bai toan mé@ Héi cé hitu han hay vo han s6 chan thuge K. ‘Ta thy 2 1A sO chan bé nhat ( va cing 1a s6 bé nat) trong K. Ta hay tim so nhat. Dinh ly 24 $6 1é n bé nhét trong K lan = 11. Chitng minh Vi (7; 1) 1a nghiém phuong trinh Pell 6? — 48a? = 1 nén theo ly réenn =12-1=11EX. Gidst tintain=2kh+1 & > 8. Mau thudn, + Néu k Ié suy ra 8|2k?+3k+7 > 2k?43k+1 = 3k+1=0 (mod 8) +k (mod 8) + k > 5. Mau thuin. Néu y chan. Ti (8) suy ra « chin do d6 4|(b + 1)(2k +1) + Néu kl suy ra 4jk +1 — & = 3. Thay vao (8) ta duge y? = 7(2? ++ x? +4=0 (mod 7) -+ 2? =3 (mod 7). Mau thuan. + Néu k chin: 4|k(k + 1)(2k +1) + 4|k + & = 4. Thay vao (8) ta yP = 3(3e? + 20) — 3|8x? + 20 Mau thuan. Dinh ly duuge ching minh. 122 PHUONG TRINH DIOPHANT Trén Nam Dang 0.1 Mo dau Phuong trinh nghiém nguyén hay cdn duge goi 1A phuong trinh Diophant 1A mot trong nhiing dang todn lau ddi nhdt cia todn hoc. Tw Euclid, Diophantus, qua Fibonacci, r6i dn Fermat, Euler, Lebesgue ... va thoi hién dai 1& Gelfold, Matiasevic, Shenzel, Serpinsky ... phuong tinh Diophant da trai qua mot lich sir phat trién lau dai. Thong qua viée gidi c4c phuong trinh Diophant, c4c nha fodn hoc da tim ra duge nhiing tinh cht sau séc cia sO nguyen, sO hitu ty, s6 dai sO. Giai phuong trinh Dio- phant da dua dén si ra déi cia Lién phan s6, Ly thuyét dudng cong elliptic, Ly thuyét xp xi Diophant, Thang du binh phuong, Sé hoc modular . Trong céc ky thi hoc sinh gidi quéc gia va quéc té, phuong trinh Diophant vin thudng xuyén xuat hién duéi cdc hinh thite khéc nhau va lu6n duge danh gid 1a khé do tinh khong mau myc cia né. BAi ging nay c6 muc dich dua ra mOt s6 phuong phép co ban dé tn cong céc bai toan vé phuong trinh Diophant. Tuy nhién, v6i khd nang con han hep cia minh, chéng 10i hdan tOan khong c6 tham vong bao quat het céc vin dé vé phuong trinh Diophant. Ching t0i cha yéu chi gidi han trong c4c phuong trinh da thtfc, bd qua céc phuong trinh Diophant bac nhét va khong dé cap dén céc phuong trinh c6 chia ham mi. Day 14 mot tap tai Ligu md, mot s6 chtmg minh dug bd qua, mot s6 ldi gidi chi trinh bay so luc hode bd qua. Ching toi rat mong nhan duoc ¥ kién déng gop cla quy thay 6, quy anh chi va ban bé déng nghiép, cing nhu cia cdc em sinh vién, hoc sinh dé tap ti Ligu duge hoan thien hon. Chiing t0i xin chan thanh cém on GS Nguyén Van Mau va Trutng Dai hoc Khoa hoc Tu nhién DHQG HN da té chitc khod béi duéng nay va cho phép ching t6i duvc béo cdo truée cée anh chi va céc ban. 0.2. Phuong phap chon mé-dun Mot s6 chinh phuong khong thé tan cing bang 2,3,7,8. Mot sO chinh phuong chia 3 du 0 hogc 1. Mot s6 chinh phuong chia 8 du 0,1 hodc 4. Nhing tinh chit don giin 6 nhiéu khi lai 1A chia khod dé gidi nhiéu phuong trinh Diophant. Va d6 chinh [a ¥ tuong chinh cia phutong php chon mo-dun. Vi du 2.1. (Viet Nam 2003, Bang B) Hoi cé tén tai hay khong céc sé ng z,y,u,v,¢ thod man diéu kién sau P+ya(etlPewa(et2Pev=(e43P +? Léi gidi: Cach 1. That vay, gid str hé trén tén tai nghigm. Dat N 1a gid wi chung c bén bidu thitc. Ta c6 2? mod 8 € {0,1,4}, suy ra 2? +4? € {0,1,2,4,5}. 1 ra, néu x chay qua mot hé thang du ddy di mod 4 thi 2? mod 8 sé Ia (0,1 hay mot hoan vi vong quanh cia bo wen. Nhu thé N mod 8 € {0, 1,4} {1,2 {0,1,4} N {0, 4,5} = 0. Mau thudn. Vay khong tén tai cde sO nguyén zy. thoa man diéu kién dau bai. Céch 2. Ti hai phuong trinh dau ta suy ra y? + v? — 2u? = 2(@ +1)? — 2? - . Nhu thé, y? + v? = 2(u? — 1), suy ray, v cing tinh chan 1é. Néu y,v 18 thi vé wi chia 8 dur 2, suy ra u chan, nhumg khi d6, vé trai chia 8 du —2, mau t Vi du 2. Ching minh ring phuong trinh x? — 3y? = —1 khong c6 nghiém ng duong. Loi gidi: x + 1 khong thé chia hét cho 3! Vi du 3. (Bé dé nghi IMO 1981) Cho phuong trinh nghiém nguyén 2? = | 1)(y? — 1) +n. Héi phuong tinh cé nghiém khong néu (a) n= 1981 (b) n = 1985 (©) n= 1984? Loi gidi. a) Ta c6 2? = (2?-1)(y?-1)+5 mod 8. Viz? =0,1,4 mod 8, 1=0,3,7° mod 8, (x?—1)(y?—1) =0,1,5 mod 8 va (2?—1)(y2-1)+5 =: mod 8 nén ta c6 z? A (a? —1)(y? —1) +5 mod 8. Mau thudn. (b) Tuong ty, xét_ mod 9. (c) n = 1984. Rut gon phuong trinh, ta duge 2? + y? + 2? — a%y? = 198 ¥ tuéng cia ta 1A di tim biéu dién 2? + y? = 1985. Khi d6 z = xy cho ta ng Bing cdch xét chit s6 cui cing, ta nhanh chong di dén nghi¢m 7? + 44? va 31? bing phuong ph4p thir va sai, Ti d6 duge cdc nghi¢m (x,y,z) = (7,44,7-4 (31, 32, 31.32). Ghi chi: Phuong tinh (1) cé v6 s6 nghiém nguyén duong, hay thir chtmg BAI TAP 2.4 (Thuy Din 2003) Tim tat cA cde cap sO nguyen duung «,y sao cho a? — 3y = 2002 124 2.2 Tim nghiem nguyen cia cdc phuong trinh ep Mata ayes tat 5yP=7 ©) 5a? +60 +11 = y? +4y 2.3 Tim nghiém nguyén cba phuong trinh 2!° + y!0 — 210 = 1999. 2.4 Ching minh rang phuong trinh 2? — 2y? + 8z = 3 khong c6 nghiém nguyen Ty, 2.5 Tim tat cd céc gid tri nguyén duong 1 < k < 10 sao cho hé phuong tinh 2? + ky? = 2, ka? + y? = t? c6 nghi¢m nguyén duong. 2.6 Chimg minh rang phutong tinh 2? + y? + 2242 +y+z=1 khong c6 nghiem hitu ty. 0.3 Gidi han mién nghiém. Thi va sai Cho lA s6 nguyén. Néu |z| < N véi sO nguyén dutong N cho truéc thi z chi 6 thé nhan hi han gié wi. Cu thé 2 € {-N,-N +1,...,0,1,..., N}. Diéu don gidn nay [a chia kho4 dé gidi nhiéu phuong trinh Diophante, sau dé ding phép thi. Vi du 3.1. (Bai tofn Arnold) Tim tat cd cdc bo ba sé nguyén duong sao cho tich cia hai s6 bat ky cong 1 chia hét cho s6 cdn fai. Vi du 3.2. (Olympic 30/4 nam 1999) Trong mat phing toa do Ozzy cho ba dudng thang c6 hé s6 g6c Ian lugt 1a 1/m, 1/n, 1/p voi m, n, p Ia cdc sO nguyén duong. Tim m,n, p sao cho ba dudng thing dé tao véi truc hoanh ba géc c6 tng s6 do la 45°. BAI TAP 3.1 Tim tt cd cdc nghiém nguyén duong cia phuong winh 1 1 1 1 gtgtptant 3.2 Gidi pitong trinh trong t4p hop cdc s nguyen duong 1,1 ,1 =4=42=51 zy 2 3.3 Tim tft cd céc bd s6 nguyén duong (x,y) sao cho 2? +-1 chia hét cho y, y? +1 chia hét cho 2?, 3.4 (Bulgaria 2001) Tim tit ca cdc bd s6 nguyén duong (a,b,c) sao cho a3 +b3 +6 chia hét cho ab, Be, a. 3.5 (Ailen 2003) Tim tit cd c4c nghiém nguyen cia phuong tinh (m?+n)(n24m) = (m+n)>. : 3.6 (Moldova 2003) Chimg minh ring phuong trinh 1/a + 1/b + 1/¢ + 1/abe = 12/(a+b+c) c6 vO sO nghiém nguyén duong. 3.7 (Rio Plate 2002) Tim tat cA cde cAp s& nguyen duong (a,b) sao cho on ab? +9 1 mot sO nguyen. 3.8 (Litva 2003) Tim tat cd nghiém nguyen cla phuong trinh 3ey — 2 — 2y = 3.9 (IMO 2003) Tim tat c& céc cap sO nguyen duong (a,b) sao cho so A 2ab? — B+ 1 la sé nguyen. 3.10 (Na Uy 2003) Tim tat cd cdc bd sO nguyen (x, y, z) sao cho 23-43 +2332 2003. 3.1L (THT 3/209) Tim tat c& nghiém nguyen (2, y) cla phuong tinh (z+y)(y+2") =(2-y)? 0.4 Phuong phap xuéng thang Fermat da ding phuong phdp nay dé chimg minh phuong trinh xt + y* = z+ k 6 nghiém nguyen duong. Va cling tir day, bang vai dong ngdn ngui trén 1é cu6n Diophantus, Ong da lam dau déu cdc nha ton hoc suét 300 nam qua bang dinh |: ni ting mang tén ong. Co sé cla phuong phdp xuéng thang [a tinh sdp thi ty tot cha N (va N*, tic c4c cia k phién ban NV): Mot tap con khéc rng bat ky clia N déu 6 phan tir nhd Dé chimg minh mot phuong trinh 12 vO nghiém, ta gid sir ngugc lai ring t cdc nghiém nguyén (tu nhién, nguyén duong) cia phuong trinh khdc réng. Ta d mot thé ty tot trén R va gid sit ao JA nghiém nhé nhat (theo thir tr néu trén). bang cdch ndo dé ta xay dung duge nghiém a; nhé hon ap thi ching ta sé di dén thudn, Mau thudn nay ching td diéu gia sir 1A sai va nhu vay phuong trinh 46 ct nghiem. Vi du 1. Chimg minh rang nghiém nguyén duy nhat cita phuong trinh 2? — 3 92° = 0 1a (0, 0,0). Vi du 2. Ching minh ring phuong trinh 2* + y* = z? khong c6 nghiém ne duong. Léi gidi. Gia sit ring phuong trinh dé cho cé nghiém nguyén duong. G (x,y) = d, te lA x = da, y = db, trong d6 (a,b) = 1. Khi d6 at +4 = (z/d”)? sit z = dc, trong dé c € Q, khi d6 at +o 126 Vic € Q,c € N nén theo Dinh ly 6.1, ¢ € N+. Trong ut cd céc nghiém cia phuong trinh (1), chon nghiém c6 c nhd nhat. Ta c6 (a?)? + (67)? = c? trong dé (a,b) = 1, suy ra (a?, 0?) = 1, tite 1a (a?, ?, c) 1a bO ba Pythagore nguyén thiy. Theo dinh ly 6.5, t6n tai céc s6 nguyén duong m,n sao cho a? = m? — n?, 0? = 2mn,c= m?+-n? trong d6 m,n khéc tinh chan 18, m > n va (m,n) = 1, nghia 1a a? = m?—n 18. Gid sit m chan, n 18. Khi dé n?, a? chia 4 du 1, nghia 1a m? = n? +a? chia 4 du 2 mau thudn. Vay m 1é, n chan, ngdai ra (a, n,m) lap thanh bd Pythagore nguyén thiy, do dé tén tai p,q € N+ sao cho a = p?—q?,n = 2pg,m = p* +g”, trong dé p, g khdc tinh chin 18, p > q va (p,q) = 1, ngdai ra b? = 2mn, nghia 1a b? = 4pq(p? +g”), suy ra b= 2h, h € N*, khi d6 W? = pq(p? +q°) Q) Gia sit rang t6n tai s6 nguyén t6 r chia hét pq, p? +9”. Vir chia hét pq nén khong mat téng quat, c6 thé gid sit r chia hét p, khi d6 r chia hét (p? + q*) — p? = gq, suy rar chia hét q, mau thuan vi (p,q) = 1. Vay (pq, p? + g?) = 1, nhw thé, tir (2), theo dinh ly 6.1, ta. 06 pq = s*,p? + q? = t? véi s,t E Nt. Vi pg = s?, (p,q) = 1 nen p=w',g =v? véi u,v € N+, nghia Ja (u?)? + (v2)? = ¢? hay ut + vt = 2, wong d6c=mt+n?>m=p+q =? >t, mau thuin vi cdch chon c. Nhu vay diéu gid sit ban ddu [a sai va ta c6 diéu phai chimg minh. BAI TAP 4.1 Ching minh rang phuong tinh 2, phuong trinh 2? + y? = 2” luor c6 vO s6 nghiém nguyén duong. Lai gidi. Xét sO phitc a = a +bi. Gia sit a =x + yi thi ta 06 Va +P =o" = (Va +b)" Tir dé 2? + y? = (a? +67)”. BAI TAP 5.1 Dya vao hang dang thite {2(32+2y+1)+} -2(4e+3y+2)? = (2¢+1)? ching minh rang phuong trinh x? + (x +1)? = y? c6 vo s6 nghiém n duong. 5.2 Dua vao hing dang thie {2(7y + 12x + 6)}? — 3{2(4y + 7x + 3) + (2y)? — 3(22 + 1)? ching minh rang phuong trinh (2 + 1)? — 2? = y? c6 nghiém nguyén duong. 5.3 Ching minh ring t6n tai v6 s6 cac cap s6 hitu ty duong (:r, y) sao cho 23+ 5.4 (Bulgaria 1999) Chimg minh ring phuong trinh x? + y? + 23 +23 = 1999 sO nghiém nguyen. aw 5.5 Ching minh ring véi moi n > 2, lun ton tai n sO nguyen duong c6 tén ‘ ich. 5.6 (IMO 82) Ching minh ring néu n 1a sO nguyén duong sao cho phuon a3 — 32y? + y® = n c6 nghiém nguyen (2, y), thi phuong trinh nay cé ba nghiem nhu vay. Chimg minh ring phuong trinh dé cho khong c6 1 khi n = 2891. 5.7 Chtmg minh ring phuong tinh (2? +2+1)(y?+y+l) =2 42410 nghiém nguyen. 128 5.8 (THTT 4/187, dy tuyén IMO 92) Chimg minh rang, v6i sO nguyen duong m bat ky sé tén tai v6 s6 cdc cAp sé nguyén (x,y) sao cho 1) 2 vay nguyen 16 cing nhau 2) y chia hét 2? +m; 3) a chia hét y? +m. 5.9 (Saint Peterburg 2003) Ching ring tén tai céc sO nguyén duong a > 1,6 > 1,¢ > 1 sao cho a? — 1 chia hét cho b, b? — 1 chia hét cho c va c® — 1 chia hét cho a vaa+b+c > 2003. 5.10 Ching minh ring phuong inh 2? + 4? + 2? = ayz cé vO sé nghiém nguyen duong. 0.6 Phuong phap sé hoc CA4c tinh chat cia sO nguyen lien quan dén sé nguyén t6, uéc s6 chung, bdi so chung nhu Dinh ly co ban cia s6 hoc, cdc dinh ly Fermat, Euler, Wilson ... déng mt vai td quan trong trong viéc tim kiém ldi gidi cia phuong trinh Diophant. Ching 10i nhac Iai mot sO dinh ly (khong ching minh) va dua ra mot s6 vi du 4p dung. Dinh ly 6.1 (B6 dé) 1) Cho n > 1 1A mot s6 nguyén duong. Néu a,b,c 1a céc s6 nguyén théa man diéu kien a.b = c? va (a,b) = 1 thi a = (a')",b = (b')” vii cdc sO nguyén a’, Y nao 46. 2) Néu a hi ty, a” nguyén véi n nguyén duong nao dé thi a nguyén. 3) Néu a nguyen 7/a hi ty thi 7/a nguyen. Dinh ly 6.2. (Dinh ly nh Fermat) Néu p Ia sO nguyen t6 va a 1 mot s6 nguyen ty ¥ thi a? — a chia hét cho p. Néu (a, p) = 1 thi a?-? = 1 (mod p). Dinh ly 6.3. Winh ly Euler). Néu m [a s6 nguyén duong, (a,m) = 1 thi a") = 1 (mod m), trong dé ¢ 18 Phi-ham Euler - s6 cdc s6 nguyén duong nhd hon m nguyen t6 cing nhau véi m. Dinh ly 6.4. (Dinh ly Wilson). p 1a sO nguyén t6 khi va chi khi (p —1)!+1 chia hét cho p. Dinh ly 6.5. (Dinh ly Fermat-Euler) Néu p = 4k + 1 thi t6n tai cdc s6 nguyen duong a,b sao cho p= a? +b. Dinh ly 6.6. (Mot tinh chat quan trong) Cho p la s6 nguyén t6 dang 4k + 3 va (a, b) = 1. Khi dé a? +b? khong chia hét cho p. Vi du 6.1. (Phuong trinh Pythagore) Tim nghiém téng qut cia phyong x? + y = 2? trong tap hop cdc sé nguyen duong. Gidi bai todn nay ta c6 két qua sau ma ta phat biéu nhu mot dinh Ly: Dinh ly 6.7 Moi nghiém nguyén duong ca phuong trinh 2? + y? = 2? d thé viet duéi dang x = (m? —n?)k, y = 2mnk, z = (m? +n?)k hoac x = 2mni (m?—n?)k, z = (m? +n?)k, trong dé cdc $6 nguyen m,n, k thoa man cdc didu kic 1) (m,n) =1,(2,y) =k 2) cdc s6 m,n khde tinh chan 18 3)m>n>0,k >0. Chitng minh. Gia sit (x,y) = k, khi d6 x = ka, y = kb trong d6 (a,b) = 6 (ka)? + (kb)? = 2? tuong duong véi a? +b? = (z/k)?. Dat z = ke,c €Q,k a? +b? =c?, Doc € Qc? EN nen theo dinh ly 6.1, c EN. Vi (a,b) = 11 nh&t mot trong hai s6 a,b phai lé. Gia sir rang b lé, khi d6 a?, b khi chia 4 du 1 thé c? chia 4 du 2, diéu nay khong thé vi c? chia hét cho 2 ma khong chia hét « Vay 6 chan va nhu thé c? = a? + b? 18. Ta c6 P=(c—a)(c+a) hay ()= c-actl 2 2 Dé dang kiém tra ring (c— a)/2, (c+a)/2 la cdc s6 nguyen nguyen 16 cing Nhu thé, theo dinh ly 6.1, t6n tai c4e s6 nguyen duong m,n sao cho (c ~ a) n®, (cta)/2 = m?, tird6 c = m? +n?,a = m?—n? va b = 2mn, trong dé (m,n) ( Vi du 6.2. (Lebesgue) Giai phuong trinh x?—y? = 7 trong tap hop cdc s6 tt Lai gidi. Néu y 1A sO chan, te 1A y = 2k th @ = 8k? +7 chia 8 du 7 B khong thé. Vay y 18, Tac} \ etisy +8 hay 2? +1=(y+2)(y?—2y44) Néu y chia 4 dir 1 thi y +2 c6 dang 4k +3, Néu y chia 4 du 3 thi y —2 ! 06 dang 4k +3. Vi vay, trong moi trutng hop, vé t4i déu c6 ude dang 4k +3 ; dé 06 uée nguyen t6 dang 4k + 3, diéu nay mau thudn véi dinh ly 6.6. Vi du 6.3. (Euler) Chéng minh ring phuong trinh dey — 2 — y = z? kho nghiém nguyén duong. Huéng dan. Viét phuong trinh dudi dang (42 — 1)(4y—1) = 42? +1 va sit dinh ly 6.6. Vi du 6.4. a) Cho x,y,z la cdc s6 nguyén théa man diéu kien x/y + y/z- nguyén. Chting minh ring «yz la lap phuong cia mot sé nguyén. b) Tim nghiém nguyen cia phuong trinh 2/y + y/z + 2/2 =3. 130 Huéng dan. Viét phuong trinh dudi dang 2?z + y*x + z*y = kayz. Goi pla ude nguyén t6 cia («,y). Hay chimg minh ring yz chia hét cho p*. BAI TAP / 6.1 Gidi phuong trinh trong tap hop cdc sé nguyén duong eqlx -rop2\xy - e003} = 2 6.2 Tim nghiém nguyén cia phuong tinh 2 - 5y?=13. Brihy o> 6.3 Chtg minh rang phuong trinh 2° +3 = 4y(y + 1) khong c6 nghiém nguyén. Lage (yay? xe 6.4 Chimg minh rang phuong trinh «7 +7 = 1998" khong c6 nghiém nguyen duong. 6.5 Ching minh ring néu p la sO nguyen 16, n la s6 nguyen duong thi phuong trinh a(x +1) =p y(y +1) khong cé nghiém nguyén duong. 6.6 Tim tat cd nghi¢m cia phuong trinh ? trong tap hop cdc sé nguyén. w+ (x44)? 6.7 Ching minh ring néu c 1a sO nguyén duong 1é thi phuong tinh a — a = (2c)8 - khong c6 nghiém nguyén duong. 6.8 (Euler) Ching minh rang phuong trinh Awyz—x2—y-t? khong c6 nghiém nguyén duong. 6.9 (Nea 1997) Tim tat cd cdc cap s6 nguyén 16 sao cho p? — g° = (p + q)?. 6.10 (Legendre) Chimg minh rang phuong trinh az? + by? = cz?, trong 46 a,b,c la cdc tham s6 khong cé uéc chinh phuong, doi mot nguyén t6 cing nhau cé nghiém nguyen duong khi va chi khi h@ phuong tinh déng du sau c6 ngbiém (a, 8,7): be (mod a), ca=? (mod b) ab= (mod c) 0.7 Phuong phap hinh hoc Hinh hoc c6 nhiing tng dung rat bat ng& trong viée gidi cdc bai todn s6 hoc. C ta chdc chan cdn nhé bai todn cta IMO 42 “Cho cdc s6 nguyen duong a,b,c, a>b>c>d>0. Gid st act+bd =(b+d+a-—c)(b+d-—a+c). Ching ring ab + cd khong phai la sO nguyen 16” d6 duge gidi hét sttc dn trong bing ... ly ham s6 cos va dinh ly Ptolémé, Duéi day, ta sé xét hai vi du tmg dung cia hin trong s6 hoc véi hai phutong ph4p tiép can khéc nhau. Phuong phép thi nat dugc goi 1A phyong phdp cét tuyén, sir dung ¥ tuén, hinh hoc gidi tich vio viée nghién ctu cdc diém nguyén va diém hi ty tren ¢ cong, Chinh huéng di nay d6 dan dén khdi nigm duéng cong elliptic, mot trong 1 vien gach co ban dat nén méng cho viec chimg minh dinh ly 1én Fermat. & day, « ta chi gidi han & mot vi du nho. Vi du 1. Tim tat cd cdc nghiém nguyén khdc (0,0, 0) céa phuong trinh a? + 2y? = 327 Léi gidi. Chia hay vé cia phuong trinh cho 2”, ta duge phuong trinh Greys Dat u = 2/z,v = y/z, ta duoc phuong trinh w +2? =3 trong d6 u,v hitu ty. Bai todn quy vé vie tim tt cd cdc nghiém hau ty cia (2). © tim t4t cd cdc diém hiu ty nim wén dudng cong (B) : u? + 2v? = 3. Cho y ra 1) la mot diém haw ty cia (B). Néu (ug, v9) [A mot diém hu ty khéc (1, 1) thi: thing qua (1, 1) va (uo, vo) sé c6 hé s6 géc hitu ty. Mat khéc, néu y = k(x — Ta dudng thang qua (1, 1) véi he s6 géc k hitu ty thi, 4p dung dinh ly Viét cho p trinh hoanh d6 giao diém, giao diém thit hai cha dutng thing trén véi (C) cing do hitu ty. Tinh ton truc tigp ta c6 toa do cia diém nay la __ 2k? —4k-1 7 —2k? —2k+1 US oR 4? 2 +1 Tir day ta cing tim duge nghiém téng quét cla (1). Vi du voi k = —1 to u = 5/3,v = 1/3 va ta c6 nghiém (5, 1,3) cita (1). Phuong php thit hai 1 phuong phép diém nguyen, Iuéi nguyen. Rat nhiéu c sau sdc cha sé hoc duge chtg minh bang céch tinh sé diém nguyén trong mot Chang han Luat thuan nghich binh phuong cho k¥ higu Legendre dé duge ching nhu vay. DuGi day, ching ta xét mot tmg dung khac ca phuong phdp ludi ngu Vi du 2, Bé dé Minkowsky va dinh ly Minkowsky Dinh ly Minkowsky Ia mot vi du réit thé vi vé tmg dung cia hinh hoc tr thuyét s6. Chung ta bat dau ti mot két qu rat don gidn nhung hdu fch. 132 Bé dé 7.1 Tren mat phing cho hinh F cé dién tich l6n hon 1. Khi dé t6n tai hai os diém A, B thuoc F, sao cho vécto AB c6 toa do nguyen. Chitng minh. Ludi nguyen cat hinh G thanh céc mdu nhd. Chéng céc mau nay len nhau, do t6ng dién tich cla céc mu 16n hon 1, nen cé ft nhat hai mdu 6 diém chung. Goi A,B 1a hai diém nguyén thuy tmg véi diém chung nay thi A, B [a hai diém can tim. Bé dé 7.2 (Bé dé Minkowsky) Trén mat phdng cho hinh Idi F nhn g6c toa do lam tam d6i ximg va c6 dién tich Ién hon 4. Khi d6 n6 chtta mot diém nguyén khéc gc toa do. Chitng minh. Xét phép vi tu tam O, ty sO 1/2, bién F thanh G. Do G c6 dign tich Ién hon 1 nén theo bé dé 1, tén tai hai diém A, B thudc G sao cho véeto AB c6 toa do nguyén. Goi A’ 1& diém d6i xtmg voi A qua O. Do hinh G déi xtng qua g6c toa do nen A’ thude G. Do G I6i nen trung diém M cia A’B thudc G. Goi N 1a diém d6i xtmg cba O qua M thi N thuge F va ON = AB, suy ra N i& diém nguyen khéc O (dpem). Dinh ly 7.3 inh ly Minkowsky) Cho a,b,c IA cdc s6 nguyen, trong dé a > 0 va ac — b? = 1. Khi 6 phuong tinh az? + 2bry + cy? = 1 c6 nghiém nguyén. BAI-TAP 7.1 Tim tat cd cde cap (x, y) ce sO hitu ty dyong sao cho 2? + 3y? = 1. “7.2 Ching minh rang mot dutng coag bac hai bat ky hodc khong chia diém hou ty nao, hoac chita v6 sé diém hiu ty. 7.3 Hay tim vi du mot dutmg cong bac hai khong chifa diém hi ty ndo. 7.4 Chimg minh ring néu D 1& s6 nguyén khong chinh phuong thi phuong trink a? — Dy* = 1 luon cé nghiém nguyén duong. 7.5 Cho p,q 18 cdc s6 nguyen duong nguyen 6 cing nhau. Bing cdch dém cdc diém nguyén, hay ching minh cong thie El+iPl+ 0.8 Phuong phap dai sé (phuong phap gien) p= Ny ~ ®-1)@-1) P 2 Néu tir mot nghiém cia phuong trinh dé cho ta c6 quy tac dé xay dung ra mét nghiém méi thi quy téc d6 chinh 1a gien. phuong phép gien 1a phuong phap dua vao gien dé tim ui ca cdc nghiém cha phuong trinh dé cho ti cdc nghiém co sé. Dé tim cic nghiém co sé, ta 4p dung bude Iii, tte 1a quy tic nguoc cia quy tic tién ndi tren. Minh hoa t6t nbat cho ¥ tuéng nay 1a phuong tinh Pell va phuong trinh Markov. Ta bat ddu bang phuong trinh Pell. Phuong trinh Pell 6 dién la phuong trinh dang, 2? ~ Dy? = 1 trong dé £ nguyen duong khong chinh phuong, Néu D = k? thi te phan tich (a —ky)(a+ky ta suy ra phuong trinh d6 cho cé cdc nghiém nguyén duy nat 1a (1, 0). Trong | hop D bat ky thi (1,0) cting IA nghiém cia phuong trinh Pell. Ta goi nghiém nghiém tim thudng Voi s6 nguyen duong D khong chink phuong cho truée, dat S = {(x (N*+)?|a? — Dy? = 1} 1a tap hop tat cd cdc nghiém nguyen duong cia p tinh Pell 2 —Dy=1 Ta cé dinh ly quan trong sau Dinh ly 8.1 Néu D la s6 nguyen duong khong chinh phuong thi S # 0, phuong trinh (1) cé nghiém khong tém thudng. Chimg minh dinh ly nay khé phic tap, dua vao ly thuyét lién phan s6 hoae p phép hinh hoc. Tuy nhién, vé mat tmg dung (trong céc bai toan phé thong), d nay la khong thuyc su céin thiét vi v6i D cho trudc, ta cé thé tim ra mot nghiém n duong cia (1) bing phuong phép thir vA sai. Ta bé qua dinh ly nay va chuyé: dinh ly mo 18 tat cA c&e nghiem cia (1) khi biét nghiém co sé. V6i (x,y), (2',y’) € S ta cé néu x > a’ thiy > y. Dodé cé thé dinh (cy) > (2'y’) hay 1a & > o!. Voi thir ty nay, $ 1a mor tap sdp thé ty tt. Goi la phn tir nhé nhat cia S theo thé ty tren. Ta goi (a, 6) 1A nghiém co sé cia ( Dinh ly 8.2 Néu (a,b) 1A nghiém co sé cia (1) va (x,y) IA mot nghiém n duong tly ¥ cila (1) thi tén tai sO nguyén duong n sao cho z+yVD = (a+b va tit dé moi nghiém ca (1) déu cé thé tim duge béi cong thitc (a+ bvD)" + (a — bVD)" y-@ +b/D)" — (a-bVD)" 2 oUF 2/D Chitng minh. Nban xét ring néu (2, y) IA nghiém cia (1) thi 2! = az — 1 y’ = ay — be cling 18 nghiem cita (1) (c6 thé khong nguyen duong) Truée het, do VD vo ty nen néu x + yVD = (a + bVD)" th 2 ~ yy (a-byD)? va tir d6 2? ~ Dy? = (a+ bVD)"(a — bYD)" = (a? — Db?)" = ra (a, y) la nghiém cia (1) va ta 6 cong thie nhu trén. Tiép theo, gid sit khong phai nghiém ndo cia (1) cling c6 dang (2). Goi (a 1a nghiem nhé nhat khong c6 dang (2) thi rd ring 2¥ > a,y* > b. Theo nhan ; 2! =ac"—Dbys y/ =ayx—bex 1a nghiém cita (1). 134 Dé dang kiém tra duge rang 1) xx > 2’ > 0 va 2) yx > y' > 0. Tit dé, do tinh nhd nhft cha (2s, y+), ton tai n nguyen dutong sao cho 2’ +y/VD = (a+bVD)". Gidi he (3) v6i dn 1A (a, yx), ta duge (cht ¥ a? — Db? = 1) ax = ax! + Dby', yx = ay’ +2’. Tu dé ae ty+VD = ax! + Dby' + (ay! + b2')VD = (a + bVD)(a' + y'VD) = (a + bVD mau thudn! Vay diéu gid sir Ia sai va (2) Ia tat cA cdc nghi¢m cha (1). Tigp theo, ta xét phuong trinh dang Pell, tic Ia phuong trinh dang 2 — Dy =k (4) trong dé D khong chinh phuong va k ¢ {0, 1}. Ta c6 mot s6 nhan xét sau (i) Khong phai v6i cip D,k nao phuong trinh (4) cing c6 nghiém. Ching han, phuong trinh x? — 3y? = —1. (ii) Néu phuong tinh (4) c6 nghiém nguyén duong thi n6 c6 vo sO nghiém nguyen duong. Ly do: néu (x,y) 1a nghiém cia (4) thi : a! =ac+Dby y! =aytbr cling 1a nghiém ciia (4), trong dé (a,b) 1a nghiém co s& cha phuong trinh. Nhu thutng le, ta dat S = {(x,y) € (N*)?|2?— Dy? = ky va goi (a,b) 1a nghiem co s& cha phuong tinh Pell tuong tng 2? — Dy” = 1. Nghiém (29, yo) thudc S duoc goi 1A nghiém co sé cha (4) néu khong tén tai (2’,y!) € S sao cho = a2! + Dby y= ay! +2" Goi So [a tap hop tat cA cdc nghiém co sd. Ta c6 dinh ly quan trong sau: Dinh ly 8.3 V6i moi D,k ta c6 |$0| < 00. Chitng minh. N&u So = @ thi |So| = 0 < 00. Tiép theo gid sit Sp # 0. Goi (x,y) 1a mot nghiém co sé ndo dé cha (4). Xét he az! + Dby!'=2 ay'+br'=y c6nghiém 2! = ax—Dby, y’ = ay—bx. Dé dang ching minh duge (a')?— D(y’)? = 1. Vi (x, y) € So nén theo dinh nghia (2’,y’) ¢ S. Diéu nay xay ra khi va chi khi 2’ < 0 hoac y’ < 0 nghia la az < Dby hodc ay < ba, tuong duong voi x? < —kDb? (5) hoac y? < kb? (6). Néu (5) xdy ra thi ta c6 dénh gid y? = (a? — k)/D < —k(Db* + 1)/D. Néu (6) xy ra thi ta c6 x? = Dy? +k < Dkb? + D. Trong ca hai tring hop, ta c6 |S] < 00. Cu6i cing, chit ¥ ring tir mot nghiém (cr, y) bat ky cla (4) khong thude So cAch di nguoc xudng bang cOng thite x’ = ax — Dby,7/ = ay — be ta luon cé dén mot nghiém co so cia (4). Nhu vay, véi dinh Ly trén, phuong trinh dang dugc gidi quyét hdan toan. Dudi day chting ta xem xét mot vi du: Vi du 8.1 Tim tat cd cde nghiém nguyén duong cla phuong trinh 2? ~ 5y? qd). Lai gidi. Bang phép thi tuan ty, ta tim duge nghiém co s8 cia phuon; a2? — Sy? = 118 (9,4). Theo phép ching minh dinh ly 8.3, nghiem co si ciia ( man a? 4.547, y? < (4.5.42 +4)/5 Tir day suy ra x < 17, y <9. Ding phép thir tuan tu, ta tim duge hai nghiém 1a (1,1) va (11,5). Tu hai nghiém nay, bing cong thitc a =9r+20y, y =4r+9y ta tim duge tat cd cdc nghiém cta (1). Vi phép gidi phuong trinh dang Pell, trén thuc té ta d6 6 thé gidi tét phuong trinh Diophant bac hai, tic 18 phuong trinh dang + ax? + bry + cy? +de+eyt+ f=0 Dua vao ly thuyét dutmg cong bac hai, ta c6 thé dua phuong trinh trén \ trong céc dang chinh sau (i) Dang ellip: ax? + by? = c (a,b,c > 0), c6 hitu han nghiém, gidi bang f phép thir va. sai (ii) Dang parabol: ax? + by +c, gidi bang déng du bac hai (iii) Dang hypebol: ax? — by? c, phuong trinh dang Pell Ngoai ra cdn c6 cdc dang suy biét nhu hai dudng thing cét nhau, hai dudng song song, ellip o ... Dui day, ta xét mot vi du 4p dung: Vi du 8.2. Tim tt cd cdc cap s6 nguyén duong (m,n) thoa man phuong t m(m +1) +n(n +1) = 3mn Léi gidi. Xét phuong trinh d6 cho nhu phuong trinh bac hai theo m. m — (3n~1)m+n(n+1) =0 Phuong trinh nay cé nghiém nguyen duong khi va chi khi A 1a s6 chinh phuo a (3n — 1)? —dn(n +1) =y? hay a y? —5(n — 1)? = -4 136 ta thu duge phuong trinh dang Pell ma ta d6 biét céch giai. Phuong trinh Markov cé dién Ia phuong trinh dang at taht +02 = keyaeen a & day n va k 1A cfc tham sO nguyen duong. Trudng hop ritng khi n = & = 3 thi phuong trinh vty +2? = Bayz @) duge nghién citu chi tiét trong bai béo cia A.Amarkov vé dang toan phuong duong dang 6 Bao céo VHL KH Lien X6 nam 1951; “Dang Markov”, lién quan chat ché dén phuong trinh dang (2) dugc sir dung trong ly thuyét xdp xi cdc s6 vo ty bing cic sé hau ty. Dau tién, ta chi y¥ dén mot tinh chat thé vi cla phuong trinh Markov. Néu phuong tinh (1) c6 mot nghiém thi né sé c6 rat nhiéu nghiém va c6 thé tao ra cdc nghiém d6 bing cach sau day. Ta sé coi mot bién, ching han xn, 1a “dn s6”, cdn tit cd cdc bién kh4c nhu céc tham s6. Khi dé, vi phuong trinh aw? — kaytyt +a} +-+++a2_)=0 1a phuong trinh bac hai theo x va c6 nghiém x = an, nén n6 c6 nghiém nguyen the hai 2!, = u; theo dinh ly Viet ta c6 = kay tp_1 — Bp = (a3 + + 27_1) fon QB) Chi y rang u key..tn-1 (4) Qué trinh nay c6 thé thuc hién vi moi bién sO x; trong vai trd cia tm. Nhumg chi d6i véi mot bign - bin 1m nhat 18 c6 thé xdy ra (4) va ta thu duge nghi¢m méi (x1, 22, ,@),) “nhd hon” nghiém ci (tint ty theo téng céc bién); nhu vay, theo da s6 1 cc nghiém tang len va ta c6 cay nghiem. Tiép theo, trit nhimg trudng hgp dac biét, ta sé gid sir rang 7) < 2p S++: < Mm. Ta sé n6i nghiém (21,22, ,7,) 1 nghiém g6c (nghi¢m co sd), néu ae4-.-ta2_,>a—n? hay wy 2, (21,22, ,2n) anghiém g6c, ngoai ra, «1 < a2 < ++ < ap. Khi d6 Chiing minh. key @n—203_4 = key Tp—2Ip—-1Iy Sapte tap) +ah = Aah + +241) =(n— Lary Bé dé 8.6 Nou oy < 22 < ++» < aq A céc sO nguyen duong bat ky tho mi kien 1 < a2 2k —3 khi van > 4k —6 khin=3,n=4, Tir c4c dinh Ly va bé dé trén, véi n cho trutéc, viée thm tit cd cdc gid tri k sao c6 nghiém thyc hién dugc dé dang. Hon nita, phuong phdp gien duoc sit dung 6 t thé 4p dung cho c4c phuong trinh dang tung tu, vi du phuong trinh (2+y+z)? = Cuéi cling IA mot vi dy khée vé tmg dung cia gien Vi du 8.3 (Iran 2001) Gia sit z, y, z 14 cdc s6 nguyén duong théa min did xy = 27 +1. Ching minh ring t6n tai cdc s6 a,b,c va d sao cho x = a? +b? Cid vaz=actbd. BAI TAP 8.1 (Ailen 1995) Tim tat c& cdc s6 nguyén a sao cho phyong trinh «?+-any+y c6 v6 sO nghiém nguyén phan biét x,y. 8.2 (Pai Loan 1998) Tén tai hay khong nghiém cia phuong trinh ety tte te? = cyzur — 65 trong tap hgp cdc sé nguyén Ién hon 1998? 8. io (Vigt Nam 2002) Tim tat c& céc sO nguyen duong n sao cho phuong oty+z+t=ny/zyzt c6é nghiém nguyén duong. 8.4 (Ba Lan 2002) Tim tit cA c4c cap sO nguyen duong 2, y thoa man phuon; (x+y)? — (ay)? = 1. 8.5 (My 2002) Tim tat cd cdc cap sap thit ty cdc sé nguyen duong (m,n) se mn —1 chia hét m? +n?, 8.6 (Viet Nam 2002, vong 2) Ching minh ring t6n tai sO nguyén m > 2002 s6 nguyén duong phan biét a1, a2, ...,am sao cho []e-ae 138 18 s6 chinh phuong. 8. cS (Viet Nam 2002, vong 2) Tim tit cd céc da thie p(z) vi he so nguyen sao cho da thite a(x) = (2? + 6a + 10)(p(a))? — 1a binh phuong cla mot da thie vi hé sé nguyén. 8.8 (THIT 6/181) V6i gid tri nguyen duong nao cia p thi phuong trinh 2? +y?41 = pry c6 nghiém nguyen duong? 8.9 (THIT 4/202) Cho ba sO nguyén a,b,c;a > 0,ac — 6? = pipepm trong d6 P1,P2),Pm Ia cdc sO nguyén t6 khéc nhau. Goi M(n) Ia 6 céc cap sO nguyén (x,y) théa man ax? + Qbry + cy? =n 8.10 (Dé dé nghi IMO 95) Tim s6 nguyén duong n nhé nhat sao cho 19n +1 va 95n+1 déu [a céc s6 chinh phyong. BL Tam giéc vi canh 3,4,5 va tam giéc véi canh 13, 14,15 ¢6 cdc canh Ia cdc s6 nguyen lién tiép va c6 dién tich nguyen. Hay tim uit cd cdc tam gidc c6 tinh chat nhu vay. 8.12 Ching minh rang néu cd 3n +1 va 4n +1 déu 1a céc sO chinh phuong thi n chia hét cho 56. 8.13 Trong céc hang cia tam gide Pascal, hay tim hang c6 chita ba s6 hang lién tiép lap thanh mét cAp s6 cong. 8.14 (My 1986) Tim s6 nguyen duong n > 1 nhé nhat sao cho trung binh binh phuong cia n s6 nguyén duong dau tién 14 mot s6 nguyén. . 8.15 Néu a,b, = (a?-+8?)/(ab+1) Ia cdc s6 nguyen duong thi q Ia s6 chinh phuong. 8.16 (MOCP 03) Tim tat c& gid trim sao cho phuong trinh («+ y + 2)? = nayz c6 nghiém nguyen duong. 8.17 (PINK 03). Tim tat cd cdc s6 nguyén duong k sao cho phuong trinh 2? — (k? — 4)y? = 24. 8.18 Ching minh ring phuong trinh (k? — 4)x? — y? = 1 khong c6 nghiém nguyen voi moi k > 3. 8.19 (Mathlinks) Cho A Ia ap hop hitu han cée sO nguyen duong. Ching minh rang tén tai tap hop hitu han c4c s6 nguyén duong B sao cho A C B va Tees? = Dees”. TAI LIEU THAM KHAO 1, Jean-Marie Monier. Dai s6' I - Gido trinh todn t@p 5, NXBGD-Dunod 1999. Rv » + w ble eos = 17. Sy 18. 19. 20. 3 . Ha Huy Khodi - Pham Huy Dién. Sé’hoc thudt todn, NXB DHQG HN 2 . Lé Hai Chau, Cde bai thi hoc sinh gidi Téan PTTH toan quéc, NX 1994, Nguyén Sinh Nguyén, Nguyén Van Nho, Le Hoanh Pho. Tuyén tap ede du tuyén Olympic Téan hoc Quéé té’ 1991-2001, NXBGD 2003. . Nguyén Van Nho. Olympic Todn hoc chau é - Thai Bink Duong 1 2002, NXBGD 2003. Tap thé tacgida. Tuyén tap 5 ném Tap chi Todn hoc va Tuéi tré, NX 2003. Arthur Engel. Problem Solving Strategies, Springer 1998 . Goeoge Polya. Gabor Szego, Problems and Theorems in Analysi. Springer 1976 Harvey Cohn, Advanced Number Theory, Dover Publications 1980 . Titu Andreescu, Juming Feng. Mathematical Olympiads 1999-2000: O Problems from Around the World, MMA 2000. . Titu Andreescu, Juming Feng, Hojoo Lee. Mathematical Olympiads 2 2002: Olympiads Problems from Around the World, MMA 2002. . Titu Andreescu Razvan Gelca. Mathematical Olympiads Challenge. Bi 2000. . Walter Mientka others. Mathematical Olympiads 1996-1997: Olymg Problems from Around the World, MMA 1997. . Walter Mientka others. Mathematical Olympiads 1997-1998: Olymz Problems from Around the World, MMA 1998. . Bugaenko B.O. Phuong trinh Pell (ting Nga), Matxcova 2001. Badzylev DE. Phuong trinh Diophdng (titng Nga), Minxk 1999. Gelphond A.O. Gidi phuwong trinh nghiém nguyén (tiéng Nga), Nauka Serpinski B. Vé gidi phwong trinh nghiém nguyén (eng Nga), FML Serpinski B. 250 bai todn so céip vé ly thuyét s6’(titng Nga), FML 1% Cfc tap chi Kvant, AMM, Todn Hoe Tuéi tré, Todn hoc trong nha trudng . Tai ligu trén Internet, dac biét la website www.mecme.ru va www.mathlir 140 PHUONG PHAP GIAI BAI TOAN CHIA HET Dang Huy Ruan Khi c6 s6 nguyen a va sO tu nhién b mot trong nhing cau héi hién nhién duoc dat ra la: Ligu a c6 chia hét cho b khong? C6 nhiéu phuong php gidi bai todn chia hét. Song viéc van dung phyong ph4p lai phai phy thudc vao dang bai toan, Dudi day xin trinh bay mét trong céc phutong ph4p d6: phuong phdp ding phép chia 6 du, phuong phép déng du, phutong phép ding tinh tun hoan Kkhi nang lén liy thita, phuong phép quy nap va sit dung tiéu chudn chia hét. 0.1 Cac sé nguyén va cdc phép tinh sé nguyén TAp hop cdc s6 nguyén gém cdc sé tu nhién 1, 2,3, s6 khong 0 va cdc s6 nguyén 4m —1,—2,—3 Trong t4p hop dé lun lon thyc hién duge phép cong va phép tix, N6i cach khéc, néu m va n IA cdc $6 nguyen, thi tong m+n cia ching ciing 1a sO nguyén. Hon nifa, véi hai sé nguyen m,n tuy y t6n tai duy nhat mot s6 z thoa man phuong tinh ntc=m $6 d6 due goi 1a higu cha céc s6 m va n déng thdi ky higu bing m—n. Hitu hai s6 nguyén bat ky cing [a s6 nguyen. Trong tap hgp c4c sO nguyen ciing luon luon thc hién dugc phép nhan, nghia 1a, néu m va n 1A cdc s6 nguyen, thi tich m.n cia ching cling Ia sO nguyen. Tuy vay, phép chia (1a phép tinh ngugc cia phép nhan) khong phi khi n&o cling thuc hién duoc trong tap hop c4c s6 nguyen. Két qua cha phép chia sO a cho s6 b #0 IA s6 & duoc ky higu bing a:b hoac $ thod man phuong trink br =a SO x d6 t6n tai va duy nhét. Song két qua clia phép chia mot s6 nguyén cho mot sO nguyen khdc khong phai khi nao cting 1a mt sO nguyén. Thi du, c4c thuong 3: 2,6: 5,(—50) : 7, (—60) : (~21) khong phai IA céc s6 nguyén. Diéu dé c6 ngbia la phép chia khong phai luén lu6n thyc hién duge trong tap hop cc sO nguyén. Thuong cla phép chia sO nguyén a cho s6 nguyen b # 0 c6 thé khong thudc tap hgp cdc s6 nguyén; cdn chinh trong tap hop cdc sé nguyén khong tim duge mot s6 nao dé ta c6 thé goi la thuong cia phép chia a cho b. ‘Tat nhién, ta cing gap cdc trusng hop: Thuong cia phép chia mot sO nguyén cho sO nguyén khdc ciing lai 1A mot sO nguyen, chang han 8: (—2) = 4,48: 12 =4,(-6):6 = -1 Dinh nghia. Neu a va b (b #0) 1a cde sO nguyen, ma thuong a : b cling nguyen, thi ta néi ring sO a chia hét cho s6'b va vigt a: b. Cling c6 thé néi céch khéc: S6 nguyen a chia hét cho s6 nguyen b # 0, né tai mdt sO nguyén k, sao cho a = kb, Dinh nghia vé chia hét trén day sé thudng sau nay. Vi chi néi dén cdc s6 nguyen, nen dé ngin gon ta sé viét “s6”, nhung ludn hiéu 1a s6 nguyén. Xin nhdn manh ring, chi cé thé néi vé thuong a : b khi b ‘Trung hop b = 0 thuong a: b khong x4c dinh, nghia 1A biéu thitc a: 0 hay # } c6 nghia. Tém lai khong thé chia cho s6 khong. Nguyc lai, khi a = 0 (va voi moi b # 0) thuong a: b x4c dinh (va bing kho $= oKnin 40 Vi trong trutmg hop nay s6 khong 1a sO nguyén, nén né chia hét cho moi s6 n; khdc khong (ngoai ra thuong bang khong). 0.2 Cc dinh ly vé chia hét Dinh Iy 1. Néu céc 86 ay, a2, ..., an chia hét cho m, thi téng a; +a2 +--+ hét cho m. ‘That vay, vi aj(1 0. Ta n6i ring s6 @ chia cho s6 6 c6 thuon va s6 du Ia r, néu a c6 thé bigu dién bing ding thiic a = bg +r, trong dé 0 < 03.2. Su tén tai va duy nhat cia phép chia cé du C6 hai vin dé duge dat ra d6i véi dinh ngnia phép chia c6 dur IA 1. Ligu c6 thé luén Juon thyc hién duge phép chia c6 du hay khong? Néi Khdc, néu cho s6 nguyen a va s6 ty nhién 6, thi ludn ludn c6 thé chon duc sO nguyén g var, dé 0 a (do |al 1). Dik ching t6 rang trong day (1) c6 mot s6 bé hon hay bing a, cdn s6 tiép theo 16n | Ky higu s6 nay Ia qb. Khi d6 s6 tiép theo 1a (q+ 1)b da Ién hon a. gb r2. Khi d6 ry — r2 > 0. Mat khéc Ty — 72 S71 2, nghia Ia tir t phép déng dur tay ¥ a;=b; (mod m), az=b;, (mod m),...,a;=0; (mod m),...,a,=b: (mod: da suy ra duge 4 09...0;...0; = byby...bj...b, (mod m) 0) Xét t+1 phép déng du bat ky, a) = b) (mod m), az = be (mod m),...,ar = by (mod m), ..., 4441 = bt41 (mod m). Khi d6, theo gid thiét quy nap tt phép déng du dau d& c6 a 09...0; = byby...b, (mod m) Ky higu, aya2...a; bang Ay, con bybz...b bing By. Khi d6, theo dinh ly 5, higu A chia hét cho m, nen t6n tai sO nguyen 1, dé A, — By = L.m. Do at41 = b¢41 (mod m), nén, theo dinh ly 5, a¢41 — bey1 chia hét cho m vay t6n tai s6 nguyén k, dé ay41 — biz1 = kom. Xét hiéu Atderi — Bebrsa = Araesi — Arbesa + Arbors — Bedesi = Ar(aee1 ~ beer) + bes1(Ae — Br) = Ap km + by.Lm = (Ack + bep1m nen @1a2....d¢0¢41 — brb2...bibe41 = Avdey1 — Bybt41 chia hét cho m. Do dé. dink Ly 5, thi 0109... = byb2...bn (mod m) Tir dinh ly 7 suy ra cdc hé qua sau: Hé qua 2. Céc phép déng du c6 thé nang len ldy thita, nghia 1, néu a (mod m) thi véi moi s6 nguyen khong am n déu c6 a” = 6" (mod m). H6 qua 3. Gia sit P(x) 1a da thie tay ¥ véi he sO nguyen P(x) = to + tia + to? ++++. + tyo™ Khi d6 néu a = 6 (mod m), thi P(a) = to +tya-+ tea? +--+ + tna” = ty +t1b + tod? +--+ tnd” = P(b) ( 0.5.2 Phuong phap déng du Ta sé ta dung c4c tinh chat cba phép déng du dé gidi bai todn chia hét. Vi du 4. Chimg minh ring s6 5°°°* + 23 chia hét cho 24. i, ‘Ta sé ching minh khang dinh tag quadt rang v6i moi sO tu nhién a" +23 chia hét cho 24. That vay, do 58” et = 588" ya 25 = 1 (mo nén 25¢8""* = 1 (mod 24). Bdi vay, 58" = 1 (mod 24). Do d6 a 4235 =0 (mod 24) Nen 5°” + 23 chia hét cho 24. Vi du 5. Chimg minh ring véi moi s6 ty nhien n, s6 122741 4 11"? chi cho 133. 152 Gidi. Ta 6 122"+1 = 12.12% = 12{(12)?}" = 12.144". Vi 144 = 11 (mod 133), nen 144" = 11” (mod 133). Nhan cd hai ve v6i 12 ta c6 12.144" = 12.11" Boi vay 12?"+1 = 12.11" (mod 133) @ Mat khéc, unt? = (117)" = 121" Do 121 =-12 (mod 133) Nen 121.11" = -12.11" (mod 1)33 Boi vay 11"? = —12.11" (mod 133) Q) Cong vé véi vé cc phép déng du (1) va (2) duge 127"+1 411"42=9 (mod 133) Do 6 12?"+1 4.11"+? chia hét cho 133. Vi du 6. Ching minh ring, néu a? +b? +c? chia hét cho 9, thi ft nhat mot trong c&c higu a? — b, a? — c, b? — c? chia hét cho 9. Gidi.Khi chia s6 nguyen tuy ¥ n cho 9 nhan dugc mot trong céc sO du 0, 1, 2,3, 4, 5,6 Béi vay, Néu n = 0 (mod 9), thi n? = 0 (mod 9) Néu n = 1 (mod 9), thin? = 1 (mod 9) N€u n = 2 (mod 9), thi n? = 4 (mod 9) Néu n = 8 (mod 9), thin? = 9 =0 (mod 9) Néu n = 4 (mod 9), thin? = 16 = 7 (mod 9) Néu n= 5 (mod 9), thin? = 25 = 7 (mod 9) Néu n= 6 (mod 9), thi n? = 36 = 0 (mod 9) Néu n= 7 (mod 9), thin? = 49 = 4 (mod 9) Né& n= 8 (mod 9), thi n? = 64=1 (mod 9) Vay di véi sO nguyen n nao di chang nia, s6 n? citing chi c6 thé cé mor cae $6 du 0, 1,4,7 khi chia cho 9. Ding ri,r2,73 dé ky higu cde sO du tuong 3, ba, C2 Khi chia cho 9. Khi d6 a2=7r (mod 9), be=rz (mod 9), cp=r3 (mod 9) Cong vé véi vé cdc phép déng dur trén ta duye | +04 =r+r2+r3 (mod 9) Vi a? +b? +c? chia hét cho 9, nen a+? 4+c?=0 (mod 9) ri+r+r3=0 (mod 9) Vi méi sO ry, 72,73 chi c6 thé nhan cdc gid tri 0,1,4,7, nén ry + rz +79 chi: chia hét cho 9 trong c4c trudng hgp sau Dn =r =r3=0, 2) Mot trong cfc sO r1,7r2,73 bang 1 va hai s6 cdn lai bing 4, 3) Mot trong cdc sO ry, 2,73 bang 7, hai s6 con lai bing 1, 4) Mot trong céc sO r,72,73 bang 4, hai sO cdn lai bing 7, Trong moi trudng hop déu cé it nhdt hai trong cdc s6 r1,r2,73 bang nhau. Dié 6 nghia 1a it nhat hai trong cdc s6 a2, ba, cz 06 cing 6 du khi chia cho 9, nér nhat mot trong céc higu a? — b?, a? — c?, b? — c? chia hét cho 9. BAI TAP 11. Véi moi sO nguyen khong am n hay chimg minh ring a) 627 4 "+2 4 3” chia hét cho 11. b) 6.2543 + 57.3"+1 chia hét cho 17 c) 52rt1 gn+2 4 gn42 92n+1 chia hét cho 19. 12 Ching minh ring khong t6n tai mot sO n nao dé céc s6 3n — 1, 5n +2, 2, 7n +3, 7n —2 I cc sO chinh phuong. 13 Chéng minh ring, véi cdc sO tenhién k,n tay 9 s6 124-1 4.2714... 4.(2r chia hét cho 2n + 1. 14 Chimg minh ring sO 100...001 (v6i s6 s6 0 chan) chia hét cho 11. 154 0.6 Phuong phép sit dung tinh tuan hoan khi nang lén lity thira 0.6.1 Su tudn hoan ciia cdc sé du khi nang lén lity thita Xét céc dy thi lien tiép cia s6 2; 23, 27, 23, 24, 25, 28, 27, 28, 29... va tim xem Khi chia c4c lu thita nay cho 5 nhan duge cdc loai s6 du nao? Néu tim true tiép thi khé phitc tap va ldy thira cang 16n, thi c’ng khé khan. Song, nhd viée nang len ly thita hai vé cia phép déng dur cé6 thé tim cc s6 du cita lily thita mot cdch dé dang. That vay, ta c6 2=2,27=4,29=8=3 (mod5),24=16=1 (mod 5) qa) Dé tim sO du khi chia 25 cho 5 ta nhan cd hai vé phép déng du (1) véi 2 sé duge 2 =2 (mod 5) 2 (mod 5) 27=4.2=8=3 (mod 5) Ti€p theo f= .2=6=1 (mod 5) Viet cdc két qua vo hai hang. Hang trén ghi céc luy thita, hang du6i ghi s6 du tuong img hi chia cc ly thita nay cho 5. 2) 22 23 24 25 26 Q7 28 2 gio Qu 24 3 12 43 12 4 3 Hang thit hai cho ta thay ring cdc sO du lap lai mot cdch tuén hoan: sau 4 sO du 2,4, 3,1 lai lap lai theo diing thé tu trén va cit tip tuc lap lai theo thé nr tren v.v Xét cdc s6 du cia phép chia Idy thira cla 3 cho 7 Ta c6 3=3 3?=9=2 (mod 7) Nhan phép déng du trén véi 3, sau lai nhan phép déng du nhan duge véi 3, ta duge 3°=6 (mod 7) 34=6x3=4 (mod 7) 3°=4x3=5 (mod7) 38=5x3=1 (mod 7) Tig tuc tinh toén nhu trén sé duge hai hang sau 3 3? 33 34 35 36 37 38 39 gl0 gi giz 3.2 6 4 5 13 2 6 4 5 «1 (dui mdi ly thiza [a s6 du cia né khi chia cho 7) Xét cdc s6 du Khi chia luf thita cia 5 cho 16 ta dugc hai hang tuong tng: 5 on 3, 5. 59 is no Bee at 8 bo old Sat 5.9 Bl 56 BL (du6i mi ldy thita 1a s6 du cia né khi chia cho 16) Nhin vao hang hai ta dé dang nhan thay cdc sO dur lap Iai sau 4 36 du 5,9, r6i lai Lap lai ding thé ty nhu tren.w... qudan sat su tudn hoan cia cdc sé du khi nang lén lily thita trong cdc vi dh mot cau héi tu nhién dat ra 1a: Phai chang véi cdc sO tu nhién bat ky a va mc ducila phép chia céc ly thita cia a cho m lap lai mot cdc tudn hoan? That vi gidi d4p cau héi tren ta chéng minh khang dinh sau Dinh ly 9. Doi v6i cdc s6 tw nhién a va m tay ¥ cdc sO dit cha phér a,a?,a3,a4,a5,a8.-. cho m lap lai mot cach tudn hoan (c6 thé khong bat d dau). Chiing minh. Ta dy m +1 ly thira déu tien a, a2, a°,...,a,a™ vA xét cde s6 du cita chting khi chia cho m. Vi khi chia cho m chi c6 thé ¢ _ 86 dur {0,1,2,...,m — 2,m — 1}, ma lai c6 m +1 s6, nén trong céc s6 trén ph hai s6 c6 ciing s6 du Khi chia cho m. Chang han, hai s6 dé 1a a* va a*+!, trong dé / Khi d6 km kt a* = ak! (mod m) V6i moi n > k nhan cA hai vé cla phép déng du (1) voi a*-* sé duge antl (mod m) Diéu nay ching td ring bit ddu tir vi tri tuong tmg voi a* cdc sO dur lap lai hoan, tte bat dau tir sé tuong tig véi a* 6 mot s6 du lap lai va lap lai vv... S61 duge goi 18 chu ky tuan ho’n cia céc s6 du khi chia ldy thita cia a che Nhan xét. Tw chimg minh tren nhan thy ring sv tun hoan cha cdc s6 du bat dau ti ¢ phét hién ra hai s6 dur tring nhau. Mat khéc dé phat hien ra hai sO du giéng nha 156 chia cho m ta khong phi quadan tam dén cd sa ma chi cén ly m +1 ly thi diu tien 1a da, Néu t6n tai sO 1, dé a! = 1 (mod m), thi voi moi sO ty nhién n a"! = a” (mod m), nen J ching nhimg 1a chu ky tudn hoan ca cdc s6 du, ma cdn cé thé xem 1a chi sO bét dau sy tun hoan cha céc s6 du. 0.6.2 Thuat toan Dé gidi bai to4n chia hét, cfin xdc dinh s6 du cia lily thita a” chia cho m, ta cn tim céc sO ty nhién k,1 nhé nhat, dé ite ght! a (mod m) Sau d6 can cf vao s6 du r cia n chia cho J, ma x4c dinh s6 du tuong tmg voi a* +", Chi ¥. 1) Trong trudng hop tén tai s6 tr nhién s, dé a*=1 (mod m) ta chi viée tim c4c sO tu nhién nho nhat k, 1 sao cho a* =a*t!=41 (mod m) sau d6 tim s6 dur cia n chia cho J va x4c dinh s6 du cia a” khi chia cho m. Day chinh 1a s6 du cia a” chia cho m. 2) Khi lay thita c6 s6 mo khong phai 14 him tuy€n tinh cia n, chang han, a?() v6i p(n) 1a mot ham mi, ma ta 6 thé thay déi co sO tira sang b, dé c6 aP() = 9”) va b (mod m) oP") = 1%") =1 (mod m) ‘Trutmg hop khong bin déi duoc co sO nhu ten cén tim céch thay déi co s6, dé loy thira c6 s6 mii 1a mot s6 tu nhién r6i tim s6 du nhw thuat todn da neu. Vi du 7. Chimg minh ring so 5° +5 chia het cho 6. Gidi. Ta sé ching minh tring hop téng qué4t: V6i moi s6 ty nhién n s6 58" +5 chia hét cho 6. Do 58" a pBX8"F L gexan8™"? _ (pp) ax™—t Vi 25 = 1 (mod 6), nen 58” = (25)**8""' =1 (mod 6) Mit khéc 5 = 5 (mod 6). Vay 58" +5 = 6 =0 (mod ). Do dé 5%" +5 chia hét cho 6. Thay n = 2004 te 38" +5 chia hét cho 6. Vi du 8. Ching minh ring 148°" +10 chia hét cho 11. Gidi. Tim s6 dur cha 148° + 8 chia cho 11. Do 14 = 3 (mod 11), nén 1 (mod 11). Do 38 = 6561 = 5 (mod 11), nen 38° = 6561204 = 520% (mod 11). ‘Xét cdc s6 du thudc lity thita cha 5 khi chia cho 11 Dn 5. Se 5, 5. oS. Sooo) 5 fd nén 54501 501 = 1501 = 1 (mod 11) Mat khic, 10=10 (mod 11) Cong vé voi vé phép déng du (1) va (2) 06 14" 4 = =0 (mod 11) 32008 Nén 14° +10 chia hét cho 11. Vi du 9. Ching minh ring s6 222555 + 555? chia hét cho 7. Gidi. 1) Do 222 =7 x 31 +5, nén 222 = 5 (mod 7). Bai vay, 222555 = 55° (mod 7) Xét su tudn hoan cia céc s6 du khi chia Iiy thita cla 5 cho 7 ta duge 5 5? Bt st 55 58 57 58 54 62 8315 4 Nhu vay =1 (mod 7) V6i moi s6 ty nhien ¢, nang cé hai vé cia phép déng du (1) len ly thira ¢ 5% =1 (mod 7) 158 Mat khéc 555 = 6.92 + 3, nén 5°55 = 569243 — 5692.53 = § (mod 7). Do d6 222555 =6§ (mod 7) Q) 2) Do 555 = 7.79 + 2, nén 555 = 2 (mod 7). Boi vay, 555%? = 222 (mod 7). Xét sy tudn hoan cba cdc s6 du khi chia ldy thia cia 2 cho 7 ta duge 2) 282 te et 24 1 2 4 2 4 21 Nhu vay 28=1 (mod 7) @) V6i moi s6 ty nhién s nang cd hai vé cha phép déng du (3) len lily s ta c6 2° =1 (mod 7) Mut khéc, 222 = 3.74, nen 5552? = 25x74 =1 (mod 7) (4) Cong vé voi vé cdc phép déng dur (2) va (4) c6 222555 4 555? =6+1=0 (mod 7) Vay s6 222555 + 555222 chia hét cho 7. BAI TAP 15 Chimg minh rang s6 71° + 11! chia hét cho 13. 16 Chimg minh rang s6 6°? + 8 chia hét cho 11. 17 Chimg minh rang s6 11! — 1 chia bét cho 100. 18 Chimg minh ring 77777? — 7 chia hét cho 10. 19 Hay tim chit s6 tan cing cha s6 vg 20 Chimg minh ring s6 141" — 6 chia hét cho 10. 21 Chimg minh rang s6 1120” — 1 chia hét cho 10188, 22 Ching minh ring s6 222553 + 3332? chia hét cho 13. 23 V6i moi s6 nguyén khong 4m chting minh ring s6 gbn+3 4 5n grt chia hét cho 17. 0.7. Phuong phap quy nap Phuong phdp quy nap cé vai trd vo cling qudan trong trong todn hoc, khoa hoc v2 sOng. Doi v6i nhiéu bai todn chia hét, phuong phdp quy nap ciing cho ta céch gi: hieu. Suy dién 1 quaé trinh tir “tinh chat” ca tap thé suy ra “tinh chat” cita c4 the ludn ludn diing, cOn quaé tink nguyc lai, tite quad tinh quy nap: di ti “tinh cia mot s6 cé thé suy ra “tinh chét” cia tap thé, thi khong phi lic nao cing din qué trinh nay chi ding khi né tho man mot s6 diéu kién nao d6, tie tho man n; ly quy nap. 0.7.1 Nguyén ly quy nap Néu khing dinh $(n) thod man hai diéu kien sau a) Ding voi n = ke (sO ty nhién nhé nhat ma $(n) xéc dinh). b) Te tinh ding din cia S(n) d6i v6i n = t (hodc A6i voi moi gid tri c ko ko. 0.7.2. Phuong phap chimg minh bang quy nap Gia sit khang dinh T(n) x4c dinh vi moi n > to. Dé chtmg minh T'(n) ding vé n(n > to) bing quy nap, ta cn thuc hién hai buéc. a. Co so quy nap. Thue hién busc nay tic 1a ta thir xem su ding din cha T(n) voi n = to, ng xét T(to) c6 ding hay khong? b. Quy nap. Gia sit khing dinh T(n) da ding d6i véi n = t (hode d6i Voi moi n (to tp). Tren co s6 gid thiét nay mA suy ra tinh ding dan cla T(n) d6i v6in = nic T(t +1) ding. Néu cd hai buéc tren déu thod man, thi theo nguyén Ly quy T(n) ding voi moi n > to. Chit y. Trong quad trinh quy nap n€u khong thyc hién day di c& hai bu6e quy nap va quy nap, thi c6 thé dan dén ket lun sai lm, chang han: - Do bé butic co sé quy nap, ta dua ra két luan khong diing: Moi s6 ty nhié bing nhau! Bang céch quy nap nhu sau: gid sit cc sO tr nhién khong vuot quid da bing nhau. Khi d6 tacék=k+1. Them vao mdi vé cia dng thitc trén mot don vi sé c6 k+l=k+1+1=k+2 160 Cit nur vay suy ra moi s6 ty nhién khong nhd hon & déu bang nhau. Két hop v6i gid thiét quy nap: moi s6 tu nhién khong vuot quad k déu bing nhau, di dén két lwan sai ldm: Tat ca cdc s6 ty nhién déu bang nhau! - Do bé qua khau quy nap, nén nha Toan hoc Phép P. Fermat (1601 - 1665) da cho ring cdc s6 dang 2?” +1 déu IA s6 nguyén t6. P. Fermat xét 5 s6 dau tien Voi n = 0 cho 22" +1 = 2141=3 1a s6 nguyen 16. V6i n = 1 cho 2” +1= 2? 41=5 18 s6 nguyen 16. V6i n = 2 cho 2” +1=244+1=17 IA sO nguyen 16. V6i n = 3 cho 2° +1 = 28 +1 = 257 IA sO nguyen 16. Voi n = 4 cho 27° + 1 = 216 + 1 = 65537 Ia s6 nguyen 16. Nhung vao thé kj 18 Euler da phat hién véi n = 5 khang dinh tren khong ding boi vi 2° 4.1 = 4294967297 = 641 x 6700417 1a hop s6. 0.7.3 Van dung phuong phap quy nap dé giai cc bai toan chia hét Phuong phap quy nap dugc sit dung trong tinh todn, trong chimg minh ya suy luan du6i nhiéu dang khdc nhau, nhung trong phén nay chi trinh bay vic van dung phuong ph4p quy nap dé gidi cdc bai ton chia het. Vi du 11. Véi n 1a s6 ty nhién, dat a adn Chimg minh rang v6i moi s6 ty nhién n > 2 sO An + 17 chia hét cho 20? Gidi. Dé c6 khang dinh phét biéu trong bai ton truée hét ta ching minh: V6i moi s6 tw nhién n > 2 déu 6 An = 20tn +3 a) Khang dinh (1) dugc ching minh bing quy nap theo n. 1. Co sé quy nap. V6i n =206 Ap = 77 =71,73, ma P = 343 = 20.17+3=20.p+3 Q) Tt = 2401 = 20.120 +1 = 20.q+1, @) Nén Ag = (20.17 + 3)(20.120 + 1) = 20.20.17.120 + 20.17 + 20.120.3 +3 = 20.41177+3 2. Quy nap Gia sir ding thitc (1) da ding v6i n = k > 2, tite da c6 Ax = 208, +3 Cén ching minh ding thie (1) ding voi n =k +1. ‘That vay, theo dinh nghia, va (2), (3), (4) c6 Apgr Tat S 72043 TAStHS — (7H)5te 73 (20g + 1)**(20p + 3) = {(20g)5* + Ch, (209)5—! +--+ + CH-*.20q + 1}(20p + 3) = 20p{(20)%* + Of, (20g)? + «++ + OBgk1.20q + 1} + 20q{(20g)5*-* + Cy, (20g)? + + + CRE} +3 = tay +3 Tur (1) c6: An +17 = ty +3-+17 = 20tn +20 = 20(tn + 1) nén A, + 17 chia hét cho 20. Vi du 12. Ching minh ring voi moi sO tw ohien n > 2 s6 P44 chia hét cho 10 (hay tan cing bing s6 0)? Gidi. Chimg minh bing quy nap theo n. 1. Co sé quy nap. Voi n = 2 c6 2% = 244.4 = 16 +4 = 20 chia hét cho 10. . 2. Quy nap. Gia sit king dinh da ding voi n = k > 2, nghia a 44 = 10te Can ching minh khing dinh ding véin =k +1. 162 That vay, tix (1) c6 2 44 = (9%) 44 = (10% —4)? +4 10tz)? — 8.10t, +1644 = 10(10t2 — 8¢, +2) nén s6 27*** 4.4 chia hét cho 10. Vi du 12. Chimg minh ring voi moi sO tr nhien n A(n) = 4" +15n-1 chia hét cho 9. Gidi. Ching minh bing quy nap theo n. 1. Co sé quy nap. V6i n= 1, A(1) = 41 + 15.1 — 1 = 18 chia hét cho 9. 2. Quy nap. Gia sit khang dinh da ding véi s6 ty nhién n = k > 1, nghia 1a, A(k) =4* + 15k -1 da chia hét cho 9. Cén chtng minh khang dinh cing ding voi n =k +1. ‘That vay, A(k-+1) = 4*#7 4. 15(k +1) —1 = 4.4" 415k4+14 44° 4 4.15k +14 43.15k 4+ 4-3.15k = 4.4" 44.15k —443.15k +18 = 4(4* + 15k — 1) +9(5k +2) Theo gia thi¢t quy nap 4* + 15k —1 chia hét cho 9, nén 4(4* + 15k — 1) chia hét cho 9 va 9(5k +2) chia hét cho 9. Boi vay A(k +1) chia hét cho 9. Vi dy 13. Chimg minh ring téng lap phuong cia ba s6 tw nhien lien tigp bao gity cting chia hét cho 9. Gidi. Chimg minh bing quy nap theo thé ng s6 tr nhién, 1. Co sé quy nap. Véi ba sO ty nhién dau tien 1,2,3 ta c6 B+B4+3 =148427 = 36 chia hét cho 9. 2. Quy nap. Gia sit khing dinh da ding véi ba sO ty nbien lien tiép tly ¥ ndo dé k 1), k+1, k+2, oghia sO A(k) = k3 + (k41)? 4 (b4+2)% da chia hét cho 9. Khi dé (Rt 18 +(k +2)? 4 (+3) =k + (K+ 1) + (bk +2)3 + (kb +3)9—b Do (b +3)? —k = (k+3—k){(k +3)? + k(k +3) +7} = 3{k? 46k 494k? + 3k +4 k?} = 3{8k? + 9k + 9} = 9(k? +3k +3) chia hét cho 9, va theo gia thiét quy nap A(k):9, nén (k + 1)° + (k +2)3 +(k chia hét cho 9. Khang dinh dugc chtmg minh. Vi dy 14. Chimg minh ring véi moi sé nguyen n > 0 so 23" +1 chia he 3+! va khong chia hét cho 3°+2, Gidi. Khang dinh duge chtmg minh bing quy nap theo n. 1. Co sé quy nap Voi n= 0 86 2° 41 =2!41=241=3 chia hét cho 3 (3 = 3°) val chia hét cho 9 (9 = 3? = 3°#?), Voi n= 1 s6 2% +1 =29+1=8+4+1=9 chia het cho 9. 2. Quy nap Gid sit khing dinh da ding voi n= k > 2, nghia fA Apa 241 164 a chia hét cho 3*+1 va khong chia hét cho 3*+2, Khi d6 t6n tai s6 nguyen M, dé Ag = M3" va do Aj, khong chia hét cho 3**?, nen M khong chia hét cho 3. Can ching minh Aj41 = 2°*** +1 chia hét cho 3*+? va khong chia hét cho 3*+3, That vay, do Aw = 2 4158" 415 (2) 41 = (2 +1){2") - 2" +1} = 3+ yy q(2%*)? + 2.25 +1 - 3.28} = 31 {(2 +1)? - 3.2%} = 31M {(2"".M)? - 3.27} = SEH yy {gtk+ yy? — 3.25*} = BF? Pet ag? — 2} nén Ax41 chia hét cho 3**?, Vi M khong chia hét cho 3, nén 3*+?.M khong chia hét cho 3*+3, Do k > 2, nen k — 2 > 0, va c6 geht ng = gkt3+k-2 yy = 343 gh-2 ay chia hét cho 3*+3, Mat khéc 23" = (23)* = 8* = +1 (mod 9) va 3*+3 = 9.3*+1, nen 2" khong chia hét cho 3*+3, Do 3?*+1_ 4? — 23" khong chia hét cho 3**3, Boi vay Ax41 khong chia hét cho 3*+2. Khang dinh dugc chtmg minh. BAI TAP 23 Chimg minh ring véi moi s6 nguyén n > 0 a) (25"+3 + 5".3"+?) chia hét cho 17 b) (2"+8.34 4. 58"+1) chia hét cho 37 c) (541 4 2"+4 4 2741) chia hét cho 23 d) (77+? + 8241) chia hét cho 57 24 Ching minh ring v6i moi s6 ty nhien n téng 204 22 4 22 4... 25R-3 4 yom? 4 gn—I chia hét cho 31? 25 Gid sit a 1A sO ty nhién no dé, ma 2° — 2 chia hét cho a, ching han a = 23 —2 = 8-2 =6 chia hét cho 3. Xéc dinh day 6 (,) ahd cdc diéu kic Zy=a, Tey =2*—1 Chimg minh ring véi moi s6 tr nhien k s6 27* — 2 chia hét cho 24? 0.8 Tiéu chuan chia hét DOi voi sé nguyen tuy ¥ a va sO ty nhien bat ky m dé wd Idi cau hdi: a c6 ch cho m khong? Trong rat nhiéu trudng hop cé thé dua vao titu chudn chia het. Be vigc tim ra c4c tiéu chudn chia hét dé van dung 1a hét stte cn thiét. Can cit va chat cia day so du nbn dugc Khi chia ldy thita co sO 10 cho m, ma 6 thé xé cée titu chudn chia hét tign ich khdc nhau. Trong phan nay trinh bay mot s6 cdc dinh tieu chudn chia hét, nhu Phuong ph4p sit dung tinh déng du v6i 1 cia ly thita co s6 10, ma goi “phuong php déng du véi 1”. Phuong php dya vao day sO du cia Idy thita co sO 10, ma goi tat 1a "pl phap day s6 du”. Phuong phap chia cdc chit s6 thanh cdc nhém, ma goi tét la "phuong php chit so”, 0.8.1 Phuong phap déng du voi 1 V6i s6 ty nhién tuy ¥ m > 2 cén tim tiéu chudn, dé sé nguyen bat ky a= Ondn, 140 = anl0" + an—110"7! +--+ +.a;10' + aj_110*! + --- +0410 +. a9 chia hét cho m. Néu t6n tai sO tg nhien &, dé 10" = 1 (mod m) thi v6i moi sO ty nhien t d 10 = 1 (mod m). Ta thuc hién thuat todn sau Thuat toan. 1. Tim s6 ty nhién J nhé nhit, dé 10! = 1 (mod m). 2. Chia day chit s6 cla a tir phi sang tri theo cdc nh6m lién tiép do dail. K v6i s6 tr nhién s, ma sl 2, chang han, m = 4,7,11, 13. 1. Tiéu chuan chia hét cho 4 Xét tinh déng dur cba Idy thita co s6 10 theo modun 4 ta c6 10=2 (mod 4), 10?=20=0 (mod 4), 10'=0 (mod 4) (i=3,4...) Téng d tung tmg véi s6 a = Ga_ay ge -aRAIaH C6 dang d= ay0 + an—10 + +> +4410 + 040 + - +» + 020 + 212 + a9 = 20; + a9 Vay tiéu chudn chia hét cho 4 1a: SO a chia hét cho 4 khi va chi khi d= 2a; +a chia hét cho 4. : Vidu. 453452 c6 d= 2x 5+2=12 chia hét cho 4, nen 4534524, 582422 cé d = 2x2+2 = 6 khong chia hét cho 4, nén 582422 khong chia hét 2. Tiéu chuan chia hét cho 7 Xét tinh déng du cia tidy thita co sO 10 theo modun 7 ta c6 10=3 (mod 7), 10?=30=2 (mod 7), 10? =20 10°=-10=-3 (mod 7), 10° = 1 (mod 7) 30=-2 (mod 7), 10°=-20=1 ( Gia sit n = 6¢ +1 v6i t > 2. Khi d6 day s6 déng du tuong ting voi day « cha a sé 1a Qn Qn-1 Qn—2 Gn—3 ++. 0g 5 a4 a3 a2 a1 a9 1-2-3 -1..1-2-3-1231 Va tng d tuong tmg voi a c6 dang d= an — 2an—1 — Ban—2 — Gn_g +++ +06 ~ 2as — Bag — a3 + 2ap + Bay + Vay tieu chudn chia hét cho 7 1&: SO a7 Khi va chi khi tng d =n — 2an_1 — 3an_2 — Qn-g +++* + 0g — 2as — 3a4 — ag + 2a2 + 3a) 4 chia hét cho 7, . 7546357 c6 d= 7-2x5-3x4-64+2x3+3x5+7=7 chia he nén 7546357:7. 863425 06 d=-2x8-3x6-34+2x44+2x3+5=-16-18-—3+8+6+5= khong chia hét cho 7, nén 863425 khong chia hét cho 7. 3. Tiéu chudn chia hét cho 11 Xét tinh déng du cia lay thita co's 10 theo mod 11, ta cé 10=-1 (mod 11), 10?=-10=1 (mod 11), 107+! = —1 (mod 11), 107*=1 (mod 170 V6i k = 0,1,2,... khi d6 day d6ng du tong img véi day chit s6 cia a sé IA Gn,» Gn—-1, An-2, 42 41 a9 (-4)"(-1)"71(-1)"F. = 1 Va tng d tuong tng v6i a c6 dang d(—1)"an + (=1)""?an—1 + (—1)"Pan-2 + +++ + a9 — ay +09 . Vay tieu chudn chia hét cho 11 1a: SO a'11 khi va chi khi d= (-1)"aq + (—1)" Tana + (-1)" Pana + ++ +09 ~ a; +9 chia hét cho 11. Vi du. 3811581939 c6 d = -3+8~1-5+8-1+4+9~3+49 = 22 chia hét cho 11, nén 3811581939 :11 ; 256743258 c6 d=2-—5+6-—7+4—3+2-—5+48= 2 khong chia hét cho 11, nén 256743258 khong chia hét cho 11. 4. Tiéu chuan chia hét cho 13 Xét tinh déng du cia IDy thita co s6 10 theo modun 13 ta c6 10 = -3 (mod 13), 10? = —4 (mod.13), 10° = —40 = —1 (mod 13), 104 = —10 = 3 (mod 13), 10° = 30 = 4 (mod 13), 10° = 40 =1 (mod 13), 107 =10 = -3 (mod 13) Gia sit n = 6¢ v6i t > 2. Khi dé day s6 déng du tuong tng v6i day chit s6 cia a a Qn Gn} On-2 Gn-3 An—4 Gn-5 + AG 5 G4 G3 a2 a ag a 3 -1 -4 -3 1 4 3 -1 -4 -3 1 va tng d tuong tmg voi a cé dang d= On + 4dn—1 + 3an—2 — Gn—3 — 4dn—4 — 3an—5 + °° +.ag + das + 3a4 — a3 — day — 3a; +09 Vay tieu chudn chia hét cho 13 1: $6 a chia hét cho 13 khi va chi khi tng d= On + 4an—1 + 3an—2 — On—3 — AAn—4 — 3an_5 + °° +g + 4as + 3a4 — a3 — 402 — 3a; + ag chia hét cho 13. Vidu. a 8588112 c6 d= 8 + 20+ 24-8 -—4—3+42 = 39 chia hét cho 13, nen 85 chia hét cho 13. 111111 c6 d=1+4+3-—1~4~3+41=1 khong chia hét cho 13, nén 11 Khong chia hét cho 13. 0.8.3 Phuong phap nhém chit sé Gia sit a = aaa, 7a IA s6 nguyen ty ¥, cdn m IA sO ug nhien bat ky ahd hon 2. V6i sO tw nhién J tuy ¥ khong nhé hon 2. Gia sit d, IA so nguyen dé v6i 10 (i = 0, 1,2,) theo modun m va c6 tri tuyét d6i nhé nhdt. Khi d6 nhy ca chat cia phép déng dur ta c6, he qua sau: 6 ayaa yaG BAO TOGO! + 10 + Baa H + G71 42 Dua vao hé qua trén c6 thuat ton xay dung tiéu chudn chia hét cho m nhu sau Thuat toan. 1. Chon s6 wr nhién bé nt c6 thé J > 2 thich hop véi m theo nghia: SO d; dut v6i 10 theo modun m.cé tri tuyet d6ibé nha. 2. Liet ke day céc s6 déng du tong img voi day 10! (i = 1,2,) rot 10D! ig. 10% 10! 10° Gp de 3. Lap téng d = TGn—1 Gade + G—10-2--.AG—yide-1 ++ + aad, + a 4, Néu d chia hét cho m, thi a chia hét cho m. Néu d khong chia hét cho 7 . a khong chia hét cho m. Bang thuat ton trén ta cé tiéu chudn chia hét c : bing phuong phép nhém chit s6 nhu sau: S6 a chia hét cho m khi va ct téng d= Gdn Gd; + Hi—-1G—2-- Ge yide—-1 + + A= 103— ay ai dy + Gj-7ay—2...0y 172 chia hét cho m. ‘Dya vao thuat todn trén ta 06 thé xay dung ti¢u chun chia hét cho bat ky s6 tu nhién m nao khong nhé hon 2, ching han m = 7, 33. Tiéu chudn chia hét cho 7 Dé cé tiéu chudn chia hét cho 7 ta thyc hién c4c bude cha thuat todn trén nhu sau 1. Do 1000 = —1 (mod 7), 1000? = 1 (mod 7), 10007 =1 (mod 7), 10007! = —1 (mod 7) i =0,1,2, nén chon 1 = 3. 2. Day s6 déng du tung tmg voi 1000*, & = 0,1,2, 1000' 1000"? ... 1000? 1000 1 (-1)' (-I)e 1 1 1 3. Téng d tong tng véi sO a cé dang d= (-1)'Gndnmidae + (-1) aga aaGy ++ — pages + TyaTAS Khi d6 tiéu chudn chia hét cho 7 duge phat biéu nhu sau: S6 a chia hét cho 7 khi va chi khi téng d chia hét cho 7. Vi du. 5781139 cé d= 5 — 781 + 139 = 637 chia hét cho 7, nén 5781139 chia hét cho 7 811582 c6 d= —811 + 582 = 229 khong chia hét cho 7, nén 811582 khong chia hét a Tiéu chun chia hét cho 33 Dé cé6 tiéu chudn chia hét cho 33 ta thuc hién cdc bude cia thuat todn trén nhu sau 1. Do 100 = 1 (mod 33), nén véi moi s = 0,1,2... déu c6 100° = 1 (mod 33) nén chon | = 2. 2. Day s6 déng du tuong tmg véi 100", k= 0,1,2 oot 1002... 1007 = 100-1 1 1 at lol 3. Téng d tuong img véi s6 a c6 dang d = G,G,] + Gn—20y-3 +--+ WZGQ+ Aap, néunlé d= an + Gciimca +++ + Weae + Aap, néu n chin. Khi d6 tiéu chudn chia hét cho 33 duge phat biéu nhu sau: S6 a chia hét cho ¢ va chi khi d chia hét cho 33. Vi du. 6021939 c6 d = 6 + 02 + 19 + 39 = 66 chia hét cho 33, nén 6021939 ch cho 33. 524631 c6 d = 52 + 46 + 31 = 129 khong chia hét cho 33, nén 524531 khong hét cho 33.

You might also like